You are on page 1of 75

RPT 05

1. भारतीय संविधान को जीिंत दस्तािेज कहा जाता है, इसका 1. Indian Constitution is described as 'a living'
वनम्न में से कौन-सा सबसे उपयुक्त कारण है? document, Which of the following is the most
(a) संविधान में संशोधन करने की कार्यपालिका की शलि। appropriate reason for this?
(a) Executives have more say in amendment of
(b) भारतीर् समाज में दीर्यकाि से व्याप्त धमयवनरपेक्षता का
the Constitution.
संविधान में दृविगोचर होना।
(b) Long-living tradition of secularism in Indian
(c) पररस्थिवतर्ों में पररितयन एिं आिश्र्कता के अनुसार भारतीर्
society is reflected in the Constitution.
संविधान द्वारा थिर्ं को समार्ोजजत करने हेतु संशोधन को (c) Indian constitution allows its amendment in
अनुमवत प्रदान करना। order to adjust itself to changing conditions
(d) उपर्युक्त में से कोई नहीं। and needs.
[c] (d) None of the above
व्याख्या: [c]
- र्द्यवप विधानसभा में कार्यपालिका को अधधक शलिर्ां प्राप्त है, Explanation:
ककिंतु संविधान संशोधन के लिए संसदीर् अनुमोदन आिश्र्क होता - Although executives have greater say in the
legislature but for Constitution amendment
है।
Parliamentary approval is required.
- संविधान के जीिंत दथतािेज कहिाने से धमयवनरपेक्षता की परंपरा
- Tradition of secularism has nothing to do with
का कोई संबंध नहीं है।
the Constitution being called 'a living' document.
- भारतीर् संविधान एक गत्र्ात्मक संविधान है जो समाज, राजनीवत - Indian Constitution is a dynamic Constitution and
ि अियव्यिथिा की प्रकृवत में पररितयन को दशायता है, इसलिए therefore is called 'a living' document, that
"जीिंत दथतािेज" कहिाता है। र्ह तथ्र् वक संविधान को 100 से reflects the changing nature of society, polity and
अधधक बार संशोधधत वकर्ा जा चुका है, थिर्ं इस विशेषता का economy. The fact that the Constitution has been
साक्ष्र् प्रथतुत करता है। संसद आिश्र्कता के अनुसार संविधान में amended more than 100 times is itself a
संशोधन कर सकती है। testimony of this feature. Parliament can amend
the Constitution according to the needs of the
time.
2. वनम्नलिखित कथनों पर विचार कीजजए - 2. Consider the following statements -
I. संविधान द्वारा विधावर्का और कार्यपालिका के शलिर्ों का I. The constitution establishes the complete
पूर्य पृिक्करर् वकर्ा गर्ा है। separation of power between the legislature
and the Executive.
II. विटिश संसद के समान, भारतीर् संसद भी संप्रभु वनकार् है।
II. Indian Parliament is a sovereign body like
III. कार्यपालिका की विधावर्का के प्रवत सामूवहक उत्तरदावर्त्ि
the British Parliament.
भारत सरकार के संसदीर् थिरूप की मुख्र् विशेषता हैं।
III. Collective responsibility of the executive to
उपययुक्त में से कौनसा/से कथन सत्य है/हैं? the legislature is one of the prominent
(a) केिि I और III features of the Parliamentary form of
(b) केिि II और III Government in India.
(c) केिि III Which of the above statement(s) is/are true?
(d) उपरोि सभी (a) I and III only
[c] (b) II and III only
व्याख्या: (c) III only
(d) All of the above
- भारत में सरकार का संसदीर् स्वरूप विद्यमान है।
[c]
- विधावर्का और कार्यपालिका के मध्र् समन्िर् ि सहर्ोग, संसदीर्
Explanation:
प्रर्ािी की प्रमुख विशेषता है।
- India has a parliamentary form of government.
- कार्यपालिका अिायत प्रधानमंत्री सहित संपूर्य मंत्री पररषद् भी One of the essential features of the
विधावर्का का अंग होती है। Parliamentary system is the cooperative
- संविधान में अनुबंधधत वकर्ा गर्ा है वक कोई मंत्री िगातार 6 महीने relationship between the legislature and the
तक संसद का सदथर् नहीं रहता िै तो िह उस अिधध की समाप्प्त executive. The executives like Prime Minister
पर मंत्री नहीं रहेगा। along with the Council of Ministers are part of
- भारतीर् संसद, विटिश संसद के समान संप्रभु वनकार् नहीं है। इसे legislation also. The constitution also stipulated
संविधान द्वारा वनधायररत अधधकार क्षेत्र के तहत् कार्य करना होता है। that minister who is not a member of the

-:: 1 ::-
RPT 05
- भारत का सिोच्च न्र्ार्ािर्, संसद द्वारा पाररत वकसी कानून को parliament for the period of six consecutive
असंिैधावनक र्ोवषत कर सकता है र्टद िह संविधान के प्रािधानों months ceases to be a minister.
का उल्िंर्न करता हो। - Indian Parliament is not a sovereign body like the
British Parliament.
- संविधान में िर्र्िंत मूि अधधकार विधावर्का के कानून विर्ान्िर्न
- It has to operate within jurisdiction earmarked
पर तानाशाही को मर्ायटदत करते हैं।
for it by the Constitution.
- अनुच्छे द 75 के तहत् मंत्रीपररषद, िोकसभा के प्रवत सामूवहक रूप
- The Supreme Court can declare a law passed by
से उत्तरदार्ी होगी। भारत में सरकार तब तक बनी रहती है जब तक Parliament unconstitutional if it contravenes the
उसे िोकसभा में बहुमत प्राप्त होता है। िोकसभा को सरकार provision of the Constitution.
(मंवत्रपररषद) के विरुद्ध अविश्वास प्रथताि पाररत करने की शलि - fundamental rights in the constitution acts as a
प्राप्त है। सामूवहक उत्तरदावर्त्ि लसद्धांत का अिय है मंवत्रमंडि के check on its authority.
वनर्यर् सभी केंद्रीर् मंवत्रर्ों के लिए बाध्र्कारी है, भिे ही मंवत्रमंडि - Article 75 says the council of ministers are
की बैठक में उनके विचार इसके विरुद्ध ही क्र्ों ना हो । र्टद कोई collectively responsible to the Lok Sabha. In
मंत्री मंवत्रमंडि के वकसी वनर्यर् से असहमत है और उसके लिए India, the government survives till the time it
enjoys support of the majority of members in the
तैर्ार नहीं है तो उसे त्र्ाग पत्र दे ना होगा।
Lok Sabha. Lok Sabha is empowered to introduce
a no-confidence motion against the government.
Collective responsibility also means that Cabinet
decisions are binding on all ministers even if they
have different opinions. If the minister disagrees
with cabinet decisions and did not defend it
he/she must resign.
3. वनम्नलिखित में से कौन-से उपबंधों को संशोधधत करने हेतु 3. Which of the following provisions require
संसद के विशेष बहुमत के साथ-साथ, आधे राज्यों का special majority of the Parliament along with
अनुसमथथन आिश्यक है? the ratification of states for their
amendment?
I. नागररकता
I. Citizenship
II. सिोच्च न्र्ार्ािर् के अधधकार क्षेत्र में िृजद्ध
II. Enlarging the Jurisdiction of the Supreme
III. अनुच्छे द 368
Court.
IV. राजभाषा III. Article 368
V. सातिीं अनुसूची से संबंधधत कोई विषर् IV. Official Language
VI. िथतु एिं सेिा कर पररषद V. Any of the list in Seventh Schedule
उपययुक्त में से कौनसा/से कथन सत्य है/हैं? VI. Goods and Services Tax Council
(a) केिि I, II और IV Which of the above statement(s) is/are true?
(b) केिि III, V और VI (a) I, II and IV only
(c) केिि I, III, V और VI (b) III, V and VI only
(c) I, III, V and VI only
(d) केिि II, III, V और VI
(d) II, III, V and VI only
[b]
[b]
व्याख्या:
Explanation:
- वैसे उपबंध जजन्हें संशोधधत करने हेतु संसद के विशेष बहुमत के - Provisions that require special majority of the
साि आधे राज्र्ों का अनुसमियन आिश्र्क है, वनम्न है: Parliament and ratification by half of the States
- संविधान को संशोधधत करने की शलि (अनुच्छे द 368 थिर्ं) include:
- राष्ट्रपवत का वनिायचन एिं इसकी प्रविर्ा। - Power to amend the constitution (Article 368)
- केंद्र एिं राज्र् के बीच विधार्ी शलिर्ों का विभाजन। itself.
- केंद्र एिं राज्र् कार्यकाररर्ी की शलिर्ों का विथतार। - Election of the President and the manner of
- GST पररषद। election.
- Extent of executive powers of the Centre and
- सातिीं अनुसूची।
States.
- संसद में राज्र्ों का प्रवतवनधधत्ि।
- Distribution of legislative powers between the
- उच्चतम न्र्ार्ािर् एिं उच्च न्र्ार्ािर्।
Centre and the States.
- GST Council
-:: 2 ::-
RPT 05
- नागररकता, राजभाषा और सिोच्च न्र्ार्ािर् के अधधकार क्षेत्र में Seventh Schedule
िृजद्ध से संबंधधत उपबंधो को संसद के साधारर् बहुमत द्वारा - Representation of States in the Parliament.
संशोधधत वकर्ा जा सकता है। इस प्रकार के संशोधन अनुच्छे द 368 Supreme Court and High Courts.
- Provisions regarding Citizenship, Official
की पररधध में नहीं आते।
Language and Enlarging the jurisdiction of the
Supreme Court can be amended by the simple
majority of the Parliament. Such amendments do
not come under the purview of Article 368.
4. भारत की प्रस्तािना भारत के सभी नागररकों के लिए स्स्थवत 4. The Preamble of India secures equality of
और अिसर की समानता को सुरक्षित करती है। वनम्नलिखित status and opportunity to all the citizens of
प्रािधानों में से संविधान का कौन सा प्रािधान समानता के India. Which of the following provisions of the
Constitution is based on Right to equality?
अधधकार पर आधाररत है?
I. Equality before law
I. विधध के समक्ष समानता
II. Prohibition of Human trafficking
II. मानि तथकरी का प्रवतषेध
III. Abolition of untouchability
III. अथपृश्र्ता का उन्मूिन IV. Prohibition of discrimination on Certain
IV. वकन्हीं आधारों पर विभेद का प्रवतषेध Grounds
V. अपनी मजी से वििाह करने का अधधकार V. Right to marry by one's choice
उपययुक्त में से कौनसा/से कथन सत्य है/हैं? Which of the above statement(s) is/are true?
(a) केिि I और V (a) I and V only
(b) केिि I, III और IV (b) I, III and IV only
(c) केिि II और IV (c) II and IV only
(d) All of the above
(d) उपरोि सभी
[b]
[b]
Explanation:
व्याख्या:
- The term ‘equality’ means the absence of special
- समाज के वकसी भी िगय के लिए विशेष सुविधाओं की अनुपस्थिवत privileges to any section of the society, and the
और सभी व्यलिर्ों के लिए वबना वकसी भेदभाि के पर्ायप्त अिसरों provision of adequate opportunities for all
का प्रािधान ही 'समानता' कहिाता है। विधध के समक्ष समानता individuals without any discrimination. The
(अनुच्छे द 14), धमय, मूििंश, जावत, लििंग र्ा जन्मथिान के आधार provisions in the Constitution embraces three
पर विभेद का प्रवतषेध (अनुच्छे द 15), िोक वनर्ोजन के विषर् में dimensions of equality–civic, political and
अिसर की समानता (अनुच्छे द 16), अथपृश्र्ता का उन्मूिन economic which are Equality before the law
(अनुच्छे द 17) तिा उपाधधर्ों का उन्मूिन (अनुच्छे द 18) संविधान (Article 14), Prohibition of discrimination on
grounds of religion, race, caste, sex or place of
के िो प्रािधान हैं जजनमें समानता के तीनो पहिू शाधमि हैं।
birth (Article 15), 4 Equality of opportunity in
नागररक, राजनीवतक तिा आर्ििंक शोषर् के विरुद्ध अधधकार में
matters of public employment (Article 16),
मानि तथकरी के प्रवतषेध (अनुच्छे द 23) का प्रािधान है और वििाह
Abolition of untouchability (Article 17), Abolition
का अधधकार, प्रार् और थितंत्रता का अधधकार (अनुच्छे द 21) का of titles (Article 18). Right against exploitation
विथतृत रूप से थपिीकरर् करता है। provides for Prohibition of Human trafficking
(Article 23) and Right to marry is wider
interpretation of (Article 21), Right to Life and
Liberty.
5. संविधान सभा की विधायी शलक्तयों पर विचार कीजिए - 5. Consider the following statements about
I. विधार्ी शलिर्ााँ भारतीर् थितंत्रता अधधवनर्म, 1947 द्वारा दी legislative powers of Constituent Assembly -
गई िीं। I. The legislative powers were given by the
Indian Independence Act, 1947.
II. संविधान वनमायर् और विधान का दोहरा कार्य संविधान के
II. The dual task of Constitution making and
प्रारंभ से अंत तक जारी रहा।
legislation continued till the commencement
III. संविधान सभा 26 जनिरी, 1950 से एक अथिार्ी संसद के
of the Constitution.
रूप में जारी रही। III. The Constituent Assembly continued as a
उपययुक्त में से कौन सा/से कथन सही है/हैं? provisional Parliament from January 26,
(a) केिि I और III 1950.
-:: 3 ::-
RPT 05
(b) केिि II Which of the above statement(s) is/are true?
(c) केिि II और III (a) I and III only
(d) केिि I और II (b) II only
(c) II and III only
[a]
(d) I and II only
व्याख्या:
[a]
- भारतीर् थितंत्रता अधधवनर्म, 1947 ने संविधान सभा को नए
Explanation:
संविधान के प्रारूप और िागू होने तक कानून बनाने का अधधकार - The Indian Independence Act, 1947 empowered
टदर्ा। 15 अगथत, 1947 के बाद पाररत विटिश संसद का कोई the Constituent Assembly to legislate till the new
अधधवनर्म भारत के डोधमवनर्न तक विथताररत नहीं िा, जब तक Constitution is drafted and enforced. No Act of
वक इसे विधावर्का के कानून द्वारा विथताररत नहीं वकर्ा गर्ा। the British Parliament passed after August 15,
- संविधान और कानून बनाने का दोहरा कार्य, 26 निंबर, 1949 1947 was to extend to Dominion of India, unless
(संविधान को अपनाने की वतलि) तक जारी रहा, जब तक संविधान it was extended thereto by a law of the
बनाने का कार्य समाप्त नहीं हुआ। legislature.
- The dual task of making the Constitution and
- 26 जनिरी, 1950 को संविधान सभा ने अपना अंवतम सत्र
legislation continued till November 26, 1949 (the
आर्ोजजत वकर्ा, जबवक 26 जनिरी, 1950 से 1951-52 में पहिे
date of adoption of the Constitution), when the
आम चुनािों के बाद नई संसद के गठन तक भारत की अंतररम संसद
task of making the Constitution was over.
के रूप में कार्ु हकर्ा। - On January 24, 1950, the Constituent Assembly
held its final session. It did not end and continued
as the provisional parliament of India from
January 26, 1950 till the formation of new
Parliament after the first general elections in
1951-52.
6. संविधान के अनुच्छे द 32 के तहत् 'संिैधावनक उपचारों के 6. Which of the following statements about
अधधकार' के बारे में वनम्नलिखित में से कौन सा कथन सही 'Right to Constitutional Remedies' under
है/हैं? Article 32 of the Constitution is/are correct?
I. It provides for the right to move the Supreme
I. र्ह मौलिक अधधकारों के प्रितयन हेतु सिोच्च न्र्ार्ािर् और
Court and High Courts for enforcement of
उच्च न्र्ार्ािर्ों में जाने का अधधकार प्रदान करता है।
Fundamental Rights.
II. सिोच्च न्र्ार्ािर् में जाने का अधधकार वकसी भी पररस्थिवत
II. The right to move Supreme Court cannot be
में वनिंवबत नहीं वकर्ा जा सकता है, हािांवक र्ह उच्च suspended in any circumstance, however
न्र्ार्ािर्ों के संबंध में वनिंवबत वकर्ा जा सकता है। this can be done with respect to High Courts.
उपययुक्त में से कौनसा/से कथन सत्य है/हैं? Which of the above statement(s) is/are true?
(a) केिि I (a) I only
(b) केिि II (b) II only
(c) I और II दोनों (c) Both I and II
(d) न तो I, न िी II (d) Neither I nor II
[d]
[d]
Explanation:
व्याख्या:
- Article 32 provides remedies for enforcement of
- अनुच्छे द 32, संविधान के भाग III द्वारा प्रदत्त मौलिक अधधकारों
fundamental rights conferred by Part III of the
के प्रितयन के लिए उपार् / उपचार प्रदान करता है। Constitution.
- अनुच्छे द 32 (1) मौलिक अधधकारों के प्रितयन के लिए उधचत - Article 32(1) provides that the right to move the
कार्यिाही के अनुसार सिोच्च न्र्ार्ािर् मे जाने का अधधकार प्रदान Supreme Court by appropriate proceedings for
करता है। कोई व्यलि संविधान के अनुच्छे द 226 के तहत् उच्च the enforcement of the rights conferred by this
न्र्ार्ािर्ों में जा सकता है। Part. A person can move to High Courts under
- अनुच्छे द 32 (4) के तहत् सिोच्च न्र्ार्ािर् जाने का अधधकार Article 226 of the Constitution.
(संविधान के अंतगयत प्रदत्त अनुच्छे द के अिािा) वनिंवबत नहीं - Article 32(4) provides that the right to move the
Supreme Court shall not be suspended except as
वकर्ा जा सकता है। र्ह वनिंबन संविधान के अनुच्छे द 358 और
otherwise provided for by this Constitution. This
359 के तहत् प्रदान वकर्ा गर्ा है।

-:: 4 ::-
RPT 05
suspension is provided under Article 358 and 359
of the Constitution.
7. संविधान के अनुच्छे द-30 (लशिा संस्थानों की स्थापना और 7. Consider the following statements about
प्रशासन के लिए अल्पसंख्यकों का अधधकार) के बारे में Article 30 (Right of minorities to establish and
वनम्नलिखित कथनों पर विचार कीजिए - administer educational institutions) of the
Constitution:
I. धार्मिंक और भाषाई अल्पसंख्र्कों को उनकी पसंद के लशक्षर्
I. Religious and linguistic minorities are
संथिानों की थिापना और प्रशासन का अधधकार प्रदान वकर्ा
provided the right to establish and
गर्ा है।
administer educational institutions of their
II. र्ह अधधकार नागररकों के साि-साि गैर-नागररकों के लिए भी choice.
उपिब्ध है। II. The right is available to citizens as well as
III. र्ह अल्पसंख्र्क शैक्षणर्क संथिानों के लिए एक राष्ट्रीर् non-citizens.
आर्ोग की थिापना का प्रावधान करता है। III. It provides for the establishment of a
उपययुक्त में से कौनसा/से कथन सत्य है/हैं? National Commission for Minority
(a) केिि I Educational Institutions.
(b) केिि II और III Which of the above statement(s) is/are true?
(a) I only
(c) केिि I और III
(b) II and III only
(d) केिि I और II
(c) I and III only
[a]
(d) I and II only
व्याख्या: [a]
- अनुच्छे द 30 धार्मिंक र्ा भाषाई अल्पसंख्र्कों को उनकी पसंद के Explanation:
शैणक्षक संथिानों की थिापना और प्रशासन का अधधकार दे ता है। - Article 30 grants religious or linguistic minorities
- अनुच्छे द 15, 16, 19, 29 और 30 के तहत् प्रदान वकए गए मौलिक the right to establish and administer educational
अधधकार केिि नागररकों के लिए उपिब्ध हैं विदे लशर्ों/गैर institutions of their choice.
नागररकों के लिए नही। - Fundamental Rights provided under Article 15,
- अनुच्छे द 30 के तहत् अल्पसंख्र्क शैणक्षक संथिानों के लिए एक 16, 19, 29 and 30 are only available to citizens and
denied to foreigners.
राष्ट्रीर् आर्ोग की थिापना का कोई प्रािधान नहीं है। र्ह 2004 में
- There is no provision for establishment of a
संसद के एक अधधवनर्म के माध्र्म से थिावपत एक िैधावनक
National Commission for Minority Educational
वनकार् है।
Institutions under Article 30. It is a statutory body
established through an Act of Parliament in 2004.
8. भारतीय संविधान के ‘अनुच्छे द 19’ के बारे में वनम्नलिखित 8. Consider the following statement(s)about
कथनों पर विचार कीजिए - "Article 19" of the Indian Constitution:
I. र्े अधधकार केिि राज्र् के प्रवतकूि हैं ना वक वकसी व्यलि के I. These rights are protected only against the
State and not against private individuals.
प्रवतकूि।
II. Reasonable restrictions can be put on all the
II. साियजवनक आदे श, शािीनता र्ा नैवतकता, न्र्ार्ािर् की
freedoms on the grounds of public order,
अिमानना, मानहावन और अपमान के आधार पर सभी
decency or morality, contempt of court,
थितंत्रता पर उधचत प्रवतबंध िगाए जा सकते हैं। defamation and incitement of an offence.
III. कोई आपलत्त नहीं की जा सकती जब राज्र् नागररकों को III. No objection can be made when the State
छोड़कर वकसी व्यापार, व्यिसार्, उद्योग र्ा सेिा पर carries on a trade, business, industry or
एकाधधकार के रूप में काम करता है। service either as a monopoly to the exclusion
उपययुक्त में से कौनसा/से कथन सत्य है/हैं? of citizens.
(a) केिि I तिा II Which of the above statement(s) is/are true?
(b) केिि I तिा III (a) I and II only
(b) I and III only
(c) केिि I
(c) I only
(d) केिि II तिा III
(d) II and III only
[b]
[b]
व्याख्या: Explanation:

-:: 5 ::-
RPT 05
- र्े अधधकार केिि राज्र् के प्रवतकूि हैं ना वक वकसी वनजी व्यलि - These rights are only against the state and not
के प्रवतकूि। against any private person
- अनुच्छे द 19 थिर्ं इस अनुच्छे द के तहत् प्रदान वकए गए अधधकारों - Article 19 itself defines reasonable restrictions to
be imposed on the rights provided under this
पर िगाए गए उधचत प्रवतबंधों को पररभावषत करता है। र्े वनम्न है-
Article. This is as follows-
- स्ितन्त्रता पर उधचत प्रवतबंध
- Freedom Reasonable Restrictions
- भाषर् और अणभव्यलि की थितंत्रता भारत की संप्रभुता और
- Freedom of speech and expression, sovereignty
अखंडता, राज्र् की सुरक्षा, विदे शी राज्र्ों के साि मैत्रीपूर्य संबंध, and integrity of India, security of the State,
साियजवनक व्यिथिा, शािीनता र्ा नैवतकता र्ा न्र्ार्ािर् की friendly relations with foreign states, public
अिमानना के संबंध में, मानहावन र्ा अपमान के लिए। order, decency or morality, or contempt of court,
- सभा की थितंत्रता भारत की संप्रभुता और अखंडता र्ा साियजवनक for defamation or insult
व्यिथिा। - Freedom of assembly Sovereignty and integrity
- संर् की थितंत्रता भारत की संप्रभुता और अखंडता र्ा साियजवनक of India or Public order
व्यिथिा र्ा नैवतकता। - Freedom of association Sovereignty and integrity
of India Public order or morality
- संचरर् की थितंत्रता और वनिास की थितंत्रता आम जनता के वहतों
Freedom of movement and freedom of residence
में र्ा वकसी अनुसूधचत जनजावत के वहतों की रक्षा के लिए।
in the interests of the general public or to protect
- अनुच्छे द 19 (6), अनुच्छे द 19 (1) (जी) के लिए अपिाद है वक
the interests of any Scheduled Tribe.
कोई आपलत्त नहीं की जा सकती जब राज्र्, नागररकों (सभी र्ा कुछ - Article 19(6), an exception to Article 19(1)(g),
र्ा वकसी भी नागररक के साि प्रवतथपधाय में) को छोड़कर वकसी provides that no objection can be made when the
व्यापार, व्यिसार्, उद्योग र्ा सेिा पर एकाधधकार (पूर्य र्ा आंलशक) State carries on any trade, business or profession
रखते हुर्े काम करता है। except by citizens (in competition with all or
some or any of the citizens) Operates with a
monopoly (full or partial) on an industry or
service.
9. संविधान के अनुच्छे द 38 के बारे में वनम्नलिखित कथनों पर 9. Consider the following statements about
विचार कीजिए - Article 38 of the Constitution -
I. र्ह राज्र् को िोगों के कल्र्ार् को बढािा दे ने का वनदे श दे ता I. It directs the State to promote the welfare of
the people.
है।
II. It provides for equal pay for equal work for
II. र्ह पुरुषों और मवहिाओं दोनों के लिए समान काम के लिए
both men and women.
समान िेतन प्रदान करने की बात करता है।
III. The Article has been amended only once by
III. इसमें केिि एक बार 44िें संिैधावनक संशोधन अधधवनर्म 44th Constitutional Amendment Act, 1978.
1978 द्वारा संशोधन वकर्ा गर्ा है। Which of the above statement(s) is/are true?
उपययुक्त कथन में से कौन सा सही है/ हैं? (a) I and III only
(a) केिि I तिा III (b) II and III only
(b) केिि II तिा III (c) I only
(c) केिि I (d) II only
(d) केिि II [a]
Explanation:
[a]
- Article 38(1) provides that the State shall strive to
व्याख्या:
promote the welfare of the people by securing
- अनुच्छे द 38 (1) र्ह प्रावधान करता है वक राज्र् िोगों के कल्र्ार्
and protecting as effectively as it may a social
को बढािा दे ने और उनकी रक्षा करने का प्रर्ास करेगा क्र्ोंवक र्ह order in which justice, social, economic and
एक सामाजजक व्यिथिा है, जजसमें न्र्ार्, सामाजजक, आर्ििंक और political, shall inform all the institutions of the
राजनीवतक रूप से राष्ट्रीर् जीिन के सभी संथिानों को सूधचत वकर्ा national life.
जाएगा। - Article-39 provides for equal pay for equal work.
- अनुच्छे द-39 समान कार्य के लिए समान िेतन का प्रािधान करता - Article-38 has been amended only once by 44 th
है। Constitutional Amendment Act, 1978. It added a
- अनुच्छे द-38 में 44िें संिैधावनक संशोधन अधधवनर्म, 1978 द्वारा directive for the state to minimize the inequalities
in income, status, facilities and opportunities.
केिि एक बार संशोधन वकर्ा गर्ा है। इसमें राज्र् के लिए आर्,

-:: 6 ::-
RPT 05
स्थिवत, सुविधाओं और अिसरों में असमानताओं को कम करने के
लिए एक वनदे श जोड़ा गर्ा है।
10. वनम्नलिखित कथनों पर विचार कीजजए - 10. Consider the following statements -
I. भारत के संविधान के अनुसार, संसद और राज्र् विधानमंडि I. According to the Constitution of India, the
िमशः सशथत्र बिों और पुलिस बिों के मौलिक अधधकारों को Parliament and State Legislatures can
restrict the fundamental rights of the armed
सीधमत कर सकते हैं।
forces and the police forces respectively.
II. सशथत्र बिों के मौलिक अधधकारों पर उधचत प्रवतबंध उनके
II. Reasonable restrictions on the fundamental
कतयव्यों का उधचत वनियहन और उनमें अनुशासन बनाए रखना
rights of the armed forces ensure the proper
सुवनणित करते हैं। discharge of their duties and the
III. मौलिक अधधकारों के उल्िंर्न पर पीधड़त पक्ष उच्च न्र्ार्ािर् maintenance of discipline among them.
र्ा सिोच्च न्र्ार्ािर् जा सकता है। III. On violation of fundamental rights, the
उपययुक्त में से कौनसा/से कथन सत्य है/हैं? aggrieved party has the option of moving
(a) केिि I और II either the high court or the supreme court.
(b) केिि II और III Which of the above statement(s) is/are true?
(c) केिि I और III (a) I and II only
(b) II and III only
(d) उपरोि सभी
(c) I and III only
[b]
(d) All of the above
व्याख्या:
[b]
- अनुच्छे द 33 संसद को र्ह अधधकार दे ता है वक िह सशथत्र बिों, Explanation:
अधयसैवनक बिों, पुलिस बिों, खुविर्ा एजेंलसर्ों एिं अन्र् के मूि - Article 33 empowers the Parliament to restrict or
अधधकारों पर र्ुलिर्ुि प्रवतबंध िगा सकती है। मूि अधधकारों पर abrogate the fundamental rights of the members
प्रवतबंध के संबंध में र्ह शलि केिि संसद को प्राप्त है, राज्र् of armed forces, paramilitary forces, police
विधानमंडि को नहीं। forces, intelligence agencies and analogous
- संसद द्वारा विविध अधधवनर्म जैसे सैन्र् अधधवनर्म (1950), forces. The power is conferred only on
नौसेना अधधवनर्म (1950), िार्ु सेना अधधवनर्म (1950) आटद Parliament and not on state legislatures.
- Various Acts such as the Army Act (1950), the
के तहत् उनकी अणभव्यलि की थितंत्रता, संगठन बनाने के
Navy Act (1950), the Air Force Act (1950) etc.
अधधकार, श्रधमक संर्ों र्ा राजनीवतक संगठनों का सदथर् बनने का
impose restrictions on their freedom of speech,
अधधकार, प्रेस से चचाय करने का अधधकार, साियजवनक बैठकों र्ा
right to form associations, right to be members
प्रदशयन का अधधकार आटद पर र्ुलिर्ुि प्रवतबंध िगाए जाते हैं। of trade unions or political associations, right to
सशथत्र बिों के मौलिक अधधकारों पर उधचत प्रवतबंध उनके कतयव्यों communicate with the press etc. and ensures the
का उधचत वनियहन और उनमें अनुशासन बनाए रखना सुवनणित करते proper discharge of their duties and the
हैं। maintenance of discipline among them.
- मौलिक अधधकारों के प्रितयन के मामिे में, सिोच्च न्र्ार्ािर् का - In case of enforcement of fundamental rights,
अधधकार क्षेत्र मौलिक है िेवकन अनन्र् नहीं है। र्ह अनुच्छे द -226 the jurisdiction of the supreme court is original
के तहत् उच्च न्र्ार्ािर् के अधधकार क्षेत्र के साि समिती है। र्ह but not exclusive. It is concurrent with
jurisdiction of the high court under article 226. It
मौलिक अधधकारों के प्रितयन के लिए सभी प्रकार के वनदे श, आदे श
vests original powers in the High Court to issue
और ररि जारी करने के लिए उच्च न्र्ार्ािर् में मूि शलिर्ां वनवहत
directions, orders and writs of all kinds for the
करता है। अिायत मूि अधधकारों के उल्िंर्न पर पीधड़त पक्ष के पास
enforcement of the fundamental rights. It means
र्ा तो सिोच्च न्र्ार्ािर् र्ा उच्च न्र्ार्ािर् जाने का विकल्प that on violation of fundamental rights, the
उपिब्ध होता। aggrieved party has the option of moving either
the high court or the supreme court
11. संघ और राज्यों के बीच विधायी संबंधों के संबंध में, 11. With regard to Legislative Relations between
वनम्नलिखित कथनों पर विचार ककजिए - Union and states, consider the following
I. संसद द्वारा बनाए गए कानून पर राज्र् का कोई कानून हािी Statements -
I. No State law can prevail over the law made
नहीं हो सकता है।
by the Parliament.
II. संविधान थपि रूप से राज्र् सूची के ऊपर संर् सूची और
समिती सूची की प्रधानता को सुरणक्षत करता है ।
-:: 7 ::-
RPT 05
III. राज्र् सूची में शाधमि सभी मामिों के विधेर्कों को संर् की II. The Constitution expressly secures
पूिय मंजूरी के वबना विधानमंडि में पेश वकर्ा जा सकता है। predominance of Union List and Concurrent
उपययुक्त में से कौनसा/से कथन सत्य है/हैं? List over the State List.
III. The Bills on all matters enumerated in the
(a) केवल I
State List can be introduced in Legislature
(b) केवल II और III
without prior sanction from the Union.
(c) केवल II
Which of the above statement(s) is/are true?
(d) I, II और III (a) I only
[c] (b) II and III only
व्याख्या: (c) II only
- समिती सूची में शाधमि सभी विषर्ों पर केंद्रीर् कानून और राज्र् (d) I, II and III
कानून के बीच संर्षय के मामिे में, केंद्रीर् कानून राज्र् के कानून पर [c]
प्रभािी रहता है । िेवकन र्हां एक अपिाद है: र्टद राज्र् के कानून Explanation:
को राष्ट्रपवत के विचार के लिए आरणक्षत वकर्ा गर्ा है और उसकी - In case of a conflict between the Central law and
the state law on a subject enumerated in the
सहमवत धमि गई है, तो उस राज्र् में राज्र् का कानून प्रभािी रहता
Concurrent List, the Central law prevails over the
है। िेवकन संसद अभी भी सक्षम होगी वक िह बाद में इसी विषर्
state law. But there is an exception. If the state
पर कानून बनाकर ऐसे कानून को प्रत्र्ाटदि कर सकती है।
law has been reserved for the consideration of
- संविधान थपि रूप से राज्र् सूची और समिती सूची पर संर् सूची the president and has received his assent, then
की प्रधानता और राज्र् सूची पर समिती सूची की प्रधानता सुरणक्षत the state law prevails in that state. But it would
करता है । संर् सूची और समिती सूची के बीच संर्षय होने के मामिे still be competent for the Parliament to override
में संर् सूची प्रबि होनी चावहए। समिती सूची और राज्र् सूची के such a law by subsequently making a law on the
बीच संर्षय के मामिे में समिती सूची प्रबि रहती है। same matter.
- राज्र् सूची में कुछ मामिों (और सभी मामिों में नहीं) पर विधेर्क - The Constitution expressly secures the
केिि राष्ट्रपवत की पूिय मंजूरी के साि राज्र् विधानमंडि में पेश predominance of the Union List over the State
List and the Concurrent List and that of the
वकए जा सकता हैं। (उदाहरर् के लिए व्यापार और िाणर्ज्र् की
Concurrent List over the State List. Thus, in case
थितंत्रता पर प्रवतबंध िगाने िािे विधेर्क)
of overlapping between the Union List and the
State List, the former should prevail. In case of
overlapping between the Union List and the
Concurrent List, it is again the former which
should prevail. Where there is a conflict between
the Concurrent List and the State List, it is the
former that should prevail.
- The bills on certain matters (and not all matters)
in the State List can be introduced in the State
Legislature only with the previous sanction of the
President (For example bills imposing restriction
on freedom of trade & commerce).
12. भारत के संघीय ढांचे के संबंध में वनम्नलिखित कथनों पर 12. Consider the following statements with
विचार ककजिए - respect to federal structure of India -
कथन (A): राष्ट्रपवत, राज्र् सरकार की सहमवत के वबना भी, केंद्र Statement (A): The President may, even without
the consent of the state government, entrust to
के कार्यकारी कार्ों में से वकसी को उस सरकार को सौंप सकते हैं।
that government any of the executive functions
कारण (R) : भारतीर् संर्ीर् प्रर्ािी में अधधक मजबूत केंद्र है।
of the Centre.
(a) A सही है, और R, A का एक उपर्ुि थपिीकरर् है।
Reason (R): Indian federal system has a strong
(b) A सही है, िेवकन R, A का एक उपर्ुि थपिीकरर् नहीं है। centre.
(c) A गित है, िेवकन R सही है। (a) A is correct, and R is an appropriate
(d) A और R दोनों गित हैं। explanation of A.
[c] (b A is correct, but R is not an appropriate
व्याख्या: explanation of A.
(c) A is incorrect, but R is correct.
-:: 8 ::-
RPT 05
- संविधान में कठोरता को कम करने और गवतरोध की स्थिवत से बचने (d) Both A and R are incorrect
के लिए कार्यकारी कार्ों के अंतर- सरकारी प्रवतवनधधमंडि का [c]
प्रािधान है। Explanation:
- The Constitution provides for inter-government
- राष्ट्रपवत राज्र् सरकार की सहमवत से उस सरकार को केंद्र के वकसी
delegation of executive functions in order to
भी कार्यकारी कार्य को सौंप सकते हैं ।
mitigate rigidity and avoid a situation of
- संविधान में केंद्र के कार्यकारी कार्ों को वकसी राज्र् को सौंपे जाने
deadlock.
का प्रािधान भी उस राज्र् की सहमवत के वबना वकर्ा गर्ा है। िेवकन - The President may, with the consent of the state
ऐसा संसद द्वारा वकर्ा जा सकता है, न वक राष्ट्रपवत द्वारा । government, entrust to that government any of
- भारतीर् संर्ीर् प्रर्ािी, मजबूत केंद्र के साि कनाडा के मॉडि पर the executive functions of the Centre.
आधाररत है । उदाहरर् के लिए, संर् सूची में राज्र् सूची से अधधक - The Constitution also makes a provision for the
विषर् शाधमि हैं। entrustment of the executive functions of the
Centre to a state without the consent of that
state. But, in this case, the delegation is by the
Parliament and not by the president.
- Indian federal system is based on Canadian
model with strong centre. For e.g., union List
contains more subjects than state list.
13. संविधान के मसौदे में राज्यपाि के प्रत्यि चुनाि का प्रािधान 13. The Draft Constitution provided for the direct
था िेवकन संविधान सभा ने राष्ट्रपवत द्वारा राज्यपाि की election of the Governor but the Constituent
वनयुलक्त का विकल्प चुना, क्यों? Assembly opted for the appointment of the
Governor by the President, why?
I. प्रत्र्क्ष चुनाि द्वारा राज्र्पाि और मुख्र्मंत्री के बीच संर्षय पैदा
I. The mode of direct election is more likely to
होने की अधधक संभािना है।
create conflicts between the Governor and
II. राज्र्पाि के केिि एक संिैधावनक (नाममात्र) प्रमुख होने के
the Chief Minister.
कारर् उसके चुनाि के लिए विथतृत व्यिथिा करने और भारी II. The Governor being only a Constitutional
मात्रा में धन खचय करने का कोई मतिब नहीं है। (nominal) head, there is no point in making
III. संविधान सभा ने अमेररकी मॉडि का अनुसरर् वकर्ा, जहां elaborate arrangements for his election and
राष्ट्रपवत द्वारा राज्र्पाि की वनर्ुलि की जाती है। spending huge amounts of money.
IV. राज्र्पाि का चुनाि अिगाििादी प्रिृलत्तर्ों को पैदा करेगा III. The constituent assembly followed the
और इस प्रकार दे श की राजनीवतक स्थिरता और एकता को American model, where the Governor is
प्रभावित करेगा। appointed by the President.
IV. The election of the Governor would create
उपययुक्त में से कौनसा/से कथन सत्य है/हैं?
separatist tendencies and thus affect the
(a) केिि I और II
political stability and unity of the country.
(b) केिि III
Which of the above statement(s) is/are true?
(c) केिि I, II और IV (a) I and II only
(d) I, II, III और IV (b) III only
[c] (c) I, II and IV only
व्याख्या: (d) I, II, III and IV
- राज्र्पाि का प्रत्र्क्ष चुनाि, मुख्र्मंत्री के खखिाि संर्षय पैदा करेगा, [c]
क्र्ोंवक मुख्र्मंत्री पहिे से ही राज्र् में िोगों का एक वनिायधचत Explanation:
प्रवतवनधध है और राज्र् में दो वनिायधचत कार्यकारी प्रमुख नहीं हो - Direct election of the governor would create a
conflict against the Chief Minister, as the Chief
सकते हैं।
Minister is already an elected representative of
- केिि राज्र् का संिैधावनक प्रमुख होने के नाते राज्र्पाि का चुनाि
the people in the state and there could not be two
पूरी तरह से अनािश्र्क है और राष्ट्रपवत नामांकन की प्रर्ािी केंद्र
elected executive heads in the state.
को राज्र्ों पर अपना वनर्ंत्रर् बनाए रखने में सक्षम बनाती है। - Being only a constitutional head of the state the
- संविधान सभा ने अमेररकी मॉडि का पािन नहीं वकर्ा िा, election of the governor is entirely unnecessary
अमेररका में एक राज्र् के राज्र्पाि को सीधे चुना जाता है और also the system of the presidential nomination
राष्ट्रपवत द्वारा वनर्ुि नहीं वकर्ा जाता है। संविधान सभा ने कनाडाई enables the Centre to maintain its control over
the states.
-:: 9 ::-
RPT 05
मॉडि का अनुसरर् वकर्ा, जहां एक राज्र् के राज्र्पाि की वनर्ुलि - The Constituent Assembly had not followed the
गिनयर-जनरि (केंद्र) द्वारा की जाती है। American model, in America the Governor of a
- र्टद राज्र्पाि सीधे चुने जाते हैं तो दो वनिायधचत वनकार् होंगे जो state is directly elected and not appointed by the
President. The Constituent Assembly followed
एक दूसरे को मनाने और एक दूसरे के खखिाि अपनी शलिर्ों का
the Canadian model, where the Governor of a
इथतेमाि करने की कोलशश करेंगे, जो अिगाििादी प्रिृलत्तर्ों को
state is appointed by the Governor- General
पैदा करेगा और इस प्रकार दे श की राजनीवतक स्थिरता और एकता
(Centre).
को प्रभावित करेगा। - If the governor is directly elected there will be
two elected bodies which would try to persuade
each other and use their powers against each
other which would have created separatist
tendencies and thus affect the political stability
and unity of the country.
14. वनम्नलिखित कथनों पर विचार कीजजए - 14. Consider the following statements -
I. नागररकों के मूि कत्तयव्यों का संविधान में समािेश तो है परन्तु I. The basic duties of the citizens are included
इन्हें िागू करने के संबंध में संविधान मौन है। in the constitution, but the constitution is
silent regarding their implementation.
II. संविधान, मूि कत्तयव्यों की अनुपािना की शतय पर हमें मौलिक
II. The constitution gives us fundamental rights
अधधकार प्रदान करता है।
subject to the performance of fundamental
उपयुथक्त में से कौन-सा/से कथन सत्य है/हैं?
duties.
(a) केिि I Which of the above statement(s) is/are true?
(b) केिि II (a) I only
(c) I और II दोनों (b) II only
(d) न तो I, न ही II (c) Both I and II
[a] (d) Neither I nor II
व्याख्या – [a]
– 42 िें संविधान संशोधन द्वारा संविधान में नागररकों के मूि कत्तयव्यों Explanation -
– A list of fundamental duties of citizens has been
की एक सूची का समािेश वकर्ा गर्ा है, परन्तु इन्हें िागू करने के
included in the Constitution by the 42 nd
संबंध में संविधान मौन है।
Constitutional Amendment, but the Constitution
– संविधान मौलिक कत्तयव्यों के अनुपािन के आधार र्ा शतय पर हमें
is silent regarding its implementation.
मौलिक अधधकार नहीं दे ता है। बस्ल्क मूि अधधकार मूि कत्तयव्य से – The constitution does not give us fundamental
थितंत्र है। rights on the basis or condition of the
performance of fundamental duties. Rather, the
fundamental right is independent of the
fundamental duty.
15. भारत में, यदद एक धार्मिक संप्रदाय/समुदाय को "राष्ट्रीय 15. In India, if a religious sect/community is given
अल्पसंख्यक का दजाथ ददया जाता है, तो िह कौनसे विशेष the status of "National Minority", to what
िाभ का हकदार है? special benefits is it entitled?
I. It can derive benefits from the 15 point
I. र्ह प्रधानमंत्री के 15 सूत्रीर् कार्यिम से िाभ प्राप्त कर सकता
program of the Prime Minister.
है।
II. The President of India automatically
II. भारत के राष्ट्रपवत थिचालित रूप से समुदार् के एक प्रवतवनधध
nominates a representative of the
को िोकसभा में नाधमत करते हैं। community to the Lok Sabha.
III. र्ह विशेष शैक्षणर्क संथिानों की थिापना और प्रशासन कर III. It can establish and administer special
सकता है। educational institutions.
उपययुक्त में से कौनसा/से कथन सत्य है/हैं? Which of the above statement(s) is/are true?
(a) केिि I (a) I only
(b) केिि II और III (b) II and III only
(c) केिि I और III (c) I and III only
(d) I, II and III
(d) I, II और III
[c]
-:: 10 ::-
RPT 05
[c] Explanation -
व्याख्या – - In India, if a religious sect/community is given the
- भारत में, र्टद वकसी धार्मिंक संप्रदार्/समुदार् को राष्ट्रीर् status of a national minority, it is entitled to the
following special benefits:
अल्पसंख्र्क का दजाय टदर्ा जाता है, तो िह वनम्नलिखखत विशेष
- It can establish and administer special
िाभों का हकदार होता है:
educational institutions. According to Article 30
- र्ह विशेष शैक्षणर्क संथिानों की थिापना और प्रशासन कर सकता
of the Indian Constitution, all minorities, whether
है। भारतीर् संविधान के अनुच्छे द 30 के अनुसार, सभी based on religion or language, have the right to
अल्पसंख्र्कों को, चाहे िे धमय र्ा भाषा पर आधाररत हों, अपनी establish and administer educational institutions
पसंद के लशक्षर् संथिानों की थिापना और प्रशासन का अधधकार of their choice. It helps in preserving and
है। र्ह उनकी संथकृवत को संरणक्षत करने और बढािा दे ने और उनके promoting their culture and providing education
समुदार् के सदथर्ों को लशक्षा प्रदान करने में मदद करता है। to the members of their community.
- इसे प्रधानमंत्री के 15 सूत्री कार्यिम से िाभ धमि सकता है। बजि - It can get benefit from Prime Minister's 15 point
का 15 प्रवतशत अल्पसंख्र्कों पर खचय करना जरूरी है। program. It is necessary to spend 15 percent of
the budget on minorities.
- भारत के राष्ट्रपवत अल्पसंख्र्क समुदार् के प्रवतवनधध को िोकसभा
- The President of India does not nominate the
में नाधमत नहीं करते हैं।
representative of the minority community in the
Lok Sabha.
16. सूची -I को सूची – II से सयमेलित कीजिए- 16. Match List – I with List – II.
सूची–I सूची–II List–I List–II
(राज्य) (िोकसभा में सीटों (State) (Number of seats in
Lok Sabha)
की संख्या)
A. Uttar Pradesh I. 80
A. उत्तर प्रदे श I. 80
B. Maharashtra II. 25
B. मिाराष्ट्र II. 25
C. Rajasthan III. 26
C. राजस्थान III. 26 D. Gujarat IV. 48
D. गयजरात IV. 48 Code:
कूट: (a) A-I B-IV C-II D-III
(a) A-I B-IV C-II D-III (b) A-IV B-I C-II D-III
(b) A-IV B-I C-II D-III (c) A-I B-III C-II D-IV
(c) A-I B-III C-II D-IV (d) A-IV B-I C-III D-II
(d) A-IV B-I C-III D-II [a]
Explanation -
[a]
- Uttar Pradesh – 80 seats
व्याख्या –
- Maharashtra – 48 seats
- उत्तर प्रदे श – 80 सीटें
- Rajasthan – 25 seats
- महाराष्ट्र – 48 सीटें - Gujarat – 26 seats
- राजथिान – 25 सीटें
- गुजरात – 26 सीटें
17. जब िार्षिक केंद्रीय बजट िोकसभा द्वारा पाररत नहीं वकया 17. When the annual Union Budget is not passed
जाता है - by the Lok Sabha -
(a) बजि संशोधधत और विर से प्रथतुत वकर्ा जाता है। (a) The budget is revised and presented again
(b) The budget is sent to the Rajya Sabha for
(b) बजि को सुझािों के लिए राज्र्सभा में भेजा जाता है।
suggestions
(c) केंद्रीर् वित्त मंत्री को इथतीिा दे ने के लिए कहा जाता है।
(c) Union Finance Minister is asked to resign
(d) प्रधानमंत्री मंवत्रपररषद् का इथतीिा सौंपते हैं।
(d) The Prime Minister submits the resignation
[d] of the Council of Ministers
व्याख्या – [d]
- जब िोकसभा द्वारा िार्षिंक केंद्रीर् बजि पाररत नहीं वकर्ा जाता Explanation -
है, तो प्रधानमंत्री मंवत्रपररषद का इथतीिा सौंप दे ते हैं। अगर संसद - When the annual Union Budget is not passed by
में वकसी भी हाित में बजि पास नहीं होता है तो समझा जाता है वक the Lok Sabha, the Prime Minister submits the
सत्ता पक्ष अल्पमत में है। तकनीकी रूप से इसका मतिब है वक resignation of the Council of Ministers. If the
-:: 11 ::-
RPT 05
सरकार िोकसभा में विश्वास मत खो चुकी है और उसे इथतीिा दे ना budget is not passed in the Parliament under any
होगा। भारत के संसदीर् इवतहास में अब तक ऐसा नहीं हुआ है। circumstances, then it can be understood that
the ruling party is in a minority. Technically this
means that the government has lost the vote of
confidence in the Lok Sabha and will have to
resign. This has not happened so far in the
parliamentary history of India.
18. भारत में प्रथम राष्ट्रीय आपातकाि की उद्घोषणा के समय दे श 18. Who was the President of the country at the
के राष्ट्रपकत कौन थे? time of declaration of the first National
(a) डॉ. राजेन्द्र प्रसाद Emergency in India?
(a) Dr. Rajendra Prasad
(b) डॉ. एस. राधाकृष्णन
(b) Dr. S. Radhakrishnan
(c) डॉ जाहकर हुसैन
(c) Dr. Zakir Hussain
(d) फखरुद्दीन अली अिमद
(d) Fakhruddin Ali Ahmed
[b] [b]
व्याख्या – Explanation -
– दे श में प्रथम राष्ट्रीर् आपातकाल की उद्दघोषणा 26 अक्टू बर 1962 - Proclamation of the first National Emergency in
को राष्ट्रपहत डॉ. एस. राधाकृष्णन द्वारा की गई थी। the country on 26 October 1962 was done by Dr.
– इस राष्ट्रीर् आपातकाल की उद्दघोषणा भारत पर चीनी आक्रमण के S. Radhakrishna.
कारण की गई थी। – This national emergency was declared due to
Chinese aggression on India.
19. भारत के ‘वित्त आयोग’ के संदभथ में, वनम्नलिखित में से कौन- 19. With reference to the 'Finance Commission' of
सा कथन सही है? India, which of the following statements is
(a) र्ह बुवनर्ादी ढ़ााँचे के विकास के लिए विदे शी पूज ाँ ी के प्रिाह correct?
(a) It encourages inflow of foreign capital for
को प्रोत्सावहत करता है।
infrastructure development.
(b) र्ह साियजवनक क्षेत्र के उपिमों के बीच वित्त के उधचत वितरर्
(b) It facilitates fair distribution of finance
की सुविधा प्रदान करता है।
among public sector undertakings
(c) र्ह वित्तीर् प्रशासन में पारदर्शिंता सुवनणित करता है। (c) It ensures transparency in financial
(d) उपर्ुयि में से कोई नहीं। administration.
[d] (d) None of the above
व्याख्या – [d]
- वित्त आयोग- संविधान के अनुच्छे द 280 के अंतगयत र्ह प्रािधान Explanation -
वकर्ा गर्ा है वक संविधान के प्रारंभ से दो िषय के भीतर और उसके - Finance Commission - Article 280 of the with be
बाद प्रत्र्ेक पााँच िषय की समाप्प्त पर र्ा पहिे उस समर् पर, जजसे Constitution provides that a Finance Commission
with be constituted within two years from the
राष्ट्रपवत द्वारा आिश्र्क समझा जाता है, एक वित्त आर्ोग का गठन
commencement of the Constitution and
वकर्ा जाएगा।
thereafter at the end of every five years or at such
- वित्त आयोग के कायथ - भारत के राष्ट्रपवत को र्ह लसिाररश करना
earlier time as the President may consider
वक संर् एिं राज्र्ों के बीच करों की शुद्ध प्राप्प्तर्ों को कैसे वितररत necessary.
वकर्ा जार्े। - Functions of the Finance Commission - To
- वनर्यर् िेना वक अनुच्छे द-275 के तहत् संधचत वनधध में से राज्र्ों को make recommendations to the President of India
अनुदान/सहार्ता टदर्ा जाना चावहर्े र्ा नहीं। as how to distribute the net proceeds of taxes
- वित्त आयोग की शलक्तयााँ - आर्ोग अपनी ररपोिय राष्ट्रपवत को between the Union and the States.
सौंपता है, जजसे राष्ट्रपवत संसद के दोनों सदनों के समक्ष रखिाता है। - To decide whether grant/assistance should be
- वित्त आर्ोग द्वारा की गई लसिाररशें सिाहकारी प्रिृलत्त की होती हैं, given to the states from the accumulated fund
under Article-275 or not.
उन्हें मानना र्ा न मानना सरकार पर वनभयर करता है।
- Powers of the Finance Commission - The
Commission submits its report to the President,
which the President gets placed before both the
Houses of the Parliament.

-:: 12 ::-
RPT 05
- The recommendations made by the Finance
Commission are of advisory nature, it depends
on the government to accept or not to accept
them.
20. वनम्नलिखित में से वकस वनकाय का संविधान में उल्िेि नहीं 20. Which of the following body is not mentioned
है? in the constitution?
I. राष्ट्रीर् विकास पररषद I. National Development Council
II. Planning Commission
II. र्ोजना आर्ोग
III. Zonal Council
III. क्षेत्रीर् पररषद
Correct Code –
सही कूट -
(a) I and II only
(a) केिि I और II (b) II only
(b) केिि II (c) I and III only
(c) केिि I और III (d) I, II and III
(d) I, II और III [d]
[d] Explanation -
व्याख्या – Constitutional Body:
संिैधावनक वनकाय: - The bodies/institutions that find a place in the
Constitution of India are called Constitutional
जजन वनकार्ों/संथिानों का उल्िेख भारत के संविधान में धमिता है,
Bodies. As -
उन्हें संिैधावनक वनकार् कहा जाता है। जैसे -
1. Advocate General of the State (Article-165)
1. राज्र् के महाधधििा (अनुच्छे द-165)
2. Attorney General of India (Article-76)
2. भारत का महान्र्ार्िादी (अनुच्छे द-76) 3. Comptroller and Auditor General of India
3. भारत के वनर्ंत्रक और महािेखा परीक्षक (अनुच्छे द-148) (Article-148)
4. वनिायचन आर्ोग (अनुच्छे द-324) 4. Election Commission (Article-324)
5. वित्त आर्ोग (अनुच्छे द-280) 5. Finance Commission (Article-280)
6. राष्ट्रीर् अनुसूधचत जावत आर्ोग (अनुच्छे द-338) 6. National Commission for Scheduled Castes
7. राष्ट्रीर् अनुसूधचत जनजावत आर्ोग (अनुच्छे द-338-ए) (Article-338)
8. भाषाई अल्पसंख्र्कों के लिए विशेष अधधकारी (अनुच्छे द- 7. National Commission for Scheduled Tribes
(Article-338-A)
350-बी)
8. Special Officer for Linguistic Minorities
9. राज्र् िोक सेिा आर्ोग (अनुच्छे द-315-323)
(Article-350-B)
10. संर् िोक सेिा आर्ोग (अनुच्छे द-315-323)
9. State Public Service Commission (Article-315-
गैर-संिैधावनक वनकाय - 323)
- िे संथिाएाँ जजन्हें संविधान में थिान नहीं धमिता है और संसद द्वारा 10. Union Public Service Commission (Article-
एक अधधवनर्म को पाररत करने के बाद थिावपत की जाती हैं। 315-323)
गैर-संिैधावनक वनकाय हैं: Non-constitutional bodies:
1. केंद्रीर् जााँच ब्र्ूरो - Those institutions which do not find place in the
2. केंद्रीर् सूचना आर्ोग constitution and are established after passing an
3. केंद्रीर् सतकयता आर्ोग act of the parliament.
Non-constitutional bodies are:
4. िोकपाि और िोकार्ुि
1. Central Bureau of Investigation
5. नीवत आर्ोग
2. Central Information Commission
6. राष्ट्रीर् विकास पररषद्
3. Central Vigilance Commission
7. राष्ट्रीर् आपदा प्रबंधन प्राधधकरर् 4. Lokpal and Lokayukta
8. राष्ट्रीर् मानि अधधकार आर्ोग 5. NITI Aayog
9. राष्ट्रीर् जााँच एजेंसी 6. National Development Council
10. राज्र् मानि अधधकार आर्ोग 7. National Disaster Management Authority
8. National Human Rights Commission
9. National Investigation Agency
10. State Human Rights Commission

-:: 13 ::-
RPT 05
21. भारत के प्रधानमंरी की वनयुलक्त के समय उन्त्हें - 21. At the time of appointment of the Prime
(a) संसद के सदनों में से एक का सदथर् होना जरूरी नहीं है, Minister of India, he -
िेवकन छह महीने के भीतर िोकसभा का सदथर् बनना (a) Need not be a member of one of the Houses
of the Parliament, but must have become a
चावहए।
member of the Lok Sabha within six months
(b) िोकसभा का सदथर् होना चावहए।
(b) Must be a member of the Lok Sabha
(c) संसद के सदनों में से एक का सदथर् होना चावहए।
(c) Must be a member of one of the Houses of
(d) संसद के सदनों में से एक का सदथर् होना आिश्र्क नहीं है, Parliament
िेवकन छह महीने के भीतर सदनों में से एक का सदथर् िोना (d) Must not be a member of one of the Houses
चावहए। of Parliament, but must have become a
[d] member of one of the Houses within six
व्याख्या – months
- अनुच्छे द-75(5) के अनुसार भारत के प्रधानमंत्री का वनर्ुलि के [d]
समर् संसद के सदनों में से वकसी एक का सदथर् होना आिश्र्क Explanation -
नहीं है, िेवकन छह महीने के भीतर संसद के सदनों में से वकसी एक - According to Article-75(5) the Prime Minister of
India need not be a member of either of the
का सदथर् बनना आिश्र्क है।
Houses of Parliament at the time of appointment,
but must become a member of either of the
Houses of Parliament within six months.
22. भारत के संविधान के अनुसार, भारत के राष्ट्रपवत का यह 22. According to the Constitution of India, it is the
कत्तथव्य है वक िह संसद के समि वनम्नलिखित में से वकसे रिे? duty of the President of India to place before
I. संर् वित्त आर्ोग की लसिाररशें the Parliament which of the following?
I. Recommendations of the Union Finance
II. िोक िेखा सधमवत की ररपोिय
Commission
III. वनर्ंत्रक-महािेखा परीक्षक की ररपोिय
II. Report of the Public Accounts Committee
IV. राष्ट्रीर् अनुसूधचत जावत आर्ोग की ररपोिय
III. Comptroller and Auditor General's Report
उपयुथक्त में कौन-सा/से कथन सत्य है/हैं? IV. Report of the National Commission for
(a) केिि I Scheduled Castes
(b) केिि II और IV Which of the above statement(s) is/are true?
(c) केिि I, III और IV (a) I only
(d) I, II, III और IV (b) II and IV only
[c] (c) I, III and IV only
व्याख्या – (d) I, II, III and IV
[c]
- भारत के राष्ट्रपवत का र्ह कत्तयव्य है वक िह संसद के समक्ष संर् वित्त
Explanation -
आर्ोग की लसिाररशें, वनर्ंत्रक-महािेखा परीक्षक की ररपोिय और
- It is the duty of the President of India to place
राष्ट्रीर् अनुसूधचत जावत आर्ोग की ररपोिय रखेगा।
before the Parliament the recommendations of
- िोक िेखा सधमवत संसदीर् सधमवत होती है तिा र्ह अपनी ररपोिय the Union Finance Commission, the report of the
थिर्ं संसद पिि पर रखती है। Comptroller and Auditor General and the report
of the National Commission for Scheduled
Castes.
- The Public Accounts Committee is a
parliamentary committee and it itself places its
report on the table of Parliament.
23. िोकसभा और राज्यसभा के बीच गवतरोध के कारण संसद की 23. Due to the deadlock between the Lok Sabha
संयुक्त बैठक कौनसा विधेयक पाररत करिाने के लिए बुिाई and the Rajya Sabha, a joint sitting of the
जाती है? Parliament is called to pass which bill?
I. Ordinary Bill
I. साधारर् विधेर्क
II. Money Bill
II. धन विधेर्क
III. Constitution amendment bill
III. संविधान संशोधन विधेर्क
Code
कूट (a) I only
-:: 14 ::-
RPT 05
(a) केिि I (b) II and III only
(b) केिि II और III (c) I and III only
(c) केिि I और III (d) I, II and III
[a]
(d) I, II और III
Explanation -
[a]
- Under Article-108, the President can call a joint
व्याख्या –
meeting in case of a deadlock in both the Houses
- अनुच्छे द-108 के तहत् संसद के दोनों सदनों में वकसी वबि को of Parliament regarding a bill. But a joint sitting
िेकर गवतरोध होने पर राष्ट्रपवत संर्ुि बैठक आहूत कर सकता है cannot be called in the matter of constitutional
परन्तु संविधान संशोधन और धन विधेर्क के मामिों में संर्ुि amendment and money bill.
बैठक नहीं बुिाई जा सकती है।
24. भारत के उपराष्ट्रपकत के सम्बंध में कनम्नलिखित में से कौनसा 24. Which of the following statements are true
कथन सत्य हैं? regarding the Vice President of India?
(a) डॉ. एस. राधाकृष्णन उपराष्ट्रपहत पद िेतय हनर्विरोध हनवाुधचत (a) Dr. S. Radhakrishnan was elected unopposed
to the post of Vice President.
हुए थे।
(b) Dr. Zakir Husain was the first Vice President
(b) डॉ. जाहकर हुसैन भारत के प्रथम उपराष्ट्रपहत थे।
of India.
(c) श्री जगदीप धनखड़ राजस्थान से प्रथम व्यक्तक्त िैं जो भारत के
(c) Shri Jagdeep Dhankhar is the first person
उपराष्ट्रपहत के रूप में हनवाुधचत हुए िैं। from Rajasthan to be elected as the Vice
(d) उपराष्ट्रपहत पद पर हनवाुचन िेतय श्री राधाकृष्णन भारतीर् President of India.
राष्ट्रीर् कांग्रेस के उम्मीदवार थे। (d) Shri Radhakrishnan was the candidate of the
[a] Indian National Congress for the election to
व्याख्या – the post of Vice President.
– डॉ. एस. राधाकृष्णन भारत के प्रथम उपराष्ट्रपहत थे। वे दस साल [a]
तक भारत के उपराष्ट्रपहत रिे थे। डॉ. जाहकर हुसैन उनके Explanation -
– Dr. S. Radhakrishnan was the first Vice President
उत्तराधधकारी थे।
of India. He was the Vice President of India for ten
– डॉ. राधाकृष्णन दोनों बार (वषु 1952 व वषु 1957) हनर्विरोध रूप
years. Dr. Zakir Hussain was his successor.
से हनवाुधचत हुए थे।
– Dr. Radhakrishnan was elected unopposed both
– श्री जगदीप धनखड़ प्रथम राजस्थानी उपराष्ट्रपहत निीं िै। उनसे times (1952 and 1957).
पिले भैरोंससिि शेखावत भी भारत के उपराष्ट्रपहत रि चयके िैं। – Mr. Jagdeep Dhankar is not the first Rajasthani
Vice President. Before him Mr. Bhero Singh
Shekhawat has also been the Vice President of
India.
25. भारत के सिोच्च न्त्यायािय की स्िायत्तता की रिा के लिए क्या 25. What is the provision to protect the autonomy
प्रािधान है? of the Supreme Court of India?
I. सिोच्च न्र्ार्ािर् के न्र्ार्ाधीशों की वनर्ुलि करते समर्, I. While appointing the judges of the Supreme
Court, the President of India has to consult
भारत के राष्ट्रपवत को भारत के मुख्र् न्र्ार्ाधीश से परामशय
the Chief Justice of India.
करना पड़ता है।
II. Judges of the Supreme Court can be
II. सिोच्च न्र्ार्ािर् के न्र्ार्ाधीशों को भारत के मुख्र् न्र्ार्ाधीश
removed only by the Chief Justice of India.
द्वारा ही हिार्ा जा सकता है। III. Judges' salaries are charged on the
III. न्र्ार्ाधीशों का िेतन भारत की संधचत वनधध पर भाररत होता Consolidated Fund of India, which the
है, जजसके लिए विधावर्का को मतदान नहीं करना पड़ता है। legislature does not have to vote on.
IV. भारत के सिोच्च न्र्ार्ािर् के अधधकाररर्ों और कमयचाररर्ों IV. All appointments of officers and employees
की सभी वनर्ुलिर्ााँ भारत के मुख्र् न्र्ार्ाधीश से परामशय करने of the Supreme Court of India are made by
के बाद ही सरकार द्वारा की जाती हैं। the government only after consultation with
कूट the Chief Justice of India.
Code
(a) केिि I और III
(a) I and III only
(b) केिि III और IV
(b) III and IV only
(c) केिि IV
(c) IV only
-:: 15 ::-
RPT 05
(d) I, II, III और IV (d) I, II, III and IV
[a] [a]
व्याख्या – Explanation -
- The following provisions have been made in the
- संविधान में वनष्पक्ष न्र्ार्पालिका के लिए सिोच्च न्र्ार्ािर् की
Constitution to protect the autonomy of the
थिार्त्तता की रक्षा के लिए वनम्न प्रािधान वकर्े गए िैं-
Supreme Court for an impartial judiciary:
- सिोच्च न्र्ार्ािर् के न्र्ार्ाधीशों की वनर्ुलि करते समर्, भारत के
- While appointing the judges of the Supreme
राष्ट्रपवत को भारत के मुख्र् न्र्ार्ाधीश से परामशय करना होता है। Court, the President of India has to consult the
अनुच्छे द -124(2) Chief Justice of India. Article-124(2)
- न्र्ार्ाधीशों का िेतन भारत की संधचत वनधध पर भाररत होता है, - The salaries of judges are charged on the
जजसके लिए विधावर्का को मतदान नहीं करना पड़ता है। अनुच्छे द– Consolidated Fund of India for which the
125। legislature does not have to vote. Article – 125
- सिोच्च न्र्ार्ािर् के न्र्ार्ाधीशों को हिाने के लिए महाणभर्ोग की - Impeachment proceedings are done to remove
कार्यिाही की जाती िै। अनुच्छे द-124(4)। the judges of the Supreme Court. Article-124(4)
- All appointments of officers and employees of
- भारत के सिोच्च न्र्ार्ािर् के अधधकाररर्ों और कमयचाररर्ों की
the Supreme Court of India are made by the
सभी वनर्ुलिर्ााँ सिोच्च न्र्ार्ािर् द्वारा थिर्ं ही की जाती हैं।
Supreme Court itself. Article-146
अनुच्छे द-146।
26. भारत की संसद में ‘स्थगन प्रस्ताि’ का उद्दे श्य है- 26. The purpose of 'Adjournment Motion' in the
(a) तत्काि साियजवनक महत्त्व के एक वनणित मामिे पर चचाय की Parliament of India is-
अनुमवत दे ने के लिए (a) To allow discussion on a certain matter of
urgent public importance
(b) विपक्षी सदथर्ों को मंवत्रर्ों से जानकारी एकवत्रत करने के लिए
(b) To opposition members to collect
(c) अनुदान की मााँग में विलशि रालश की कमी की अनुमवत दे ने के
information from ministers
लिए
(c) To allow reduction of specific amount in the
(d) कुछ सदथर्ों की ओर से अनुधचत र्ा कहिंसक व्यिहार को रोकने Demand for Grants
के लिए कार्यिाही को थिवगत करने हेतु (d) To adjourn proceedings to prevent unfair or
[a] violent behavior on the part of some
व्याख्या – members
- थिगन प्रथताि का अणभप्रार् एक ऐसी प्रविर्ा से है, जजसके थिीकृत [a]
होने पर िोक महत्त्ि के वकसी वनणित मामिे की चचाय करने के लिर्े Explanation -
सभा का सामान्र् कार्य रोक टदर्ा जाता है। - Adjournment motion means such a procedure,
on the acceptance of which the general work of
- इस प्रथताि को थिीकार करने के लिए न्र्ूनतम 50 सांसदों के समियन
the House is stopped to discuss certain matters
की आिश्र्कता होती है। सदन में थिगन प्रथताि के तहत् चचाय करने
of public importance.
के लिए वकसी विषर् को थिीकार करने र्ा अथिीकार करने का पूरा
- To accept this proposal, the support of minimum
अधधकार सदन के पीठासीन अधधकारी के पास होता है। र्टद िह 50 MPs is required. The Presiding Officer of the
वकसी मामिे को अथिीकृत करता है तो पीठासीन अधधकारी उसका House has full authority to accept or reject any
कारर् बताने के लिए बाध्र् नहीं होता है। subject for discussion under the adjournment
motion in the House. If he disallows a case, the
Presiding Officer is not bound to give reasons for
the same.
27. भारत के संविधान में वनवहत राज्य के नीवत वनदे शक तत्त्वों के 27. Consider the following provisions under the
तहत् वनम्नलिखित प्रािधानों पर विचार करें- Directive Principles of State enshrined in the
I. भारत के नागररकों के लिए एक समान नागररक संवहता Constitution of India-
I. To ensure a uniform civil code for the citizens
सुवनणित करना
of India
II. ग्राम पंचार्तों का गठन
II. Formation of Gram Panchayats
III. ग्रामीर् क्षेत्रों में कुिीर उद्योगों को बढािा दे ना
III. Promotion of cottage industries in rural
IV. सभी कमयचाररर्ों के लिए उधचत अिकाश और सांथकृवतक areas
अिसर सुवनणित करना IV. Ensuring appropriate leisure and cultural
opportunities for all employees

-:: 16 ::-
RPT 05
उपयुथक्त में से कौन से गााँधीिादी लसद्ांत हैं जो राज्य के नीवत Which of the above are Gandhian principles
वनदे शक तत्त्वों में पररिक्षित होते हैं? that are reflected in the Directive Principles of
(a) केिि I, II और IV State Policy?
(a) I, II and IV only
(b) केिि II और III
(b) II and III only
(c) केिि I, III और IV
(c) I, III and IV only
(d) I, II, III और IV
(d) I, II, III and IV
[b] [b]
व्याख्या – Explanation -
- ग्राम पंचार्तों का गठन करना तिा ग्रामीर् क्षेत्रों में कुिीर उद्योगों - Formation of Gram Panchayats and promotion of
को बढािा दे ना गााँधीिादी लसद्धांत हैं जो राज्र् के नीवत वनदे शक cottage industries in rural areas are Gandhian
लसद्धांतों में पररिणक्षत होते हैं। principles which are reflected in the Directive
Principles of State Policy.
28. वनम्नलिखित कथनों पर विचार करें - 28. Consider the following statements -
I. जब कोई संविधान संशोधन विधेर्क भारत के राष्ट्रपवत के I. When a constitution amendment bill is
समक्ष प्रथतुत वकर्ा जाता है, तो भारत के राष्ट्रपवत के लिए presented before the President of India, it is
mandatory for the President of India to give
अपनी सहमवत दे ना अवनिार्य होता है।
his assent.
II. संविधान में संशोधन करने िािे विधेर्क के लिए भारत के
II. A bill to amend the constitution requires the
राष्ट्रपवत की पूिय अनुशंसा की आिश्र्कता होती है।
prior recommendation of the President of
III. एक संविधान संशोधन विधेर्क को िोकसभा और राज्र्सभा India.
दोनों द्वारा विशेष बहुमत से पाररत वकर्ा जाना चावहए तथा III. A constitution amendment bill has to be
संर्ुि बैठक का कोई प्रािधान नहीं है। passed by both Lok Sabha and Rajya Sabha
उपयुथक्त कथनों में से कौन सा/से सत्य है/हैं? by special majority and there is no provision
(a) केिि I और II for joint sitting.
(b) केिि II और III Which of the above statement(s) is/are true?
(c) केिि I और III (a) I and II only
(b) II and III only
(d) I, II और III
(c) I and III only
[c]
(d) I, II and III
व्याख्या –
[c]
- संविधान में संशोधन की विर्ाविधध अनुच्छे द-368 में उल्ललखखत है। Explanation -
- संविधान में संशोधन केिि संसद के वकसी भी सदन में इस उद्दे श्र् - The procedure for amendment of the
के लिए एक विधेर्क पेश करके शुरू वकर्ा जा सकता है और जब constitution is mentioned in article 368.
विधेर्क प्रत्र्ेक सदन में मतदान के साि, उस सदन की कुि - An amendment of this Constitution may be
सदथर्ता के विशेष बहुमत से पाररत हो जाता है तब िह राष्ट्रपवत के commenced only by the introduction of a Bill for
समक्ष प्रथतुत वकर्ा जाएगा, जो विधेर्क पर अपनी थिीकृवत दे गा। the purpose in either House of Parliament, and
- िषय 1971 के 24िें संविधान संशोधन अधधवनर्म ने राष्ट्रपवत के लिए when the Bill is passed by a special majority of the
total membership of that House, with a vote in
एक संिैधावनक संशोधन विधेर्क पर अपनी सहमवत दे ना अवनिार्य
each House It will be presented to the President,
कर टदर्ा है। संविधान संशोधन विधेर्क के संबंध में राष्ट्रपवत के पास
who will give his assent to the bill.
कोई िीिो शलि नहीं है।
- The 24th Constitutional Amendment Act of 1971
- एक संिैधावनक संशोधन विधेर्क को पेश करने के लिए राष्ट्रपवत की made it mandatory for the President to give his
पूिय लसिाररश की आिश्र्कता नहीं होती है। दोनों सदनों के बीच assent to a constitutional amendment bill. The
असहमवत की स्थिवत में संसद के संर्ुि सत्र के आह्वान का कोई President has no veto power with respect to the
उपबंध नहीं है। Constitution Amendment Bill.
- The introduction of a constitutional amendment
bill does not require the prior recommendation
of the President. There is no provision for calling
a joint session of the Parliament in case of
disagreement between the two Houses.
29. वनम्नलिखित कथनों पर विचार करें - 29. Consider the following statements -
-:: 17 ::-
RPT 05
I. केंद्र शालसत प्रदे शों का राज्र्सभा में प्रवतवनधधत्ि नहीं है। I. Union territories are not represented in the
II. भारत के संविधान के अनुसार, संसद में केिि िोकसभा और Rajya Sabha.
राज्र्सभा शाधमल हैं। II. As per the Constitution of India, the
Parliament consists of only the Lok Sabha
III. चुनाि वििादों का वनर्यर् करना मुख्र् चुनाि आर्ुि के
and the Rajya Sabha.
कार्यक्षेत्र में आता है।
III. The decision of election disputes comes
उपययुक्त में से कौनसा/से कथन सत्य है/हैं?
under the purview of the Chief Election
(a) केिि I Commissioner.
(b) केिि II और III Which of the above statement(s) is/are true?
(c) केिि I और III (a) I only
(d) इनमें से कोई नहीं (b) II and III only
[d] (c) I and III only
व्याख्या – (d) None of these
- केंद्र शालसत प्रदे शों को राज्र्सभा में प्रवतवनधधत्ि टदर्ा गर्ा है। [d]
Explanation -
- चुनाि वििादों का वनर्यर् करना उच्च न्र्ार्ािर् और सिोच्च
- Union Territories have been given representation
न्र्ार्ािर् के कार्यक्षेत्र में आता है।
in the Rajya Sabha.
- भारत के संविधान के अनुसार, संसद राष्ट्रपवत, िोकसभा और
- Deciding election disputes comes under the
राज्र्सभा से धमिकर बनती हैं। purview of the High Court and the Supreme
Court.
- According to the Constitution of India, the
Parliament consists of the President, the Lok
Sabha and the Rajya Sabha.
30. वनम्नलिखित में से कौनसे कायथ सिोच्च न्त्यायािय के मूि 30. Which of the following functions are included
िेराधधकार में शाधमि हैं? in the original jurisdiction of the Supreme
I. भारत सरकार और एक र्ा अधधक राज्र्ों के बीच वििाद Court?
I. Dispute between the Government of India
II. दो र्ा दो से अधधक राज्र्ों के बीच वििाद
and one or more States
सही कूट
II. Dispute between two or more states
(a) केिि I
Correct code
(b) केिि II (a) I only
(c) I और II दोनों (b) II only
(d) न तो I, न िी II (c) Both I and II
[c] (d) Neither I nor II
व्याख्या – [c]
- अनुच्छे द 131 सिोच्च न्र्ार्ािर् के मूि अधधकार क्षेत्र को थपि Explanation -
करता है। र्ह प्रावधान करता है वक न्र्ार्ािर् मूि अधधकार क्षेत्र - Article 131 specifies the original jurisdiction of
the Supreme Court. It provides that the Court
का प्रर्ोग करने के लिए सक्षम होगा र्टद मामिा-
shall be competent to exercise original
- केंद्र सरकार और एक र्ा अधधक राज्र्ों के बीच वििादों में,
jurisdiction if the case-
- ऐसे वििादों में, जिााँ केंद्र सरकार और एक र्ा अधधक राज्र् एक
- In disputes between the Central Government and
पक्ष का गठन करते हैं और एक र्ा अधधक राज्र् दूसरे पक्ष का गठन one or more States,
करते हैं, - In disputes where the Central Government and
- दो र्ा दो से अधधक राज्र्ों के बीच वििादों में। one or more States constitute one party and one
or more States constitute the other party,
- In disputes between two or more states.
31. वनम्नलिखित कथनों पर विचार करें - 31. Consider the following statements -
I. भारत के अिॉनी जनरि और भारत के सॉलिलसिर जनरि I. The Attorney General of India and the
सरकार के एकमात्र अधधकारी हैं जजन्हें भारत की संसद की Solicitor General of India are the only officers
of the government who are allowed to attend
बैठकों में भाग िेने की अनुमवत है।
the meetings of the Parliament of India.

-:: 18 ::-
RPT 05
II. भारत के संविधान के अनुसार, भारत का महान्र्ार्िादी अपना II. As per the Constitution of India, the Attorney
इथतीिा तब सौंपता है जब उसे वनर्ुि करने िािी सरकार General of India submits his resignation
इथतीिा दे दे ती है। when the government that appointed him
resigns.
उपययुक्त में से कौनसा/से कथन सत्य है/हैं?
Which of the above statement(s) is/are true?
(a) केिि I
(a) I only
(b) केिि II
(b) II only
(c) I और II दोनों (c) Both I and II
(d) न तो I, न ही II (d) Neither I, nor II
[d] [d]
व्याख्या – Explanation -
भारत का महान्त्यायिादी Attorney-General of India
- अनुच्छे द 76(1) के अनुसार, राष्ट्रपवत भारत के महान्र्ार्िादी पद - According to Article 76(1), the President shall
हेतु सिोच्च न्र्ार्ािर् के न्र्ार्ाधीश वनर्ुि होने के र्ोग्र् व्यलि को appoint a person who is qualified to be appointed
as a Judge of the Supreme Court to the post of
वनर्ुि करेगा।
Attorney General of India.
- महान्र्ार्िादी को संसद की सभी प्रविर्ाओं में बोिने और
- The Attorney General has the right to speak and
भागीदारी का अधधकार है, िेवकन िह वकसी भी संसदीर् प्रविर्ा में
participate in all proceedings of Parliament, but
मतदान नहीं कर सकता। he can’t vote in any parliamentary proceedings.
- अनुच्छे द 76(4) के अनुसार, महान्र्ार्िादी राष्ट्रपवत के प्रसादपर्यन्त - As per Article 76(4), the Attorney General shall
पद धारर् करेगा और राष्ट्रपवत द्वारा वनधायररत पाररश्रधमक प्राप्त hold office during the pleasure of the President
करेगा। अत: महान्र्ार्िादी को राष्ट्रपवत द्वारा वकसी भी समर् उसे and shall receive such remuneration as may be
उसके पद से हिार्ा जा सकता है। चूाँवक उसे राष्ट्रपवत द्वारा fixed by the President. Therefore, the Attorney
मंवत्रपररषद की सिाह पर वनर्ुि वकर्ा जाता है, पारंपररक रूप से General can be removed from his post by the
िह लोकसभा के भंग होने पर इथतीिा दे दे ता है। िेवकन र्ह President at any time. Since he is appointed by
the President on the advice of the Council of
अवनिार्य नहीं है।
Ministers, he traditionally resigns when the
- महाधधििा संसद की बैठकों में भाग नहीं िेता है।
Council is dissolved. but it is not necessary.
- The Advocate General does not participate in the
meetings of the Parliament.
32. भारत के संविधान द्वारा राज्यसभा को वनम्नलिखित में से 32. Which of the following special powers have
कौन-सी विशेष शलक्तयााँ प्रदान की गई हैं? been conferred on the Rajya Sabha by the
(a) वकसी राज्र् के मौजूदा क्षेत्र को बदिने और राज्र् का नाम Constitution of India?
(a) To alter the existing area of a State and to
बदिने के लिए।
change the name of the State.
(b) राज्र् सूची पर कानून बनाने और एक र्ा अधधक अखखि
(b) To pass a resolution empowering Parliament
भारतीर् सेिाओं को स्थाहपत करने के लिए संसद को सशि
to make laws in the State List and to create
बनाने के लिए एक प्रथताि पाररत करने के लिए। one or more All India Services.
(c) राष्ट्रपवत की चुनाि प्रविर्ा में संशोधन करना और राष्ट्रपवत की (c) To amend the election process of the
सेिावनिृलत्त के बाद उनकी पेंशन का वनधायरर् करना। President and to determine the pension of
(d) चुनाि आर्ोग के कार्ों का वनधायरर् करना और चुनाि आर्ुिों the President after his retirement.
की संख्र्ा वनधायररत करना। (d) To determine the functions of the Election
[b] Commission and to determine the number of
व्याख्या – Election Commissioners.
[b]
- एक पररसंर्ीर् सदन होने के नाते राज्र् सभा को संविधान के अधीन
Explanation -
तीन विशेष शलिर्ााँ प्राप्त िैं -
- Being a federal house, the Rajya Sabha has three
- अनु.-249 के अंतगयत राज्र् सूची के विषर् पर एक िषय का
special powers under the constitution-
अधधवनर्म बनाने का िेत।य - Under article-249, the right to make an Act of one
- अनु. 312 के अंतगयत निीन अखखि भारतीर् सेिा का गठन 2/3 year on the subject of the State List.
बहुमत से करना। - Formation of new All India Service under article-
312 with 2/3rd majority.
-:: 19 ::-
RPT 05
- अनु. 67B के तित् उपराष्ट्रपवत को हिाने िािा प्रथताि राज्र्सभा - Article-67B The motion for the removal of the
में ही िार्ा जा सकेगा। Vice-President can be brought only in the Rajya
Sabha.
33. भारत के संविधान के वनम्नलिखित प्रािधानों में से कौन सा 33. Which one of the following provisions of the
लशिा पर प्रभाि डािता है? Constitution of India affects education?
I. राज्र् के नीवत वनदे शक लसद्धांत I. Directive Principles of State Policy
II. Rural and Urban Local Bodies
II. ग्रामीर् और शहरी थिानीर् वनकार्
III. Fifth schedule
III. पााँचिीं अनुसूची
IV. Sixth schedule
IV. छठी अनुसूची
V. Seventh schedule
V. सातिीं अनुसूची Correct code
सही कूट (a) I and II only
(a) केिि I और II (b) III, IV and V only
(b) केिि III, IV और V (c) I, II and V only
(c) केिि I, II और V (d) I, II, III, IV and V
(d) I, II, III, IV और V [c]
[c] Explanation -
Education is included in the following
व्याख्या –
provisions of the Constitution of India –
भारत के संविधान के वनम्नलिखित प्रािधानों में लशिा शाधमि
- Directive Principles of State Policy - Part IV
है -
- Rural and Urban Local Bodies - Part IXA
- राज्र् के नीवत वनदे शक लसद्धांत- भाग IV - Seventh Schedule - State List Subjects
- ग्रामीर् और शहरी थिानीर् वनकार्- भाग IXA
- सातिीं अनुसूची- राज्र् सूची के विषर्
34. भारत में संपलत्त के अधधकार की क्या स्स्थवत है? 34. What is the status of property rights in India?
(a) केिि नागररकों के लिए उपिब्ध कानूनी अधधकार (a) Legal rights available only to citizens.
(b) वकसी भी व्यलि को उपिब्ध कानूनी अधधकार (b) Legal rights available to any person.
(c) Fundamental rights available only to citizens.
(c) केिि नागररकों के लिए उपिब्ध मौलिक अधधकार
(d) Neither a fundamental right nor a legal right.
(d) न तो मौलिक अधधकार और न ही कानूनी अधधकार
[b]
[b]
Explanation -
व्याख्या – - The Right to Property was removed from the list
- संपलत्त के अधधकार को 44िें संशोधन अधधवनर्म, 1978 द्वारा of fundamental rights by the 44th Amendment
मौलिक अधधकारों की सूची से हिा टदर्ा गर्ा िा। इसे संविधान के Act, 1978. It has been made a statutory or
भाग XII में अनुच्छे द 300-ए के तहत् कानूनी अििा विधधक statutory right under Article 300-A in Part XII of
अधधकार बनार्ा गर्ा है। the Constitution.
35. भारत में सािथजवनक वित्त पर संसदीय वनयंरण की कौन-सी 35. What are the methods of parliamentary
विधधयााँ हैं? control over public finance in India?
I. संसद के समक्ष िार्षिंक वित्तीर् वििरर् प्रथतुत करना। I. Laying of Annual Financial Statements before
the Parliament.
II. विवनर्ोग विधेर्क पाररत होने के बाद ही भारत की संधचत वनधध
II. Withdrawal of money from the Consolidated
से धन की वनकासी।
Fund of India only after the passage of the
III. अनुपूरक अनुदान और िेखानुदान के प्रािधान।
Appropriation Bill.
IV. संसदीर् बजि कार्ायिर् द्वारा व्यापक आर्ििंक पूिायनुमानों और III. Supplementary Grants and Vote-on-Account
व्यर् के खखिाि सरकार के कार्यिम की आिधधक र्ा कम से Provisions.
कम मध्र्-िषय की समीक्षा। IV. Periodic or at least mid-year review of the
V. संसद में वित्त विधेर्क पेश करना। government's program against
उपययुक्त में से कौनसा/से कथन सत्य है/हैं? macroeconomic forecasts and expenditure
(a) केिि I, II, III और V by the Parliamentary Budget Office.
(b) केिि I, II और IV V. Introduction of Finance Bill in Parliament.
Which of the above statement(s) is/are true?
(c) केिि III, IV और V
(a) I, II, III and V only
-:: 20 ::-
RPT 05
(d) I, II, III, IV और V (b) I, II and IV only
[a] (c) III, IV and V only
व्याख्या – (d) I, II, III, IV and V
[a]
भारत में सािथजवनक वित्त पर संसदीय वनयंरण के तरीके -
Explanation -
- भारतीर् संविधान का अनुच्छे द-112 संसद में िार्षिंक वित्तीर्
Methods of parliamentary control over public
वििरर् रखने से संबंधधत है।
finance in India -
- विवनर्ोग विधेर्क एक धन विधेर्क है जो सरकार को वित्त िषय के - Article-112 of the Indian Constitution is related to
दौरान अपने खचों को पूरा करने के लिए भारत के संधचत वनधध से keeping the annual financial statement in the
धन वनकािने की अनुमवत दे ता है। Parliament.
- संविधान के अनुच्छे द-114 के अनुसार, सरकार संसद से अनुमोदन - An Appropriation Bill is a money bill that allows
प्राप्त करने के बाद ही इससे धन वनकाि सकती है। the government to withdraw money from the
- िेखानुदान, िेधडि अनुदान और असाधारर् अनुदान संविधान के Consolidated Fund of India to meet its expenses
अनुच्छे द-116 में वनर्दिंि हैं। during the financial year.
- According to Article-114 of the Constitution, the
- सरकार के कार्यिमों की समीक्षा करने के लिए कोई संसदीर् बजि
government can withdraw money from it only
कार्ायिर् नहीं है। र्द्यवप नीवतर्ों की समीक्षा करने और िषयभर
after obtaining approval from the Parliament.
समर्-समर् पर अनुमानों की वनरंतर जााँच करने के लिए संसद की
- Vote on account, credit vote and extraordinary
एक अनुमावनत सधमवत है। vote are specified in Article 116 of the
- वित्त विधेर्क केंद्रीर् बजि का एक वहथसा है, जो वित्त मंत्री द्वारा Constitution.
प्रथतावित कराधान में बदिाि के लिए आिश्र्क सभी कानूनी - There is no parliamentary budget office to review
संशोधनों को वनधायररत करता है। government programs. Although there is an
Estimates Committee of the Parliament to review
the policies and keep a constant check on the
estimates from time to time throughout the year.
- The Finance Bill is a part of the Union Budget,
which sets out all the legal amendments required
for changes in taxation proposed by the Finance
Minister.
36. वनम्नलिखित में से कौन-सी विशेषता भारत सरकार 36. Which of the following is a feature of the
अधधवनयम, 1919 की है? Government of India Act, 1919?
I. प्रांतों की कार्यकारी सरकार में द्वै ध शासन का पररचर् I. Introduction of diarchy in the executive
government of the provinces
II. मुसिमानों के लिए अिग साम्प्रदावर्क वनिायचक मंडि की
II. Introduction of separate communal
शुरुआत
electorate for Muslims
III. केंद्र द्वारा प्रांतों को विधार्ी शलि का हथतांतरर्
III. Transfer of legislative power by the center to
उपययुक्त में से कौनसा/से कथन सत्य है/हैं? the provinces
(a) केिि I Which of the above statement(s) is/are true?
(b) केिि II और III (a) I only
(c) केिि I और III (b) II and III only
(d) I, II और III (c) I and III only
[c] (d) I, II and III
व्याख्या – [c]
Explanation -
भारत सरकार अधधवनयम, 1919 की प्रमुि विशेषता -
Salient features of the Government of India
- प्रान्तीर् थतर पर द्वै ध शासन थिावपत वकर्ा गर्ा।
Act, 1919 -
- 1919 के भारत सरकार अधधवनर्म ने केंद्रीर् और प्रांतीर् विषर्ों
- Diarchy was established at the provincial level.
को िगीकृत करने के लिए एक प्रर्ािी थिावपत की। इस अधधवनर्म - The Government of India Act of 1919 established
ने आर्कर को संर्ीर् सरकार के लिए एक राजथि स्रोत के रूप में a system for classifying central and provincial
संरणक्षत वकर्ा। subjects. The act preserved the income tax as a
revenue stream for the federal government.

-:: 21 ::-
RPT 05
37. यदद राज्यसभा द्वारा धन विधेयक में पयाथप्त संशोधन वकया 37. What happens if substantial amendments are
जाता है तो क्या होगा? made to the Money Bill by the Rajya Sabha?
(a) िोकसभा अभी भी राज्र्सभा की लसिाररशों को थिीकार र्ा (a) Lok Sabha can still go ahead with the bill
accepting or not accepting the
थिीकार नहीं करते हुए विधेर्क के साि आगे बढ सकती है।
recommendations of Rajya Sabha.
(b) िोकसभा आगे विधेर्क पर विचार नहीं कर सकती है।
(b) Lok Sabha cannot consider the bill further.
(c) िोकसभा विधेर्क को पुनर्ििंचार के लिए राज्र् सभा को भेज
(c) Lok Sabha can send the bill to Rajya Sabha
सकती है। for reconsideration.
(d) विधेर्क पाररत करने के लिए राष्ट्रपवत संर्ुि बैठक बुिा सकते (d) President can call a joint sitting to pass the
हैं। [a] bill.
व्याख्या – [a]
- धन विधेर्क में िोकसभा को पूर्य अधधकार है, अतः िोकसभा Explanation -
अभी भी राज्र्सभा की लसिाररशों को थिीकार र्ा अथिीकार करते - Lok Sabha has absolute power in money bill so
हुए विधेर्क के साि आगे बढ सकती है। Lok Sabha can still go ahead with the bill
accepting or not accepting the recommendations
of Rajya Sabha.
38. भारत में न्त्यायाियों द्वारा जारी ररटों के संदभथ में, वनम्नलिखित 38. With reference to writs issued by courts in
कथनों पर विचार करें - India, consider the following statements -
I. परमादे श वकसी वनजी संगठन के विरुद्ध नहीं होगा जब तक वक I. The injunction shall not be against a private
organization unless it is entrusted with a
उसे साियजवनक कत्तयव्य न सौंपा गर्ा हो।
public duty.
II. परमादे श वकसी कंपनी के विरुद्ध नहीं होगा चाहे िह सरकारी
II. Mandamus will not be against any company
कंपनी ही क्र्ों न हो।
even if it is a government company.
III. कोई भी साियजवनक विचारधारा िािा व्यलि अधधकार-पृच्छा III. Any public minded person can be a petitioner
ररि प्राप्त करने के लिए न्र्ार्ािर् जाने के लिए र्ाधचकाकताय to approach the court to obtain a writ of Quo-
हो सकता है। warranto.
उपययुक्त में से कौनसा/से कथन सत्य है/हैं? Which of the above statement(s) is/are true?
(a) केिि I और II (a) I and II only
(b) केिि II और III (b) II and III only
(c) केिि I और III (c) I and III only
(d) I, II and III
(d) I, II और III
[c]
[c]
Explanation -
व्याख्या –
- Mandamus- It literally means 'We Command'. It
- परमादे श- इसका शाखब्दक अिय है 'हम आदे श दे ते हैं'। र्ह अदाित is an order issued by the court to a public official
द्वारा एक साियजवनक अधधकारी को जारी वकर्ा गर्ा एक आदे श है asking him to perform his official duties when he
जो उसे अपने आधधकाररक कत्तयव्यों का पािन करने के लिए कहता has failed to do so or has refused to perform
है जबवक िह ऐसा करने में वििि रहा है र्ा उसने प्रदशयन करने से them. It can also be issued against any public
इनकार कर टदर्ा है। र्ह वकसी भी साियजवनक वनकार्, वनगम, अिर body, corporation, inferior court, tribunal or
न्र्ार्ािर्, न्र्ार्ाधधकरर् र्ा सरकार के खखिाि भी इसी उद्दे श्र् के government for the same purpose. Mandamus
लिए जारी वकर्ा जा सकता है। परमादे श ररि र्ा र्ाधचका जारी नहीं writ or petition cannot be issued against private
persons and companies which have no public
की जा सकती र्ह उन वनजी व्यलिर्ों और कंपवनर्ों के विरुद्ध नहीं
duty.
की जाएगी जजनका कोई साियजवनक कत्तयव्य नहीं है।
39. भारत के महान्त्यायिादी के अधधकार से संबंधधत वनम्नलिखित 39. Consider the following statements regarding
कथनों पर विचार कीजजए - rights of the Attorney General of India -
I. िोकसभा की कार्यिाही में भाग िेना I. Participating in the proceedings of the Lok
Sabha
II. िोकसभा की वकसी सधमवत का सदथर् होना
II. To be a member of a committee of the Lok
III. िोकसभा में बोिना
Sabha
IV. िोकसभा में मतदान करना
III. Right tp speak and to take part in the Lok
उपययुक्त में से कौनसा/से कथन सत्य है/हैं? Sabha

-:: 22 ::-
RPT 05
(a) केिि I IV. Right to vote in Lok Sabha
(b) केिि II और IV Which of the above statement(s) is/are true?
(c) केिि I, II और III (a) I only
(b) II and IV only
(d) केिि I और III
(c) I, II and III only
[c]
(d) I and III only
व्याख्या –
[c]
- भारत के महान्र्ार्िादी को संसद के दोनों सदनों र्ानी िोकसभा Explanation -
और राज्र्सभा की कार्यिाही में बोिने और भाग िेने का अधधकार - The Attorney General of India has the right to
है। speak and participate in the proceedings of both
- भारत का महान्र्ार्िादी संसद की वकसी सधमवत का सदथर् हो the Houses of the Parliament i.e. Lok Sabha and
सकता है। Rajya Sabha.
- भारत के महान्र्ार्िादी के पास संसद की कार्यिाही में मतदान का - The Attorney General of India can be a member
कोई अधधकार नहीं होता है। of any committee of the Parliament.
- The Attorney General of India does not have any
- भारत के महान्र्ार्िादी को सरकारी कमयचारी के रूप में नहीं माना
right to vote in the proceedings of the
जाता है, बस्ल्क वि संर् कार्यकाररर्ी का एक वहथसा है।
Parliament.
- भारत के महान्र्ार्िादी केंद्र सरकार के खखिाि एक संणक्षप्त सिाह
- The Attorney General of India is not considered
र्ा रार् नहीं दे सकते, साि ही केंद्र सरकार की अनुमवत के वबना as a government servant, but he is a part of the
वकसी आरोपी का बचाि नहीं कर सकते। Union Executive.
- The Attorney General of India cannot give a brief
advice or opinion against the Central
Government, as cannot well as defend an
accused without the permission of the Central
Government.
40. वनम्नलिखित कथनों पर विचार करें - 40. Consider the following statements -
I. राष्ट्रपवत भारत सरकार के कार्य के अधधक सुविधाजनक I. The President shall make rules for the more
संचािन के लिए तिा उि कार्य के मंवत्रर्ों के बीच आिंिन के convenient transaction of the business of the
Government of India and for the allocation
लिए वनर्म बनार्ेंगे।
among Ministers of the said business.
II. भारत सरकार की सभी कार्यकारी कारयिाइर्ााँ प्रधानमंत्री के
II. All executive actions of the Government of
नाम पर की जाएाँगी।
India will be taken in the name of the Prime
उपययुक्त में से कौनसा/से कथन सत्य है/हैं? Minister.
(a) केिि I Which of the above statement(s) is/are true?
(b) केिि II (a) I only
(c) I और II दोनों (b) II only
(d) न तो I, न ही II (c) Both I and II
[a] (d) Neither I nor II
व्याख्या – [a]
Explanation -
- भारतीर् संविधान के अनुच्छे द 77 (3) ‘भारत सरकार के कार्ु का
- Article 77 (3) of the Constitution of India 'Conduct
संचािन’ र्े कहता है ‘राष्ट्रपवत भारत सरकार के कार्ु के अधधक
of the business of the Government of India'
सुविधाजनक संचालन के क्तलए और उि कार्ु के मंवत्रर्ों के बीच
states that 'The President shall make rules for the
आिंिन के लिए वनर्म बनाएंगे।‘ more convenient transaction of the business of
- भारतीर् संविधान के अनुच्छे द 77 के खंड (1) में र्ह उपबंध है वक the Government of India and for the allocation
‘भारत सरकार की सभी कार्यकारी कारयिाई राष्ट्रपवत के नाम पर की among Ministers of the said business.'
जानी चावहए’। - Clause (1) of Article 77 of the Constitution of India
provides that 'all executive action of the
Government of India shall be taken in the name
of the President'.
41. भारतीय संविधान के उद्दे श्यों में से एक के रूप में 'आर्थिक 41. 'Economic Justice' has been provided as one of
न्त्याय' प्रदान वकया गया है - the objectives of the Indian Constitution -
-:: 23 ::-
RPT 05
(a) प्रथतािना और मौलिक अधधकार में (a) Preamble and Fundamental Rights
(b) प्रथतािना और राज्र् नीवत के वनदे शक लसद्धांत में (b) Preamble and Directive Principles of State
(c) मौलिक अधधकार और राज्र् नीवत के वनदे शक लसद्धांत में Policy
(c) Fundamental Rights and Directive Principles
(d) उपर्ुयि में से कोई नहीं
of State Policy
[b]
(d) None of the above
व्याख्या –
[b]
- प्रस्तािना का उद्दे श्य - सभी नागररकों को न्र्ार्, थितंत्रता, Explanation -
समानता को सुरणक्षत करना और भारत की एकता और अखंडता - Objective of the Preamble - To secure justice,
को बनाए रखने के लिए भाईचारे को बढािा दे ना। liberty, equality to all citizens and promote
- भारतीर् संविधान की प्रथतािना अपने िोगों के सामाजजक, आर्ििंक fraternity to maintain the unity and integrity of
और राजनीवतक न्र्ार् को सुरणक्षत करती है। India.
- भारतीर् संविधान की प्रथतािना विचार, अणभव्यलि, विश्वास, आथिा - The Preamble of the Indian Constitution secures
और पूजा वक थिंत्रता अपने िोगों को दे ती है। social, economic and political justice to its people.
- The Preamble of the Indian Constitution gives
- भारतीर् संविधान की प्रथतािना अपने िोगों को समानता और
freedom of thought, expression, belief, faith and
स्थिवत का अिसर दे ती है।
worship to its people.
राज्य के नीवत वनदे शक लसद्ांत :
- The Preamble of the Indian Constitution gives its
- राज्र् के नीवत वनदे शक लसद्धांत के अनुच्छे द 38 के अनुसार, राज्र् people the opportunity of equality and status.
अपने सामाजजक, आर्ििंक और राजनीवतक न्र्ार् को सुरणक्षत और Directive Principles of State Policy:
संरणक्षत करके भारत के िोगों के कल्र्ार् को बढािा दे ने का प्रर्ास - According to Article 38 of the Directive Principles
करेगा। of State Policy, the State shall endeavor to
promote the welfare of the people of India by
securing and protecting their social, economic
and political justice.
42. ‘िोकिेिा’ संबंधी संसदीय सधमवत के संबंध में वनम्नलिखित 42. Consider the following statements regarding
कथनों पर विचार करें- the Parliamentary Committee on 'Public
I. इसमें िोकसभा के 25 से अधधक सदथर् नहीं होते हैं। Accounts'-
I. It consists of not more than 25 members of
II. र्ह सरकार के विवनर्ोग वित्त खातों की जााँच करता है।
the Lok Sabha
III. र्ह भारत के वनर्ंत्रक और महािेखा परीक्षक की ररपोिय की
II. It examines the Appropriation Finance
जााँच करता है।
Accounts of the Government
उपययुक्त में से कौनसा/से कथन सत्य है/हैं? III. It examines the reports of the Comptroller
(a) केिि I and Auditor General of India
(b) केिि II और III Which of the above statement(s) is/are true?
(c) केिि III (a) I only
(d) I, II और III (b) II and III only
[b] (c) III only
व्याख्या – (d) I, II and III
[b]
- िोक िेखा सधमवत में 22 से अधधक सदथर् नहीं होते हैं, जजसमें प्रवत
Explanation -
िषय िोकसभा में 15 सदथर् होते हैं, जो इसके सदथर्ों में से होते हैं।
- The Public Accounts Committee consists of not
- िोकिेखा सधमवत भारत सरकार के रेििे, रक्षा सेिाओं, पीएंडिी
more than 22 members, with 15 members in the
विभाग और अन्र् लसविि मंत्रािर्ों से संबंधधत विवनर्ोग खातों की Lok Sabha per year from among its members.
जााँच करती है और वनर्ंत्रक एिं महािेखा परीक्षक की ररपोिें और - The Public Accounts Committee examines the
साि ही साि वनर्ंत्रक विभाग की ररपोिय और राजथि प्राप्प्तर्ों पर Appropriation Accounts relating to Railways,
महािेखा परीक्षक मुख्र् रूप से सधमवत के विचार-विमशय का आधार Defense Services, P&T Department and other
बनाते हैं। Civil Ministries of the Government of India and
- वनर्ंत्रक एिं महािेखा परीक्षक इस सधमवत का "धमत्र, दाशयवनक और reports of the Comptroller and Auditor General
मागयदशयक" होता है। िह सधमवत की बैठक में भाग िेता है और as well as the reports of the Comptroller and
Auditor General on Revenue Receipts mainly
इसके विचार-विमशय में सहार्ता करता है।

-:: 24 ::-
RPT 05
form the basis for the deliberations of the
committee.
- The Comptroller and Auditor General is the
"friend, philosopher and guide" of this
committee. He attends the meeting of the
committee and helps in its deliberations.
43. वनम्नलिखित में से वकसे अवनिायथ रूप से समािेशी शासन का 43. Which of the following can essentially be
अंग कहा जा सकता है? called a part of inclusive governance?
I. गैर-बैंककिंग वित्तीर् कंपवनर्ों को बैंककिंग करने की अनुमवत दे ना। I. Permitting non-banking financial companies
to do banking.
II. सभी जजिों में प्रभािी जजिा र्ोजना सधमवतर्ों की थिापना
II. Establishment of effective district planning
करना।
committees in all districts.
III. साियजवनक थिाथथ्र् पर सरकारी खचय बढाना।
III. Increase government spending on public
IV. मध्र्ाह्न भोजन र्ोजना को सुदृढ बनाना। health.
सही कूट IV. Strengthening the Mid Day Meal Scheme.
(a) केिि I और II Correct Code
(b) केिि III और IV (a) I and II only
(c) केिि II, III और IV (b) III and IV only
(d) I, II, III और IV (c) II, III and IV only
[c] (d) I, II, III and IV
[c]
व्याख्या –
Explanation –
- सभी जजिों में प्रभािशािी जजिा सधमवतर्ों की थिापना करना, जन-
- Establishing effective district committees in all
थिाथथ्र् पर सरकारी खचय में बढोत्तरी करना तिा मध्र्ाह्न भोजन
districts, increasing government expenditure on
र्ोजना को सशिीकरर् प्रदान करना समािेशी शासन के विणभन्न public health and strengthening the mid-day
अंग हैं। गैर-बैंककिंग वित्तीर् कंपवनर्ों को बैंककिंग करने की अनुमवत meal scheme are various parts of inclusive
दे ने को समािेशी शासन का भाग नहीं माना जा सकता है। governance. Allowing NBFCs to do banking
cannot be considered a part of inclusive
governance.
44. वनम्नलिखित कथनों पर विचार करें - 44. Consider the following statements -
I. केंद्र में मंवत्रपररषद सामूवहक रूप से संसद के प्रवत उत्तरदार्ी I. The Council of Ministers at the Center shall
होगी। be collectively responsible to the Parliament.
II. The Union Ministers shall hold office during
II. केन्द्रीर् मंत्री भारत के राष्ट्रपवत के प्रसादपर्ंत पदधारर् करेंगे।
the pleasure of the President of India.
III. प्रधानमंत्री कानून के प्रथतािों के बारे में राष्ट्रपवत को सूधचत
III. The Prime Minister will inform the President
करेंगे।
about the proposals for legislation.
उपययुक्त में से कौनसा/से कथन सत्य है/हैं? Which of the above statement(s) is/are true?
(a) केिि I (a) I only
(b) केिि II और III (b) II and III only
(c) केिि I और III (c) I and III only
(d) I, II और III (d) I, II and III
[b] [b]
व्याख्या – Explanation -
- The Council of Ministers at the Center shall be
- केंद्र में मंवत्रपररषद सामूवहक रूप से िोकसभा के प्रवत उत्तरदार्ी
collectively responsible to the Lok Sabha. Article-
होगी। अनुच्छे द-75(3)
75(3)
- केन्द्रीर् मंत्री भारत के राष्ट्रपवत के प्रसादपर्ंत पदधारर् करेंगे।
- The Union Ministers shall hold office during the
अनयच्छे द-75(2) pleasure of the President of India. Article-75(2)
- प्रधानमंत्री कानून के प्रथतािों के बारे में राष्ट्रपवत को सूधचत करेंगे। - The Prime Minister will inform the President
about the proposals for legislation.
45. वनम्नलिखित कथनों पर विचार कीजजए - 45. Consider the following statements -
I. राष्ट्रीर् विकास पररषद र्ोजना आर्ोग का एक अंग है।
-:: 25 ::-
RPT 05
II. भारत के संविधान में आर्ििंक एिं सामाजजक वनर्ोजन को I. The National Development Council is a part
समिती सूची में रखा गर्ा है। of the Planning Commission.
III. भारत का संविधान वनधायररत करता है वक पंचार्तों को आर्ििंक II. Economic and social planning has been kept
in the concurrent list in the Constitution of
विकास और सामाजजक न्र्ार् के लिए र्ोजनाएाँ तैर्ार करने का
India.
कार्य सौंपा जाना चावहए।
III. The Constitution of India stipulates that
उपययुक्त में से कौनसा/से कथन सत्य है/हैं?
Panchayats should be entrusted with the
(a) केिि I task of preparing plans for economic
(b) केिि II और III development and social justice.
(c) केिि I और III Which of the above statement(s) is/are true?
(d) I, II और III (a) I only
[b] (b) II and III only
व्याख्या – (c) I and III only
- NDC को र्ोजना आर्ोग के सिाहकार वनकार् के रूप में सूचीबद्ध (d) I, II and III
[b]
वकर्ा गर्ा िा। इसके अिािा, र्ोजना आर्ोग द्वारा बनाई गई
Explanation -
र्ोजनाओं को इसकी थिीकृवत के लिए NDC के समक्ष रखा जाता
- The NDC was listed as an advisory body to the
था।
Planning Commission. In addition, the plans
- अब, राष्ट्रीर् विकास पररषद (NDC) और र्ोजना आर्ोग (PC) का made by the Planning Commission were placed
अंत कर टदर्ा गर्ा है। before the NDC for its approval.
- 73िें संशोधन 1992 ने संविधान में एक नर्ा भाग IX जोड़ा जजसका - Now, the National Development Council (NDC)
शीषयक िा ‘पंचार्त’ जजसमें अनुच्छे द 243 से 243 (O) के and Planning Commission (PC) have been
प्रािधान; और पंचार्तों के कार्ों के 29 विषर्ों को शाधमि करते abolished.
हुए एक नई ग्र्ारहिीं अनुसूची शाधमि हैं। - The 73rd Amendment 1992 added a new Part IX to
the Constitution titled ‘Panchayats’ containing
the provisions of Articles 243 to 243(O); and a
new Eleventh Schedule comprising 29 subjects
within the functions of the Panchayats.
46. वनम्नलिखित कथनों पर विचार कीजजए - 46. Consider the following statements -
I. भारत के संविधान में संशोधन केिि िोकसभा में एक विधेर्क I. The Constitution of India can be amended
पाररत करके वकर्ा जा सकता है। only by passing a bill in the Lok Sabha.
II. If such an amendment seeks to change the
II. र्टद ऐसा कोई संशोधन संविधान के संर्ीर् ढााँचा में पररितयन
federal structure of the constitution, then the
करना चाहता है, तो संशोधन को भारत के सभी राज्र्ों की
amendment also needs to be ratified by the
विधावर्का द्वारा अनुसमर्ििंत करने की भी आिश्र्कता है।
legislature of all the states of India.
उपययुक्त में से कौनसा/से कथन सत्य है/हैं? Which of the above statement(s) is/are true?
(a) केिि I (a) I only
(b) केिि II (b) II only
(c) I और II दोनों (c) Both I and II
(d) न तो I, न ही II (d) Neither I nor II
[d] [d]
व्याख्या – Explanation -
- The Constitution of India can be amended by
- भारत के संविधान में संशोधन संसद में एक विधेर्क पाररत करके
passing a bill in the Parliament.
वकर्ा जा सकता है।
- If such an amendment seeks to change the
- र्टद ऐसा कोई संशोधन संविधान के संर्ीर् ढााँचा में पररितयन करना
federal structure of the constitution, then the
चाहता है, तो संशोधन को भारत के आधे राज्र्ों की विधावर्का द्वारा amendment also needs to be ratified by the
अनुसमर्ििंत करने की भी आिश्र्कता है। legislature of half the states of India.
47. वनम्नलिखित कथनों पर विचार करें - 47. Consider the following statements -
I. राज्र्सभा के सभापवत और उपसभापवत उस सदन के सदथर् I. The Chairman and the Deputy Chairman of
नहीं होते हैं। the Rajya Sabha are not members of that
House.
-:: 26 ::-
RPT 05
II. संसद के दोनों सदनों के मनोनीत सदथर्ों को राष्ट्रपवत चुनाि में II. While the nominated members of both the
मतदान का अधधकार नहीं होता है, जबहक उन्हें उपराष्ट्रपवत के Houses of Parliament do not have the right
चुनाि में िोि दे ने का अधधकार होता है। to vote in the election of the President, they
do have the right to vote in the election of the
उपययुक्त में से कौनसा/से कथन सत्य है/हैं?
Vice President.
(a) केिि I
Which of the above statement(s) is/are true?
(b) केिि II
(a) I only
(c) I और II दोनों (b) II only
(d) न तो I, न ही II (c) Both I and II
[b] (d) Neither I nor II
व्याख्या – [b]
- कथन I गलत िै क्र्ोंवक र्ि सभापहत के साि-साि उपसभापहत Explanation -
की भी बात करता है। - Statement I is wrong because it talks about the
- भारत के उपराष्ट्रपवत राज्र्सभा के पदे न अध्र्क्ष होते हैं। Chairman as well as the Deputy Chairman.
- The Vice President of India is the ex-officio
- राज्र्सभा भी अपने सदथर्ों में से एक उपाध्र्क्ष का चर्न करती है।
chairman of the Rajya Sabha.
- राष्ट्रपवत का वनिाथचक मंडि - संसद के वनिायधचत सदथर्
- The Rajya Sabha also elects a Deputy chairman
(िोकसभा के साि-साि राज्र् सभा के सांसद) टदल्िी और पयडयचेरी
from among its members.
के हवधानसभा सवहत राज्र् विधार्ी सदथर्ों के वनिायधचत सदथर्। - Electoral College of President - Elected
- उपराष्ट्रपवत का वनिाथचक मंडि - उपराष्ट्रपवत का चुनाि अप्रत्र्क्ष Members of Parliament (MPs from Lok Sabha as
रूप से संसद के दोनों सदनों के सदथर्ों के वनिायचक मंडि द्वारा well as Rajya Sabha) Elected Members of State
वकर्ा जाता है। मनोनीत सदस्र् भी चयनाव में हिस्सा लेते िैं। Legislatures including NCT of Delhi and
Puducherry.
- Electoral College of Vice-President - The Vice-
President is indirectly elected by the Electoral
College of the members of both the Houses of
Parliament. Nominated members also take part
in the elections.
48. भारत के संदभथ में, वनम्नलिखित में से कौन-सा/से लसद्ांत 48. In the context of India, which of the following
संसदीय सरकार में संस्थागत रूप से वनवहत है/हैं? principle(s) is/are institutionally inherent in
I. मंवत्रमंडि के सदथर् संसद के सदथर् होते हैं। parliamentary government?
I. The members of the cabinet are the
II. लोकसभा में विश्वास होने तक मंत्री अपने पद पर बने रहते हैं।
members of the parliament.
III. मंवत्रमंडि का प्रमुख राज्र् का प्रमुख होता है।
II. Till the confidence of the Loksabha, the
कूट -
ministers remain in their posts.
(a) केिि I और II III. The head of the cabinet is the head of state.
(b) केिि III Code -
(c) केिि II और III (a) I and II only
(d) I, II और III (b) III only
[a] (c) II and III only
व्याख्या – (d) I, II and III
संसदीय सरकार - [a]
Explanation -
- मंवत्रमंडि के सदथर् संसद के सदथर् होते हैं।
Parliamentary Government -
- मंत्री तब तक पद पर रहते हैं जब तक उन्हें लोकसभा में बहुमत प्राप्त
- The members of the cabinet are the members of
होता हैं अिायत लोकसभा में विश्वास होने तक मंत्री अपने पद पर बने
the parliament.
रहते हैं। - Ministers hold the office till they get majority in
- राज्र् का प्रमुख राष्ट्रपवत होता है और िह मंवत्रमंडि का प्रमुख नहीं the Loksabha, that is, till the confidence of the
होता है। Loksabha, the ministers remain in their post.
- The head of the state is the president and he is
not the head of the cabinet.

-:: 27 ::-
RPT 05
49. भारत में अविश्िास प्रस्ताि के संबंध में वनम्नलिखित कथनों 49. Consider the following statements regarding
पर विचार करें: No Confidence Motion in India:
I. भारत के संविधान में अविश्वास प्रथताि का कोई उल्िेख नहीं I. There is no mention of no-confidence motion
in the Constitution of India.
है।
II. No-confidence motion can be introduced
II. अविश्वास प्रथताि केिि िोकसभा में पेश वकर्ा जा सकता है।
only in the Lok Sabha.
उपययुक्त में से कौनसा/से कथन सत्य है/हैं?
Which of the above statement(s) is/are true?
(a) केिि I (a) I only
(b) केिि II (b) II only
(c) I और II दोनों (c) Both I and II
(d) न तो I, न ही II (d) Neither I nor II
[c] [c]
व्याख्या – Explanation -
- भारतीर् संविधान का अनुच्छे द 75 र्ह वनर्दिंि करता है वक - Article 75 of the Indian Constitution specifies that
the Council of Ministers shall be collectively
मंवत्रपररषद् सामूवहक रूप से िोक सभा के प्रवत उत्तरदार्ी होगी।
responsible to the Lok Sabha.
- अविश्वास प्रथताि िोकसभा में ही पेश वकर्ा जा सकता है।
- No-confidence motion can be presented in the
- भारतीर् संविधान का अनुच्छे द 118 संसद के दोनों सदनों
Lok Sabha only.
(िोकसभा और राज्र्सभा) को अपने कामकाज के अपने वनर्म - Article 118 of the Indian Constitution allows both
बनाने की अनुमवत दे ता है। the Houses of Parliament (Lok Sabha and Rajya
- भारतीर् संविधान में ‘अविश्वास’ प्रथताि का कोई उल्िेख नहीं है। Sabha) to frame their own rules of functioning.
िोकसभा के वनर्म 198 में अविश्वास प्रथताि की प्रविर्ा का - There is no mention of a "no-confidence" motion
उल्िेख है। in the articles of the Indian Constitution. Rule 198
of the Lok Sabha mentions the procedure for a
no-confidence motion.
50. वनम्नलिखित में से कौन संसद की सबसे बडी सधमवत है? 50. Which of the following is the largest
(a) िोक िेखा संबंधी सधमवत committee of the Parliament?
(b) प्राक्किन सधमवत (a) Committee on Public Accounts
(b) Estimates Committee
(c) साियजवनक उपिमों संबंधी सधमवत
(c) Committee on Public Undertakings
(d) र्ाधचका सधमवत
(d) Petition Committee
[b]
[b]
व्याख्या – Explanation -
प्राक्किन सधमवत - Estimates Committee -
- र्ह संसद की सबसे बड़ी सधमवत है। - It is the largest committee of the Parliament.
- प्राक्किन सधमवत में 30 सदथर् हैं और र्े सभी सदथर् िोकसभा से - There are 30 members in the Estimates
होते हैं। र्ह सधमवत अियव्यिथिा और व्यर् में दक्षता की ररपोिय Committee and all these members are from the
करती है। सदथर्ों को हर साि एक दूसरे के बीच से एक एकि Lok Sabha. This committee reports on economy
हथतांतरर्ीर् िोि के माध्र्म से आनुपावतक प्रवतवनधधत्ि प्रणाली and efficiency in expenditure. The members are
elected by the Lok Sabha members every year
द्वारा िोकसभा सदथर्ों द्वारा चुना जाता है तावक सभी दिों को इसमें
from amongst each other by the principles of
एक उधचत प्रवतवनधधत्त्ि धमि सके।
proportional representation by means of a single
- एक मंत्री को प्राक्किन सधमवत के सदथर्/अध्र्क्ष के रूप में नहीं
transferable vote so that all parties get a fair
चुना जा सकता है। representation in it.
िोक िेिा सधमवत - - A minister cannot be selected as the
- िोक िेखा सधमवत में 22 सदथर् िोते हैं, जजनमें से 15 िोकसभा से member/chairman of the Estimates Committee.
िोते हैं जबवक 7 राज्र्सभा से होते हैं। Public Accounts Committee -
सािथजवनक उपक्रमों की सधमवत - - The Public Accounts Committee has 22 members,
- साियजवनक उपिमों की सधमवत में संसद के दोनों सदनों के 22 out of which 15 are from the Lok Sabha while 7
सदथर् होते हैं (15 िोकसभा से और 7 राज्र्सभा से)। are from the Rajya Sabha.
Committee on Public Undertakings -

-:: 28 ::-
RPT 05
- िोकसभा अध्र्क्ष द्वारा वनर्ुि इस सधमवत का अध्र्क्ष हमेशा - The Committee on Public Undertakings consists
िोकसभा से होता है। of 22 members from both the Houses of
याधचका सधमवत - Parliament (15 from Lok Sabha and 7 from Rajya
Sabha).
- िोकसभा और राज्र्सभा दोनों में ही र्ाधचकाओं पर उनकी अपनी
- The chairman of this committee appointed by the
सधमवत िोती है। िोकसभा र्ाधचका सधमवत में 15 सदथर् हैं, जबवक
Speaker of the Lok Sabha is always from the Lok
राज्र्सभा र्ाधचका सधमहत में 10 सदथर् हैं।
Sabha.
Petition Committee -
- Both Lok Sabha and Rajya Sabha have their
committee on petitions. The Lok Sabha
committee has 15 members, while the Rajya
Sabha has 10 members.
51. एक कानून जो कायथकारी या प्रशासवनक प्राधधकरण को कानून 51. A law which confers on the executive or
के अनयप्रयोग के मामिे में एक अवनदे लशत और अवनयंवरत administrative authority an undirected and
वििेकाधीन शलक्त प्रदान करता है, भारत के संविधान के uncontrolled discretionary power in the
matter of application of the law violates which
वनम्नलिखित अनुच्छे दों में से वकसका उल्िंघन करता है?
of the following Articles of the Constitution of
(a) अनुच्छे द 14
India?
(b) अनुच्छे द 28
(a) Article 14
(c) अनुच्छे द 32 (b) Article 28
(d) अनुच्छे द 44 (c) Article 32
[a] (d) Article 44
व्याख्या – [a]
अनुच्छे द 14 - Explanation -
- राज्र् वकसी भी व्यलि को कानून के समक्ष समानता र्ा भारत के Article 14 -
क्षेत्र में कानूनों के समान संरक्षर् से िंधचत नहीं करेगा। उि अनुच्छे द - The State shall not deny to any person is equality
before the law or the equal protection of the laws
थपि रूप से दो भागों में है - र्ह राज्र् को वकसी व्यलि को 'कानून
within the territory of India. The said article is
के समक्ष समानता' से िंधचत नहीं करने का आदे श दे ता है, साथ िी
clearly in two parts - while it enjoins the State not
र्ह राज्र् को कानूनों के समान संरक्षर् से इनकार नहीं करने का भी
to deprive any persons 'equality before law', it
आदे श दे ता है। कानून िागू करने के मामिे में कार्यकारी र्ा also enjoins the State not to deny equal
प्रशासवनक प्राधधकरर् को एक अहनदे क्तशत र्ा अवनर्ंवत्रत protection of the laws. The law confers an
वििेकाधीन शलि प्रदान करता है र्ा इसे मामिों का चर्न करने की unfettered or unchecked discretionary power on
अनुमवत दे ता है जो अनुच्छे द 14 का उल्िंर्न करता है। the executive or administrative authority in the
matter of enforcement or allows it to select cases
which violates Article 14.
52. केंद्र और राज्यों के बीच वििादों का फैसिा करने के लिए भारत 52. The power of the Supreme Court of India to
के सिोच्च न्त्यायािय की शलक्त इसके अंतगथत आती है? decide disputes between the Center and the
(a) सिाहकार क्षेत्राधधकार States comes under?
(a) Advisory jurisdiction
(b) अपीिीर् क्षेत्राधधकार
(b) Appellate jurisdiction
(c) मूि क्षेत्राधधकार
(c) Original Jurisdiction
(d) ररि क्षेत्राधधकार
(d) Writ jurisdiction
[c] [c]
व्याख्या – Explanation -
सिोच्च न्त्यायािय के िेराधधकार और शलक्तयां - Jurisdiction and Powers of the Supreme Court-
अनुच्छे द-131 - मूि िेराधधकार Article-131 - Original Jurisdiction
- सिोच्च न्र्ार्ािर् (भारत की एक संर्ीर् अदाित के रूप में) - The Supreme Court (as a federal court of India)
भारतीर् संर् की विणभन्न इकाइर्ों के बीच उत्पन्न होने िािे वििादों has original jurisdiction to decide disputes
को तर् करने के लिए मूि अधधकार क्षेत्र रखता है जैसे: केंद्र और arising between various units of the Indian Union
such as: the Center and one or more states; or

-:: 29 ::-
RPT 05
एक र्ा अधधक राज्र्; र्ा एक तरि केंद्र और कोई राज्र् र्ा राज्र् the Center on one side and any State or States
और दूसरी तरि एक र्ा अधधक राज्र्। and one or more States on the other side.
- दो र्ा दो से अधधक राज्र्ों के बीच उपरोि संर्ीर् वििादों में, सिोच्च - In the above federal disputes between two or
more states, the Supreme Court has exclusive
न्र्ार्ािर् के पास अनन्र् मूि अधधकार क्षेत्र है।
original jurisdiction.
53. भारत के सिोच्च न्त्यायािय में न्त्यायाधीशों की संख्या बढाने की 53. The Supreme Court of India is vested with the
शलक्त वनवहत है - power to increase the number of judges -
(a) भारत के राष्ट्रपवत (a) President of India
(b) Parliament
(b) संसद
(c) Chief Justice of India
(c) भारत के मुख्र् न्र्ार्ाधीश
(d) Law Commission
(d) विधध आर्ोग
[b]
[b] Explanation -
व्याख्या – - The Parliament of India has the power to make
- भारत की संसद के पास कानून बनाने, अधधकार क्षेत्र को व्यिस्थित laws, settle jurisdiction and modify the power of
करने और सिोच्च न्र्ार्ािर् की शलि को संहवधान सम्मत the Supreme Court According to the
संशोधधत करने की शलि है। सिोच्च न्र्ार्ािर् में न्र्ार्ाधीशों की Constitution. The number of judges in the
संख्र्ा संसद द्वारा बढाई र्ा र्िाई जा सकती है। Supreme Court can be increased or decreased by
the Parliament.
54. एक संिैधावनक सरकार के सन्दभु में वनम्नलिखित कथनों पर 54. With reference to a constitutional
विचार करें- government, consider the following
I. र्ि राज्र् प्राधधकरर् के वहत में व्यलिगत थितंत्रता पर प्रभािी statements -
I. It imposes effective restrictions on Individual
प्रवतबंध िगाती है।
liberty in the interest of state authority.
II. र्ि व्यलिगत थितंत्रता के वहत में राज्र् की सत्ता पर प्रभािी
II. It imposes effective restrictions on the
प्रवतबंध िगाती है।
Authority of the state in the interest of
उपययुक्त में से कौनसा/से कथन सत्य है/हैं? individual liberty
(a) केिि I Which of the above statement(s) is/are true?
(b) केिि II (a) I only
(c) I और II दोनों (b) II only
(d) न तो I, न ही II (c) Both I and II
[b] (d) Neither I nor II
व्याख्या – [b]
Explanation -
- संिैधावनक सरकार व्यलिगत थितंत्रता के वहत में राज्र् के अधधकार
- Constitutional government imposes effective
पर प्रभािी प्रवतबंध िगाती है। नागररकों को अपनी इच्छानुसार
restrictions on the authority of the state in the
समाज में रहने और काम करने की थितंत्रता है। हािांवक, नागररकों
interest of individual liberty. Citizens have the
को समाज के समग्र कार्ों की बेहतरी के लिए पािन करने के लिए freedom to live and work in the society as they
कुछ वनर्मों का पािन करना चाहिए। wish. However, there are certain rules to be
followed by the citizens for the betterment of the
overall functioning of the society.
55. वनम्नलिखित कथनों पर विचार कीजजए - 55. Consider the following statements -
I. एचएन सान्र्ाि सधमवत की ररपोिय के अनुसार, न्र्ार्ािर् की I. As per the HN Sanyal Committee report, the
अिमानना अधधवनर्म, 1971 पाररत वकर्ा गर्ा िा। Contempt of Courts Act, 1971 was passed.
II. The Constitution of India empowers the
II. भारत का संविधान सिोच्च न्र्ार्ािर् और उच्च न्र्ार्ािर्ों को
Supreme Court and the High Courts to
थिर्ं की अिमानना के लिए दं धडत करने का अधधकार दे ता है।
punish for contempt of themselves.
III. भारत का संविधान नागररक अिमानना और आपराधधक
III. The Constitution of India defines civil
अिमानना को पररभावषत करता है। contempt and criminal contempt.
IV. भारत में, संसद को न्र्ार्ािर् की अिमानना पर कानून बनाने IV. In India, Parliament is vested with the
की शलिर्ााँ प्राप्त हैं। powers to legislate on contempt of court.
उपययुक्त में से कौनसा/से कथन सत्य है/हैं?
-:: 30 ::-
RPT 05
(a) केिि I और II Which of the above statement(s) is/are true?
(b) केिि I, II और IV (a) I and II only
(c) केिि III और IV (b) I, II and IV only
(c) III and IV only
(d) केिि III
(d) III only
[b]
[b]
व्याख्या –
Explanation -
न्त्यायािय की अिमानना - Contempt of Court -
- र्ह न्र्ार्ािर् की शलि है वक िह अपने गौरि और सम्मान की रक्षा - It is the power of the court to protect its pride and
करे। न्र्ार्ािर् की अिमानना अधधवनर्म, 1971 में इस शलि को honour. The power is regulated but not
विवनर्धमत वकर्ा जाता है िेवकन प्रवतबंधधत नहीं। एच. एन. सान्र्ाि prohibited in the Contempt of Courts Act, 1971
सधमवत की ररपोिय के अनुसरर् में, न्र्ार्ािर् की अिमानना Pursuant to the HN Sanyal Committee report, the
अधधवनर्म, 1971 पाररत वकर्ा गर्ा िा। Contempt of Courts Act, 1971 was passed.
- भारत का संविधान सिोच्च न्र्ार्ािर् और उच्च न्र्ार्ािर्ों को - The Constitution of India empowers the Supreme
Court and the High Courts to punish for
अपनी अिमानना के लिए दं ड दे ने हेतु, शलि प्रदान करता है तिा
contempt of themselves and the High Courts
उच्च न्र्ार्ािर्ों को अपने संबंधधत क्षेत्राधधकार के तहत् वनचिी
have the power to punish for contempt of lower
अदाितों के लिए अिमानना के मामालें में दं धडत करने की शलि है।
courts under their respective jurisdictions.
- भारत का संविधान वकसी भी प्रकार की अदाित की अिमानना, न - The Constitution of India defines contempt of
तो लसविि अिमानना और ना ही आपराधधक अिमानना को court of any kind, neither civil contempt nor
पररभावषत करता है। criminal contempt.
56. राज्य सभा को िोकसभा के बराबर शलक्तयााँ प्राप्त हैं - 56. Rajya Sabha has the same powers as the Lok
(a) नई अखखि भारतीर् सेिाएं बनाने िेतय Sabha -
(b) संविधान में संशोधन िेतय (a) For the issue of creating new All India
Services
(c) सरकार को हिाने िेतय
(b) For amendment of the constitution
(d) किौती प्रथताि प्रथतुत करने के विषर् में
(c) For removal of the government
[b]
(d) Regarding the presentation of cut motion
व्याख्या – [b]
वनम्नलिखित स्स्थवतयों में, राज्य सभा की शलक्तयााँ और स्स्थवत Explanation -
िोकसभा के बराबर होती हैं - In the following situations, the powers and
- साधारर् विधेर्क को िाना और पाररत कराना। status of the Rajya Sabha are equal to that of
- संविधान संशोधन विधेर्कों का पररचर् और पाररत कराना। the Lok Sabha -
- राष्ट्रपवत का चुनाि और महाणभर्ोग। - Introduction and passage of ordinary bills.
- उपराष्ट्रपवत का चुनाि और मिाभभर्ोग। हािााँवक, राज्र्सभा अकेिे - Introduction and passing of constitutional
amendment bills.
उपराष्ट्रपवत को हिाने की पहि कर सकती है। उन्हें राज्र्सभा द्वारा
- Election and impeachment of the President.
विशेष बहुमत से पाररत एक प्रथताि द्वारा हिार्ा जाता है और
- Election and removal of the Vice President.
िोकसभा द्वारा साधारर् बहुमत से सहमवत व्यि की जाती है।
However, the Rajya Sabha alone can initiate the
- सिोच्च न्र्ार्ािर् और उच्च न्र्ार्ािर्ों के मुख्र् न्र्ार्ाधीश और removal of the Vice President. He is removed by
न्र्ार्ाधीशों, मुख्र् चुनाि आर्ुि और वनर्ंत्रक और महािेखा a resolution passed by the Rajya Sabha with a
परीक्षक को हिाने के लिए राष्ट्रपवत को लसिाररशें करना। special majority and agreed upon by the Lok
- राष्ट्रपवत द्वारा जारी अध्र्ादे शों का अनुमोदन। Sabha with a simple majority.
- राष्ट्रपवत द्वारा तीनों प्रकार की आपात स्थिवतर्ों की उद्घोषर्ा की - Making recommendations to the President for
थिीकृवत। the removal of the Chief Justice and judges of the
- उच्चतम न्र्ार्ािर् और संर् िोक सेिा आर्ोग के क्षेत्राधधकार का Supreme Court and High Courts, the Chief
Election Commissioner and the Comptroller and
विथतार।
Auditor General.
अपने संघीय स्िरूप के कारण, राज्य सभा को दो विलशष्ट या
- Approval of ordinances issued by the President.
विशेष शलक्तयााँ दी गई हैं जो िोकसभा को प्राप्त नहीं हैं -
- Approval of the proclamation of all three types of
- र्ह संसद को राज्र् सूची में उस्ल्िखखत विषर् पर कानून बनाने के emergency by the President.
लिए अधधकृत कर सकती है। (अनुच्छे द 249)
-:: 31 ::-
RPT 05
- र्ह केंद्र और राज्र्ों दोनों के लिए नई अखखि भारतीर् सेिाओं को - Extension of jurisdiction of the Supreme Court
बनाने के लिए संसद को अधधकृत कर सकती है। (अनुच्छे द 312) and the Union Public Service Commission.
Due to its federal nature, the Rajya Sabha has
been given two specific or special powers
which are not available to the Lok Sabha -
- It can authorize the Parliament to make laws on
the subject mentioned in the State List (Article
249).
- It can authorize the Parliament to create new All
India Services for both Center and States (Article
312)
57. वनम्नलिखित कथनों पर विचार करें - 57. Consider the following statements -
I. भारत का संविधान मंवत्रर्ों को चार श्रेभणर्ों में िगीकृत करता I. The Constitution of India classifies the
है। कैवबनेि मंत्री, थितंत्र प्रभार िािे राज्र् मंत्री, राज्र्मंत्री और ministers into four ranks. Cabinet Ministers,
Ministers of State with Independent Charge,
उप मंत्री।
Ministers of State and Deputy Ministers.
II. प्रधानमंत्री सवहत केंद्र सरकार में मंवत्रर्ों की कुि संख्र्ा
II. The total number of ministers in the Central
िोकसभा में सदथर्ों की कुि संख्र्ा के 15 प्रवतशत से अधधक
Government including the Prime Minister
नहीं होगी। shall not exceed 15 percent of the total
उपययुक्त में से कौनसा/से कथन सत्य है/हैं? number of members in the Lok Sabha.
(a) केिि I Which of the above statement(s) is/are true?
(b) केिि II (a) I only
(c) I और II दोनों (b) II only
(d) न तो I, न ही II (c) Both I and II
[b] (d) Neither I nor II
[b]
व्याख्या –
Explanation -
- हमारा संविधान मंवत्रर्ों को चार श्रेणर्र्ों, अिायत कैवबनेि मंत्री,
- Our constitution does not classify the ministers
थितंत्र प्रभार िािे राज्र् मंत्री, राज्र् मंत्री और उपमंत्री मे िगीकृत
into four categories, viz. cabinet ministers,
नहीं करता है। हम इंग्िैंड की परम्परा से इस िगीकरर् का पािन ministers of state with independent charge,
करते हैं िेवकन र्ह हमारे संविधान में कहीं पर भी उल्ललखखत नहीं ministers of state and deputy ministers. We
है अिायत अनुच्छे द 74 में केिि मंवत्रपररषद् का उल्िेख है। follow this classification from the tradition of
- 91वााँ संकवधान संशोधन 2003- मंवत्रपररषद् में प्रधानमंत्री सवहत England but it is not mentioned anywhere in our
मंवत्रर्ों की कुि संख्र्ा िोक सभा के सदथर्ों की कुि संख्र्ा के 15 constitution i.e. Article 74 only mentions the
प्रवतशत से अधधक नहीं होनी चावहए। Council of Ministers.
- अनुच्छे द 74 - प्रधानमंत्री के नेतृत्ि में एक मंवत्रपररषद् राष्ट्रपवत को - 91st Constitution Amendment 2003 - The total
उसके कत्तयव्यों के पािन में सहार्ता और सिाह दे गी। number of ministers including the Prime Minister
in the Council of Ministers should not exceed 15
percent of the total number of members of the
Lok Sabha.
- Article 74 - A Council of Ministers headed by the
Prime Minister shall aid and advise the President
in the performance of his duties.
58. भारतीय संविधान में केंद्र और राज्यों के बीच शलक्तयों का 58. The distribution of powers between the
वितरण ___________ में प्रदान की गई योजना पर आधाररत है। Center and the States in the Indian
(a) मॉिे-ममिंिो सुधार, 1909 Constitution is based on the scheme provided
in ___________ ।
(b) मोंिग्र्ू-चेम्सिोडय अधधवनर्म, 1919
(a) Morley-Minto Reforms, 1909
(c) भारत सरकार अधधवनर्म, 1935
(b) Montagu-Chelmsford Act, 1919
(d) भारतीर् थितंत्रता अधधवनर्म, 1947
(c) Government of India Act, 1935
[c] (d) Indian Independence Act, 1947
व्याख्या – [c]
-:: 32 ::-
RPT 05
- भारतीर् संविधान में केंद्र और राज्र्ों के बीच शलिर्ों का वितरर् Explanation -
भारत सरकार अधधवनर्म, 1935 में प्रदान की गई र्ोजना पर - The distribution of powers between the Center
आधाररत है। and the States in the Indian Constitution is based
on the scheme provided in the Government of
India Act, 1935.
59. भारत में दिबदि विरोधी कानून के संदभथ में वनम्नलिखित 59. With reference to the Anti-defection law in
कथनों पर विचार कीजजए - India, consider the following statements -
I. कानून वनर्दिंि करता है वक एक मनोनीत विधार्क सदन में I. The law specifies that a nominated legislator
cannot join any political party within six
वनर्ुि होने के छह महीने के भीतर वकसी भी राजनीवतक दि
months of being appointed to the House.
में शाधमि नहीं हो सकता है।
II. The law does not provide for any time limit
II. कानून कोई समर्-सीमा प्रदान नहीं करता है जजसके भीतर
within which the Presiding Officer has to
पीठासीन अधधकारी को दि-बदि मामिे का िैसिा करना decide the defection case.
होता है। Which of the statements given above is/are
उपयुथक्त कथनों में से कौन सा/से सही है/हैं? correct?
(a) केिि I (a) I only
(b) केिि II (b) II only
(c) I और II दोनों (c) Both I and II
(d) न तो I, न ही II (d) Neither I nor II
[b]
[b]
Explanation -
व्याख्या –
- The Tenth Schedule contains the following
- दसिीं अनुसूची में दिबदि के आधार पर संसद और राज्र्
provisions regarding disqualification of
विधानसभाओं के सदथर्ों की अर्ोग्र्ता के संबंध में वनम्नलिखखत members of Parliament and State Legislatures on
प्रािधान शाधमि हैं- सदन का एक मनोनीत सदथर् सदन के सदथर् the ground of defection - Joins any political party
होने के लिए अर्ोग्र् हो जाता है र्टद िह सदन में अपना थिान ग्रहर् after the expiry of six months.
करने की तारीख से छह महीने की समाप्प्त के बाद वकसी राजनीवतक - Any question regarding disqualification effected
दि में शाधमि होता है। by defection shall be decided by the Presiding
- दिबदि से प्रभावित होने िािी अर्ोग्र्ता के संबंध में वकसी भी Officer of the House.
प्रश्न का वनर्यर् सदन के पीठासीन अधधकारी द्वारा वकर्ा जाएगा। - As per the law, there is no time limit for the
presiding officers to consider the petition of
- कानून के अनुसार, पीठासीन अधधकाररर्ों के पास अर्ोग्र्ता की
disqualification.
र्ाधचका पर विचार करने के लिए कोई समर् सीमा नहीं है।
60. वित्तमंरी संसद में बजट प्रस्तुत करते हुए उसके साथ अन्त्य 60. While presenting the budget in the
प्रिेि भी प्रस्तुत करते हैं जजनमें 'वृहद आर्थिक रूपरेिा Parliament, the Finance Minister presents
वििरण (The Macro Economic Framework other documents along with it, in which 'The
Macro Economic Framework Statement' is
Statement)' भी सम्म्मलित रहता है । पूिोक्त यह प्रिेि
also included. This aforesaid document is
वनम्न आदे शन के कारण प्रस्तुत वकया जाता है -
produced on account of the following order -
(a) िंबे समर् से चिी आ रही संसदीर् परंपरा।
(a) Long standing parliamentary tradition.
(b) भारत के संविधान के अनुच्छे द 112 और अनुच्छे द 110(1) (b) Because of Article 112 and Article 110(1) of
के कारर्। the Constitution of India.
(c) भारत के संविधान के अनुच्छे द 113 के कारर्। (c) Because of Article 113 of the Constitution of
(d) राजकोषीर् उत्तरदावर्त्ि और बजि प्रबंधन अधधवनर्म, 2003 India.
के प्रािधान के कारर्। (d) Due to the provision of Fiscal Responsibility
[d] and Budget Management Act, 2003.
व्याख्या – [d]
Explanation -
- िृहद-आर्ििंक ढााँचा वििरर् एक बजि है जजसे केंद्रीर् बजि के
- The Macro-Economic Framework Statement is a
समर् राज वित्तीर् उत्तरदावर्त्ि और बजि प्रबंध अधधवनर्म, 2003
budget that is presented under section 3(5) of
की धारा 3 (5) के तहत् प्रथतुत वकर्ा जाता है।
the Fiscal Responsibility and Budget

-:: 33 ::-
RPT 05
राजकोषीय उत्तरदाधयत्ि और बजट प्रबंधन अधधवनयम, 2003 Management Act, 2003 at the time of the Union
द्वारा अवनिायथ दस्तािेज - Budget.
- िृहद-आर्ििंक ढााँचा वििरर् Documents Mandated by the Fiscal
Responsibility and Budget Management Act,
- राजकोषीर् नीवत रर्नीवत वििरर्
2003 -
- मध्र्म अिधध की राजकोषीर् नीवत वििरर्
- Macro-economic Framework Statement
- मध्र्म अिधध व्यर् रूपरेखा वििरर्
- Fiscal Policy Strategy Statement
- Medium Term Fiscal Policy Statement
- Medium Term Expenditure frame work Details
61. िोकसभा के उपाध्यि के संदभथ में वनम्नलिखित कथनों पर 61. With reference to the Deputy Speaker of the
विचार कीजजए - Lok Sabha, consider the following statements-
I. िोकसभा की प्रविर्ा और कार्य-संचािन वनर्मों के अनुसार, I. In accordance with the Rules of Procedure
and Conduct of Business in the Lok Sabha,
उपाध्र्क्ष का चुनाि उस वतलि को होगा, जजसे अध्र्क्ष वनर्त
the election of the Deputy Speaker shall be
कर सकता है।
held on such date as the Speaker may fix.
II. एक अवनिार्य प्रािधान है वक िोकसभा के उपाध्र्क्ष के रूप में
II. There is a mandatory provision that the
उम्मीदिार का चुनाि र्ा तो मुख्र् विपक्षी पािी र्ा सत्ताधारी candidate to be elected as the Deputy
पािी से होगा। Speaker of the Lok Sabha shall be from either
III. सदन की बैठक की अध्र्क्षता करते समर् उपाध्र्क्ष के पास the main opposition party or the ruling party.
अध्र्क्ष के समान शलि होती है और उनके िैसिों के खखिाि III. While presiding over the meeting of the
कोई अपीि नहीं होती है। House, the Deputy Speaker has the same
IV. उपाध्र्क्ष की वनर्ुलि के संबंध में अच्छी तरह से थिावपत power as the Speaker and there is no appeal
संसदीर् प्रिा र्ह है वक प्रथताि अध्र्क्ष द्वारा पेश वकर्ा जाता against his decisions.
IV. The well-established parliamentary practice
है और प्रधानमंत्री द्वारा विधधित अनुमोटदत वकर्ा जाता है।
with regard to the appointment of the
उपययुक्त में से कौनसा/से कथन सत्य है/हैं?
Deputy Speaker is that the motion is moved
(a) केिि I और III
by the Speaker and duly seconded by the
(b) केिि I, II और III Prime Minister.
(c) केिि III और IV Which of the above statement(s) is/are true?
(d) केिि II और IV (a) I and III only
[a] (b) I, II and III
व्याख्या – (c) III and IV only
िोकसभा के उपाध्यि- (d) II and IV only
- अध्र्क्ष के समान, उपाध्र्क्ष भी िोकसभा द्वारा अपने सदथर्ों में से [a]
Explanation -
ही चुना जाता है। अध्र्क्ष का चुनाि होने के बाद उसका चुनाि वकर्ा
Deputy Speaker of the Lok Sabha
जाता है। उपाध्र्क्ष के चुनाि की वतलि अध्र्क्ष द्वारा वनधायररत की
- Like the Speaker, the Deputy Speaker is also
जाती है।
elected by the Lok Sabha from among its
- जब भी उपाध्र्क्ष का पद ररि होता है, िोकसभा ररलि को भरने members. He is elected after the election of the
के लिए वकसी अन्र् सदथर् का चुनाि करती है। Speaker. The date for the election of the Vice
- 10िीं िोकसभा तक, अध्र्क्ष और उपाध्र्क्ष दोनों आमतौर पर President is fixed by the President.
सत्ताधारी दि से होते िे। 11िीं िोकसभा के बाद से र्ह आम - Whenever the office of the Deputy Speaker
सहमवत रही है वक अध्र्क्ष सत्ताधारी दि (र्ा सत्तारूढ गठबंधन) से becomes vacant, the Lok Sabha elects another
आता है और उपाध्र्क्ष का पद मुख्र् विपक्षी दि को जाता है। member to fill the vacancy.
- उपाध्र्क्ष अध्र्क्ष के पद के ररि होने पर अपने कत्तयव्यों का वनियहन - Until the 10th Lok Sabha, both the Speaker and
the Deputy Speaker were usually from the ruling
करता है। उपाध्र्क्ष, अध्र्क्ष के रूप में भी कार्य करता है जब अध्र्क्ष
party. Since the 11th Lok Sabha, it has been a
सदन की बैठक से अनुपस्थित रहता है। दोनों ही मामिों में, िह
consensus that the Speaker comes from the
अध्र्क्ष की सभी शलिर्ों को ग्रहर् करता है।
ruling party (or ruling coalition) and the post of
- उपाध्र्क्ष के पास सदन की बैठक की अध्र्क्षता करते समर् अध्र्क्ष Deputy Speaker goes to the main opposition
के समान शलि होती है और उसके िैसिों के खखिाि कोई अपीि party.
नहीं होती है।
-:: 34 ::-
RPT 05
- The Vice President discharges his duties when
the post of the President is vacant. The Deputy
Speaker also acts as Speaker when the Speaker is
absent from the meetings of the House. In both
the cases, he assumes all the powers of the
Speaker.
- The Deputy Speaker has the same power as the
Speaker while presiding over the meeting of the
House and there is no appeal against his
decisions.
62. भारतीय संविधान के तहत् धन का संकेंद्रण उल्िंघन करता है- 62. Under the Indian Constitution, the
(a) समानता का अधधकार का concentration of wealth violates-
(b) राज्र् नीवत के वनदे शक लसद्धांत का (a) Right to equality
(b) Directive Principles of State Policy
(c) थितंत्रता का अधधकार का
(c) Right to Freedom
(d) कल्र्ार् की अिधारर्ा का
(d) Concept of welfare
[b]
[b]
व्याख्या – Explanation -
- भारतीर् संविधान के वनदे शक लसद्धांतों के अनुच्छे द-39 (c) के - According to Article 39(c) of the Directive
अनुसार, आर्ििंक व्यिथिा के संचािन के पररर्ामथिरूप धन और Principles of the Indian Constitution, the
उत्पादन के साधनों का संकेंद्रर् नहीं होना चावहए। अतः धन का operation of the economic system should not
संकेंद्रर् राज्र् के नीवत वनदे शक तत्िों का उल्िंर्न करता है। result in concentration of wealth and means of
production to the general disadvantage.
Therefore, the concentration of wealth violates
the Directive Principles of State Policy.
63. 26 जनिरी, 1950 को भारत की िास्तविक संिैधावनक स्स्थवत 63. What was the real constitutional position of
क्या थी? India on January 26, 1950?
(a) एक िोकतांवत्रक गर्राज्र् (a) A democratic republic.
(b) A sovereign, democratic republic.
(b) एक संप्रभु, िोकतांवत्रक, गर्राज्र्
(c) A sovereign, secular, democratic republic.
(c) एक संप्रभु, धमयवनरपेक्ष, िोकतांवत्रक गर्राज्र्
(d) A sovereign, socialist, secular, democratic
(d) एक संप्रभु, समाजिादी, धमयवनरपेक्ष, िोकतांवत्रक गर्राज्र्
republic. [b]
[b] Explanation -
व्याख्या – - The Constitution of India came into force on 26
- भारत का संविधान 26 जनिरी, 1950 को िागू हुआ। 26 जनिरी, January 1950. On 26 January 1950, India had the
1950 को, भारत को 'संप्रभु, िोकतांवत्रक, गर्राज्र्' का constitutional status of 'Sovereign, Democratic,
संिैधावनक दजाय प्राप्त हुआ िा। 'संप्रभु, िोकतांवत्रक, गर्राज्र्' का Republic'. 'Sovereign, Democratic, Republic' It
उल्िेख भारत के संविधान की प्रथतािना में वकर्ा गर्ा िा। was mentioned in the Preamble of the
Constitution of India.
64. भारत के संविधान में अंतरराष्ट्रीय शांवत और सुरिा को बढािा 64. The promotion of international peace and
दे ना शाधमि है - security is included in the Constitution of
(a) संविधान की प्रथतािना में India -
(a) In preamble of the Constitution
(b) राज्र् नीवत के वनदे शक लसद्धांत (DPSP) में
(b) In directive Principles of State Policy (DPSP)
(c) मौलिक कत्तयव्य में
(c) In fundamental Duties
(d) नौिीं अनुसूची में
(d) In ninth Schedule
[b] [b]
व्याख्या – Explanation -
- भारत के संविधान में, अंतरराष्ट्रीर् शांवत और सुरक्षा को बढािा दे ना - In the Constitution of India, the promotion of
‘राज्र् नीवत के वनदे शक लसद्धांतों’ में वनवहत है। (अनयच्छे द-51) international peace and security is enshrined in
the ‘Directive Principles of State Policy’. (Article-
51)
-:: 35 ::-
RPT 05
65. भारत के संदभथ में वनम्नलिखित कथनों पर विचार कीजजए- 65. With reference to India, consider the
I. न्र्ावर्क वहरासत का मतिब है वक आरोपी संबंधधत मजजथरे ि following statements-
की वहरासत में है और ऐसा आरोपी जेि में नहीं, िाने में बंद I. Judicial Custody means that the accused is in
the custody of the concerned Magistrate and
है।
such an accused is locked in the police
II. न्र्ावर्क वहरासत के दौरान मामिे के प्रभारी पुलिस अधधकारी
station and not in jail.
को अदाित की मंजूरी के वबना संटदग्ध से पूछताछ करने की
II. During judicial custody, the police officer in
अनुमवत नहीं है। charge of the case is not allowed to
उपयुथक्त कथनों में से कौन सा/से सही है/हैं? interrogate the suspect without the approval
(a) केिि I of the court.
(b) केिि II Which of the statements given above is/are
(c) I और II दोनों correct?
(d) न तो I, न ही II (a) I only
[b] (b) II only
(c) Both I and II
व्याख्या –
(d) Neither I nor II
पुलिस वहरासत और न्त्याधयक वहरासत
[b]
- पुलिस वहरासत का मतिब है वक पुलिस के पास आरोपी की
Explanation -
शारीररक वहरासत है जबवक न्र्ावर्क वहरासत का मतिब है वक Police Custody and Judicial Custody
आरोपी संबंधधत मजजथरे ि की वहरासत में है। पुलिस वहरासत में - Police custody means that the police have
आरोपी को िाने के िॉकअप में रखा जाता है जबवक न्र्ावर्क physical custody of the accused while judicial
वहरासत जेि में होती है। custody means that the accused is in the custody
- न्र्ावर्क वहरासत के दौरान, मामिे के प्रभारी पुलिस अधधकारी को of the concerned magistrate. In police custody,
संटदग्ध से पूछताछ करने की अनुमवत नहीं होती है। हािांवक, the accused is kept in the lockup of the police
अदाित के सामने पेश वकए गए तथ्र्ों के तहत् पूछताछ आिश्र्क station, while in judicial custody, it is a jail.
- During judicial custody, the police officer in
होने पर अदाित पूछताछ करने की अनुमवत दे सकती है।
charge of the case is not allowed to interrogate
the suspect. However, the court may allow the
inquiry to be conducted if the inquiry is necessary
under the facts presented before the court.
66. सरकार की संसदीय प्रणािी िह है जजसमें - 66. A parliamentary system of government is in
(a) संसद में सभी राजनीवतक दिों को सरकार में प्रवतवनधधत्ि ददर्ा which -
जाता है। (a) All political parties are represented in the
government in the Parliament.
(b) सरकार संसद के प्रवत उत्तरदार्ी है और उसके द्वारा हिार्ा जा
(b) The government is responsible to the
सकता है।
Parliament and can be removed by it.
(c) सरकार िोगों द्वारा चुनी जाती है और उनके द्वारा हिार्ा जा
(c) The government is elected by the people and
सकता है। can be removed by them.
(d) सरकार संसद द्वारा चुनी जाती है िेवकन वनणित अिधध पूरी (d) The government is elected by the parliament
होने से पहिे सरकार को हिार्ा नहीं जा सकता है। but cannot be removed before the
[b] completion of a fixed term.
व्याख्या – [b]
- संसदीर् प्रर्ािी, सरकार का िोकतांवत्रक रूप है जजसमें संसद Explanation -
(विधावर्का) में सबसे बड़े प्रवतवनधधत्ि िािी पािी (र्ा पार्ििंर्ों का - A parliamentary system is a democratic form of
government in which the party (or coalition of
गठबंधन) सरकार बनाती है, उसका नेता प्रधानमंत्री बनता है।
parties) with the largest representation in the
- प्रधानमंत्री द्वारा कैवबनेि के लिए वनर्ुि संसद के सदथर्ों द्वारा
parliament (legislature) forms the government,
कार्यकारी कार्ों का उपर्ोग वकर्ा जाता है। जब भी िे सत्ताधारी
becomes the Prime Minister.
पािी र्ा संसद के अधधकांश सदथर्ों का विश्वास खोते हैं, तो उन्हें - Executive functions are exercised by members of
सत्ता से हिार्ा जा सकता है। parliament appointed by the prime minister to
the cabinet. They can be removed from power

-:: 36 ::-
RPT 05
whenever they lose the confidence of the ruling
party or a majority of members of parliament.
67. भारतीय राजनीवत में वनम्नलिखित में से कौन-सी एक 67. Which one of the following is an essential
आिश्यक विशेषता है जो इंवगत करती है वक यह चररर में feature of the Indian polity which indicates
संघीय है? that it is federal in character?
(a) The independence of the judiciary is
(a) न्र्ार्पालिका की थितंत्रता सुरणक्षत है।
protected.
(b) संर् की विधावर्का में संर्िक इकाइर्ों के वनिायधचत प्रवतवनधध
(b) The legislature of the Union consists of the
होते है।
elected representatives of the constituent
(c) केंद्रीर् मंवत्रमंडि में क्षेत्रीर् दिों के वनिायधचत प्रवतवनधध हो units.
सकते हैं। (c) There can be elected representatives of
(d) मूि अधधकार न्र्ार्ािर्ों द्वारा प्रितयनीर् है। regional parties in the Union Cabinet.
[a] (d) Fundamental Rights are enforceable by the
व्याख्या – courts. [a]
- संविधान के मूि प्रािधानों को एक थतर की सरकार द्वारा एक तरिा Explanation -
नहीं बदिा जा सकता है। इस तरह के पररितयनों के लिए सरकार के - The basic provisions of the constitution cannot
be changed unilaterally by one level of
दोनों थतरों की सहमवत की आिश्र्कता होती है। न्र्ार्ािर्ों के पास
government. Such changes require consent from
संविधान और सरकार के विणभन्न थतरों की शलिर्ों की व्याख्र्ा
both levels of government. The courts have the
करने की शलि है। सिोच्च न्र्ार्ािर् मध्र्थिता के रूप में कार्य
power to interpret the constitution and the
करता है र्टद सरकार के विणभन्न थतरों के बीच उनकी संबंधधत powers of the various levels of government. The
शलिर्ों के प्रर्ोग में वििाद उत्पन्न होता है। Supreme Court acts as an arbitrator if a dispute
- भारतीर् संविधान द्वारा अपनाई गई संर्ीर् व्यिथिा की सबसे arises between the different levels of
महत्त्वपूणु विशेषता र्ह लसद्धांत है वक राज्र्ों और केंद्र के बीच संबंध government in the exercise of their respective
सहर्ोग पर आधाररत होंगे और इसके लिए थितंत्र न्र्ार्पालिका powers.
पहिी शतय है। - The most important feature of the federal system
adopted by the Constitution of India is the
principle that the relationship between the states
and the center will be based on cooperation and
for this an independent judiciary is the first
condition.
68. वनम्नलिखित में से कौन सा कथन ‘राज्य’ को सिोत्तम रूप से 68. Which of the following statements best
पररभावषत करता है? defines ‘State’?
(a) व्यलिर्ों का एक समुदार् थिार्ी रूप से बाहरी वनर्ंत्रर् से (a) A community of individuals permanently
occupying a definite territory independent of
थितंत्र एक वनणित क्षेत्र पर कब्जा कर रहा है और एक संगटठत
external control and having an organized
सरकार रखता है।
government.
(b) एक वनणित क्षेत्र के राजनीवतक रूप से संगटठत िोग और उन
(b) Politically organized people of a definite
पर शासन करने, कानून और व्यिथिा बनाए रखने, उनके territory and empowered to govern them,
प्राकृवतक अधधकारों की रक्षा करने और उनके जीविका के maintain law and order, protect their natural
साधनों की रक्षा करने का अधधकार रखते हैं। rights and protect their means of livelihood.
(c) बहुत से व्यलि जो अपनी संथकृवत, परंपरा और सरकार के (c) A number of persons who have been living in
साि बहुत िंबे समर् से एक वनणित क्षेत्र में रह रहे हैं। a certain area for a very long time with their
(d) एक केंद्रीर् प्राधधकरर् के साि थिार्ी रूप से रहने िािा एक own culture, tradition and government.
समाज, केंद्रीर् प्राधधकरर् के लिए जजम्मेदार एक कार्यकारी (d) A society living permanently with a central
authority, an executive responsible to the
और एक थितंत्र न्र्ार्पालिका।
central authority and an independent
[a]
judiciary.
व्याख्या –
[a]
- राज्य- व्यलिर्ों का एक समुदार् थिार्ी रूप से बाहरी वनर्ंत्रर् से Explanation -
थितंत्र एक वनणित क्षेत्र पर कब्जा कर रहा है और एक संगटठत - State - A community of individuals permanently
occupying a definite territory independent of
-:: 37 ::-
RPT 05
सरकार रखता है। र्ह किन राज्र् को सिोत्तम रूप से पररभावषत external control and having an organized
करता है। government. This statement best defines the
state.
69. भारत के संविधान की प्रस्तािना को सवोत्तम रूप से 69. The Preamble of the Constitution of India is
पररभाकषत ककया िा सकता है - the best defines as –
(a) संविधान का एक वहथसा है िेवकन इसका कोई कानूनी प्रभाि (a) A part of the constitution but has no legal
effect.
नहीं है।
(b) Not a part of the constitution and has no
(b) संविधान का वहथसा नहीं है और इसका कोई कानूनी प्रभाि भी
legal effect either.
नहीं है।
(c) Part of the Constitution and has the same
(c) संविधान का वहथसा और वकसी भी अन्र् वहथसे के समान legal effect as any other part.
कानूनी प्रभाि है। (d) A part of the constitution but has no legal
(d) संविधान का एक वहथसा िै िेवकन अन्र् भागों से थितंत्र रूप effect independently for other parts.
में इसका कोई कानूनी प्रभाि नहीं है। [d]
[d] Explanation -
व्याख्या – - In the Keshavananda Bharati case (1973), the
- केश्वानंद भारती केस (1973) में, उच्चतम न्र्ार्ािर् ने कहा वक Supreme Court held that the Preamble is part of
the Constitution. The Preamble sets out the aims
प्रथतािना संविधान का भाग है। प्रथतािना व्यलिर्ों के उद्दे श्र् और
and aspirations of the individuals and these are
आकांक्षाओं को वनधायररत करती है और इन्हें संविधान के विणभन्न
embodied in various provisions of the
प्रािधानों में सन्न्नवहत वकर्ा गर्ा है। र्ह थितंत्र रूप से कानूनन
Constitution. It does not have independent legal
प्रभािी नहीं है क्र्ोंवक इसे न्र्ार्ािर् द्वारा प्रिर्तिंत नहीं वकर्ा जा effect as it cannot be enforced by the court.
सकता है।
70. भारत के संदभथ में वनम्नलिखित कथनों पर विचार कीजजए- 70. With reference to India, consider the
I. केिि एक नागररकता और एक अधधिास है। following statements-
II. केवल जन्द्म से नागररक ही राज्र् का मुखखर्ा बन सकता है। I. There is only one citizenship and one
domicile.
III. वकसी विदे शी को एक बार नागररकता प्रदान करने के बाद उसे
II. Only a citizen by birth can become the head
वकसी भी पररस्थिवत में इससे िंधचत नहीं वकर्ा जा सकता है।
of state.
उपयुथक्त कथनों में से कौन सा/से सत्य है/हैं?
III. A foreigner once granted citizenship cannot
(a) केिि I be deprived of it under any circumstances.
(b) केिि II Which of the above statements is/are true?
(c) केिि I और III (a) I only
(d) केिि II और III (b) II only
[a] (c) I and III only
व्याख्या – (d) II and III only
- एक व्यलि के पास एक से अधधक अधधिास नहीं हो सकते हैं। भारत [a]
Explanation -
का संविधान भारतीर् नागररकता और वकसी विदे शी दे श की
- A person cannot have more than one domicile.
नागररकता एक साि रखने की अनुमवत नहीं दे ता है। भारत में, जन्म
The Constitution of India does not allow holding
से नागररक और साि ही नागररकता वनर्मान्तगयत नागररकता प्राप्त
Indian citizenship and citizenship of a foreign
नागररक दोनों राष्ट्रपवत के पद के लिए पात्र हैं। country together. In India, both a citizen by birth
- नागररकता अधधवनर्म, 1955, नागररकता खोने के तीन तरीकों को as well as an indigenous citizen are eligible for
वनधायररत करता है, चाहे िह अधधवनर्म के तहत् हालसि वकर्ा गर्ा the office of the President.
हो र्ा इससे पहिे संविधान र्ानी त्र्ाग, समाप्प्त और पृिक्करर् के - The Citizenship Act, 1955, prescribes three modes
तहत्। of losing citizenship, whether acquired under the
Act or earlier under the Constitution i.e.
renunciation, termination and deprivation.
71. कनम्नलिखित में से भारतीय संकवधान में ककस संकवधान 71. By which of the following Constitutional
संशोधन द्वारा अनयच्छे द 31(ग) को समाकहत ककया गया है? Amendment Article 31(c) has been
(a) 21वााँ संहवधान संशोधन अधधहनर्म incorporated in the Indian Constitution?
(a) 21st Constitutional Amendment Act
(b) 31वााँ संहवधान संशोधन अधधहनर्म
-:: 38 ::-
RPT 05
(c) 24 वााँ संहवधान संशोधन अधधहनर्म (b) 31st Constitutional Amendment Act
(d) 25 वााँ संहवधान संशोधन अधधहनर्म (c) 24th Constitutional Amendment Act
[d] (d) 25th Constitutional Amendment Act
[d]
व्याख्या –
Explanation -
25 वााँ संकवधान संशोधन अधधकनयम, 1971 -
25th Constitutional Amendment Act, 1971-
– अनुच्छे द 31C में दो प्रािधान शाधमि िे -
– Article 31C It contains two provisions -
– र्ह कहता है वक कोई भी कानून जजसमें अनुच्छे द 39 (B) और 39 – It says that no law seeking to implement the
(C) में विवनर्दिंि समाजिादी वनदे शक लसद्धांतों को िागू करने की Socialist Directive Principles specified in Article
मांग की गई है, अनुच्छे द 14 र्ा अनुच्छे द 19 द्वारा प्रदत्त मौलिक 39(B) and 39 (C) shall be declared invalid on the
अधधकारों के उल्िंर्न के आधार अमान्र् र्ोवषत नहीं होंगे। ground of being violative of the fundamental
– इसके अवतररि कोई भी कानून जो र्ह र्ोषर्ा करे वक र्ह ऐसी rights conferred by Article 14 or Article 19.
नीवत को प्रभावित करने हेतु है, जजसे वकसी भी न्र्ार्ािर् में इस – Further, any law which declares that it is intended
आधार पर चुनौती दी जा सकती है वक र्ह ऐसी नीवत को प्रभावित to affect such policy can be challenged in any
court on the ground that it does not affect such
नहीं करता है।
policy.
72. वनम्नलिखित में से कौन सा कारक उदार िोकतंर में स्ितंरता 72. Which of the following factors constitutes the
की सिोत्तम सुरिा प्रदान करता है? best safeguard of liberty in a liberal
(a) एक प्रवतबद्ध न्र्ार्पालिका democracy?
(a) A committed judiciary
(b) शलिर्ों का केंद्रीकरर्
(b) Centralization of powers
(c) वनिायधचत सरकार
(c) Elected government
(d) शलिर्ों का पृिक्करर्
(d) Separation of powers
[d] [d]
व्याख्या – Explanation -
शलक्तयों का पृथक्करण - Separation of Powers
- र्ह सरकार के विधार्ी, कार्यकारी और न्र्ावर्क कार्ों के बीच - It is the division of powers between the
शक्तक्तर्ों का विभाजन है। र्ह सरकार द्वारा मनमानी ज्र्ादवतर्ों की legislative, executive and judicial functions of the
संभािना को कम करता है क्र्ोंवक कानून बनाने, विर्ाप्न्ित करने government. It minimizes the possibility of
और प्रशासन के लिए तीनों शाखाओं की मंजूरी आिश्र्क है। arbitrary excesses by the government as the
approval of all the three branches is necessary
संिैधावनक सीमांकन सरकार की वकसी भी शाखा द्वारा अत्र्धधक
for law making, execution and administration.
शलि की संकेंद्रर् को रोकता है।
Constitutional limits prevent the concentration of
excessive power by any branch of government.
73. वनम्नलिखित में से कौनसी िोक सभा की अनन्त्य शलक्तयााँ 73. Which of the following is/are the exclusive
हैं/हैं? powers of the Lok Sabha?
I. आपातकाि की र्ोषर्ा की पुवि करना I. Ratification of Proclamation of Emergency.
II. Passing a motion of no confidence against
II. मंवत्रपररषद् के विरुद्ध अविश्वास प्रथताि पाररत करना
the Council of Ministers.
III. भारत के राष्ट्रपवत पर महाणभर्ोग िगाना
III. To Impeach the President of India.
सही कूट
Correct Code
(a) केिि I और II (a) I and II only
(b) केिि II (b) II only
(c) केिि I और III (c) I and III only
(d) केिि III (d) III only
[b] [b]
व्याख्या – Explanation -
- संविधान का अनुच्छे द-352 राष्ट्रीर् आपातकाि की र्ोषर्ा का - Article-352 of the constitution provides the
declaration of national emergency. Within one
उपबंध करता है। इसके जारी होने के एक महीने के भीतर,
month of its issue, the Proclamation of
आपातकाि की र्ोषर्ा को संसद के दोनों सदनों द्वारा अनुमोटदत
Emergency must be approved by both the
वकर्ा जाना चावहए।
Houses of Parliament.
-:: 39 ::-
RPT 05
- मंवत्रपररषद् सामूवहक रूप से िोकसभा के प्रवत उत्तरदार्ी होती है। - The Council of Ministers is collectively
अविश्वास प्रथताि के लिए वनम्नलिखखत अवनिार्यता हैं - responsible to the Lok Sabha. The following are
- अविश्वास प्रथताि केिि िोकसभा में पेश वकर्ा जा सकता है। इसे the essentials for a motion of no confidence.
- No-confidence motion can be introduced only in
राज्र्सभा में पेश नहीं वकर्ा जा सकता है।
the Lok Sabha. It cannot be introduced in the
- राष्ट्रपवत पर महाणभर्ोग की प्रविर्ा भारतीर् संविधान के अनुच्छे द-
Rajya Sabha.
61 में उस्ल्िखखत है। संविधान के उल्िंर्न के लिए राष्ट्रपवत के
- The process of impeachment of the President is
खखिाि महाणभर्ोग का आरोप संसद के वकसी भी सदन द्वारा िार्ा mentioned in Article-61 of the Indian
जा सकता है। Constitution. Impeachment charges against the
President for violation of the Constitution can be
brought by any House of Parliament.
74. 'वनजता का अधधकार' भारत के संविधान के वकस अनुच्छे द 74. The 'right to privacy' is protected under which
के अंतगथत संरक्षित है? article of the Constitution of India?
(a) अनुच्छे द 15 (a) Article 15
(b) Article 19
(b) अनुच्छे द 19
(c) Article 21
(c) अनुच्छे द 21
(d) Article 29
(d) अनुच्छे द 29
[c]
[c] Explanation -
व्याख्या – - Article 21- The right to privacy is protected under
- अनुच्छे द 21- वनजता का अधधकार अनुच्छे द 21 के तहत् जीिन Article 21 as an intrinsic part of the right to life
जीने के अधधकार और व्यलिगत थितंत्रता के आंतररक वहथसे के and personal liberty and as a part of the
रूप में और संविधान के भाग तृतीर् द्वारा गारंिीकृत थितंत्रता के freedoms guaranteed by Part III of the
एक वहथसे के रूप में संरणक्षत है। Constitution.
75. कनम्नलिखित कवकल्पों पर कवचार कीजिए – 75. Consider the following options –
I. कृष्ण स्वामी अय्र्र I. Krishna swami Iyer
II. एन. माधवराव II. N. Madhavrao
III. Gopal swami Iyengar
III. गोपाल स्वामी आर्ंगर
IV. H. C. Mukherjee
IV. एच. सी. मयखजी
V. J. B. Kriplani
V. जे. बी. कृपलानी
Who among the above was not a member of
उपययुक्त में से कौन ‘प्रारूप सधमकत’ के सदस्य नहीं थे? the 'Drafting Committee'?
(a) केवल II, IV और V (a) II, IV and V only
(b) केवल I, III और IV (b) I, III and IV only
(c) केवल IV और V (c) IV and V only
(d) केवल III, IV और V [c] (d) III, IV and V only [c]
व्याख्या – Explanation -
प्रारूप सधमकत- Drafting Committee-
– Formation – August 29, 1947
– गठन – 29 अगस्त, 1947
Total members – (1+6) = 7
कयल सदस्र् – (1+6) = 7
– Dr. B. R. Ambedkar (Chairman)
– डॉ. बी. आर. अंबेडकर (अध्र्क्ष)
– N. Gopal Swami Iyengar
– एन. गोपाल स्वामी आर्ंगर – Alladi Krishna Swami Iyer
– अललादी कृष्ण स्वामी अय्र्र – Dr. K. M Munshi
– डॉ. के. एम. मयंशी – Mohammad Sadullah
– मोिम्मद सादुलला – N. Madhavrao (in place of B.L. Mitra)
– एन. माधवराव (बी. एल. धमत्र की जगि) – T.T. Krishnamachari (in place of D.P. Khaitan)
– टी. टी. कृष्णामाचारी (डी.पी. खेतान की जगि)
76. अस्पृश्यता के खििाफ भेदभाि के रूप में शाधमि 'मौलिक 76. Which one of the following categories of
अधधकारों' की वनम्नलिखित श्रेक्षणयों में से कौन सी एक है? 'Fundamental Rights' is included in the form
(a) शोषर् के खखिाि अधधकार of discrimination against untouchability?

-:: 40 ::-
RPT 05
(b) थितंत्रता का अधधकार (a) Right against exploitation
(c) संिैधावनक उपचार का अधधकार (b) Right to freedom
(d) समानता का अधधकार (c) Right to Constitutional Remedies
(d) Right to equality
[d]
[d]
व्याख्या –
Explanation -
- भारतीर् संविधान के अनुच्छे द 14 -18 के तहत् समानता का
- Right to equality is given under Articles 14-18 of
अधधकार टदर्ा गर्ा है। अनुच्छे द 15 धमय, जावत, लििंग र्ा जन्म थिान the Indian Constitution. Article 15 deals with the
के आधार पर भेदभाि के वनषेध से संबंधधत है। prohibition of discrimination on grounds of
- अनुच्छे द 16 साियजवनक रोजगार के मामिों में अिसर की समानता religion, race, caste, sex or place of birth.
की गारंिी दे ता है। अनुच्छे द 17 अथपृश्र्ता को समाप्त करता है। - Article 16 guarantees equality of opportunity in
matters of public employment. Article 17
abolishes untouchability.
77. भारत में न्त्यायपालिका को कायथपालिका से अिग करने का 77. By whom is the order to separate the Judiciary
आदे श वकसके द्वारा ददया जाता है? from the Executive in India?
(a) संविधान की प्रथतािना (a) Preamble of the Constitution
(b) A Directive Principle of State Policy
(b) राज्र् नीवत वनदे शक तत्त्व
(c) Seventh Schedule
(c) सातिीं अनुसूची
(d) Traditional Practice
(d) पारंपररक अभ्र्ास
[b]
[b] Explanation -
व्याख्या – - Article 36 to Article 51 of our Constitution deals
- हमारे संविधान का अनुच्छे द 36 से अनुच्छे द 51 राज्र् नीवत के with the Directive Principles of State Policy.
वनदे शक लसद्धांतों से संबंधधत है। - Article 50 determines the separation of the
- अनुच्छे द 50 कार्यपालिका से न्र्ार्पालिका के पृिक्करर् को judiciary from the executive. The Directive
वनधायररत करता है। राज्र् नीवत के वनदे शक लसद्धांत न्र्ार्ािर् द्वारा Principles of State Policy are not enforceable by
प्रितयनीर् नहीं हैं। the courts.
78. मौलिक अधधकारों के अिािा, भारत के संविधान में 78. Apart from the Fundamental Rights, which of
वनम्नलिखित में से कौन सा भाग मानि अधधकारों की the following parts of the Constitution of
सािथभौम घोषणा (1948) के लसद्ांतों और प्रािधानों को India reflects/reflects the principles and
provisions of the Universal Declaration of
दशाथता/प्रवतबबिवबत करता है?
Human Rights (1948)?
I. प्रथतािना
I. Preamble
II. राज्र् के नीवत वनदे शक तत्त्व
II. Directive Principles of State Policy
III. मौलिक कत्तयव्य III. Fundamental duty
सही कूट - Correct Code –
(a) केिि I और II (a) I and II only
(b) केिि II (b) II only
(c) केिि I और III (c) I and III only
(d) I, II और III (d) I, II and III
[d] [d]
Explanation -
व्याख्या –
- Universal Declaration of Human Rights has been
- पेररस में संर्ुि राष्ट्र के संकल्प 217 A के तहत् मानि अधधकारों
made in Paris under United Nations Resolution
की साियभौधमक र्ोषर्ा की गई है।
217A.
- भारत की प्रथतािना में “व्यलि की गररमा, एकता और अखंडता का - India's Preamble also talks about "equality of
आश्वासन दे ते हुए स्थिवत और अिसर की समानता" के बारे में भी status and opportunity while assuring the dignity
बात की गई है। र्ूएचडीआर का अनुच्छे द 22 कहता है वक मानिीर् and unity and integrity of the individual". Article
गररमा और मानि व्यलित्ि के विकास के लिए आर्ििंक, सामाजजक 22 of the UHDR says that economic, social, and
और सांथकृवतक अधधकार अपररहार्य हैं। इसी तरह की अिधारर्ाएाँ cultural rights are indispensable. Similar
भारतीर् संविधान के डीपीएसपी में भी मौजूद हैं।

-:: 41 ::-
RPT 05
- मानि अधधकारों की साियभौधमक र्ोषर्ा के अनुच्छे द 29 में कत्तयव्यों concepts are also present in the DPSP of the
का उल्िेख है। इसी तरह की अिधारर्ा जजसे भारतीर् संविधान में Indian Constitution.
संविधान के भाग IV-A (अनुच्छे द 51A) के तहत् मौलक कत्तुव्य - Duties are mentioned in Article 29 of the
Universal Declaration of Human Rights. A similar
को 42िें संविधान संशोधन अधधवनर्म, 1976 द्वारा डािा गर्ा िा।
concept was inserted in the Indian Constitution
as fundamental duties by the 42nd Constitutional
Amendment Act, 1976 under Part IV-A (Article
51A) of the Constitution.
79. कवभािन के उपरांत, पयनगुठित संकवधान सभा में डॉ. भीमराव 79. After partition, from which province did Dr.
अम्बेडकर ककस प्रांत से प्रकतकनधधत्व करते थे? Bhimrao Ambedkar represented in the
(a) बंगाल reconstituted Constituent Assembly?
(a) Bengal
(b) मध्र् प्रांत एवं बरार
(b) Central Provinces and Berar
(c) बम्बई
(c) Bombay
(d) मरास [c]
(d) Madras
व्याख्या – [c]
डॉ. भीमराव अम्बेडकर- Explanation -
– कैहबनेट धमशन र्ोजना के अंतगुत जयलाई-अगस्त 1946 में संहवधान Dr. Bhimrao Ambedkar-
सभा में अम्बेडकर सवुप्रथम बम्बई से उम्मीदवार थे, जजसमें वे िार – Ambedkar was the first candidate from Bombay
गर्े थे। in the Constituent Assembly in July-August 1946
– अम्बेडकर को हफर से मयल्स्लग लीग के सिर्ोग से बंगाल से under the Cabinet Mission Plan, in which he was
उम्मीदवार बनार्ा गर्ा और वे हवजर्ी रिे। defeated.
– हवभाजन के उपरांत उन्द्िें पयन: बंबई से चयनकर भेजा गर्ा। – Ambedkar was again made a candidate from
Bengal with the support of the Muslim League
and he was victorious.
– After partition, he was re-elected from Bombay.
80. मूि भारतीय संकवधान की कवशेषताओं के संदभु में 80. With reference to the features of the original
वनम्नलिखित कथनों पर विचार कीजजए - Indian Constitution, consider the following
I. भारत का संविधान संर्िाद, धमय वनरपेक्षता, मौलिक अधधकारों statements -
I. The Constitution of India defines its 'Basic
और िोकतंत्र के संदभय में अपनी 'मूि संरचना' को पररभावषत
Structure' in terms of federalism, secularism,
करता है।
fundamental rights and democracy.
II. भारत का संविधान नागररकों की थितंत्रता की रक्षा करने और
II. The Constitution of India provides for
उन आदशों को संरणक्षत करने के लिए 'न्र्ावर्क समीक्षा' प्रदान 'Judicial Review' to protect the liberties of the
करता है जजन पर संविधान आधाररत है। citizens and preserve the ideals on which the
उपयुथक्त कथनों में से कौन सा/से सत्य है/हैं? Constitution is based.
(a) केिि I Which of the above statements is/are true?
(b) केिि II (a) I only
(c) I और II दोनों (b) II only
(d) न तो I, न ही II (c) Both I and II
(d) Neither I nor II
[d]
[d]
व्याख्या –
Explanation -
- सुप्रीम कोिय ने बुवनर्ादी ढ़ााँचे को पररभावषत वकर्ा है। 1973 में
- The Supreme Court has defined infrastructure.
केशिानंद भारती बनाम केरि राज्र् मामिे में ऐवतहालसक वनर्यर् में The concept of 'basic structure' came into
'मूि संरचना' की अिधारर्ा अस्थतत्ि में आई। existence in 1973 in the landmark judgment in
- न्र्ावर्क समीक्षा का तात्पर्य अन्र् सरकारी समन्िर् वनकार्ों की Kesavananda Bharati vs State of Kerala.
न्र्ार्पालिका द्वारा शलि के प्रर्ोग की वनगरानी करना है, तावक र्ह - Judicial review refers to monitoring the exercise
सुवनणित वकर्ा जा सके वक िे अपनी शलिर्ों पर संविधान द्वारा of power by the judiciary of other government
वनधायररत सीमाओं के भीतर रहें। coordinating bodies, to ensure that they stay
within the limits set by the constitution on their
powers.
-:: 42 ::-
RPT 05
- न्र्ावर्क समीक्षा की अिधारर्ा भारत में संविधान का मूि लसद्धांत - The concept of judicial review is a fundamental
है। र्द्यवप न्र्ावर्क समीक्षा के लिए भारतीर् संविधान में कोई थपि principle of the Constitution in India. Although
प्रािधान नहीं है, र्ह हमारे संविधान का एक अणभन्न अंग है। there is no explicit provision in the Indian
Constitution for judicial review, it is an integral
part of our Constitution.
81. भारत के संविधान के भाग IV में वनवहत प्रािधानों के संदभथ में, 81. With reference to the provisions contained in
वनम्नलिखित में से कौन सा/से कथन सही है/हैं? Part IV of the Constitution of India, which of
I. िे न्र्ार्ािर्ों द्वारा प्रितयनीर् होंगे। the following statements is/are correct?
I. They shall be enforceable by the courts.
II. िे वकसी भी न्र्ार्ािर् द्वारा प्रितयनीर् नहीं होंगे।
II. They shall not be enforceable by any court.
III. इस भाग में वनधायररत लसद्धांत राज्र् द्वारा कानून बनाने को
III. The principles laid down in this part are to
प्रभावित करने के लिए हैं।
influence the making of laws by the State.
उपयुथक्त कथनों में से कौन सा/से सत्य है/हैं? Which of the above statements is/are true?
(a) केिि I (a) I only
(b) केिि II (b) II only
(c) केिि I और III (c) I and III only
(d) केिि II और III (d) II and III only
[d] [d]
व्याख्या – Explanation -
- Part IV of the Indian Constitution deals with the
- भारतीर् संविधान का भाग IV हमारी राज्र् नीवत (DPSP) के
Directive Principles of our State Policy (DPSP).
वनदे शक लसद्धांतों से संबंधधत है। इस भाग में वनवहत प्रािधानों को
The provisions contained in this part cannot be
वकसी भी न्र्ार्ािर् द्वारा िागू नहीं वकर्ा जा सकता है।
enforced by any court.
- डीपीएसपी दे श के शासन में मूल क्तसद्ांत है और कानून बनाने में इन - The DPSP is fundamental in the governance of
लसद्धांतों को िागू करना राज्र् का कत्तयव्य होगा। DPSP राज्र् पर the country and it shall be the duty of the State to
सकारात्मक दावर्त्ि हैं। र्े राज्र् द्वारा कानून बनाने को प्रभावित apply these principles in making laws. DPSP is a
करते हैं। positive obligation on the state. They influence
the making of laws by the state.
82. वनम्नलिखित कथनों पर विचार कीजजए - 82. Consider the following statements -
I. भारत के संविधान के अनुसार, कोई भी ऐसा व्यलि जो मतदान I. According to the Constitution of India, any
के लिए र्ोग्र् है, वकसी राज्र् में छह माह के लिए मंत्री बनार्ा person who is eligible to vote can be made a
minister in a state for six months even if he is
जा सकता है जब वक िह उस राज्र् के विधान-मंडि का सदथर्
not a member of the legislature of that state.
नहीं है।
II. According to the Representation of the
II. िोक प्रवतवनधधत्ि अधधवनर्म, 1951 के अनुसार, कोई भी
People Act, 1951, any person who has been
ऐसा व्यलि जो दांधडक अपराध के अन्तगयत दोषी पार्ा गर्ा है convicted of a criminal offense and
और जजसे पााँच िषय के लिए कारािास का दं ड टदर्ा गर्ा है, sentenced to imprisonment for a term of five
चुनाि िड़ने के लिए थिार्ी तौर पर वनरहयत हो जाता है भिे ही years, is permanently disqualified from
िह कारािास से मुि हो चुका हो। contesting elections even if he had become
उपयुथक्त कथनों में से कौन सा/से सत्य है/हैं? free.
(a) केिि I Which of the above statements is/are true?
(b) केिि II (a) I only
(b) II only
(c) I और II दोनों
(c) Both I and II
(d) न तो I, न ही II
(d) Neither I nor II
[d]
[d]
व्याख्या – Explanation -
- प्रत्र्ेक नागररक जो 18 िषय की आर्ु के अहयकारी वतलि को र्ोग्र् - Every citizen who is disqualified on the qualifying
र्ोवषत वकर्ा जाता है, मतदाता के रूप में नामांवकत होने के र्ोग्र् date of 18 years of age is eligible to be enrolled
है। एक व्यलि जो विधार्क नहीं है, िह मंत्री बन सकता है र्टद िह as a voter. A person who is not an MLA can
अपनी वनर्ुलि की तारीख से छह महीने के भीतर राज्र् विधानमंडि become a minister if he gets himself elected to
के लिए वनिायधचत िोता है। हािांवक, जबवक मतदान की उम्र 18 िषय the State Legislature within six months from the
-:: 43 ::-
RPT 05
है, एक व्यलि 25 िषय की आर्ु के बाद ही विधार्क हो सकता। date of his appointment. However, while the
RPA, 1951 की धारा 8 के अनुसार कुछ अपराधों के दोषी ठहराए voting age is 18 years, a person can be an MLA
जाने पर अर्ोग्र्ता का आधार बताती है। दोषी उम्मीदिार को संसद only after the age of 25 states that conviction of
certain offenses as grounds for disqualification.
र्ा राज्र् विधानमंडि के वकसी भी सदन का सदथर् होने से अर्ोग्र्
A convicted candidate is disqualified from being
र्ोवषत वकर्ा जाता है। र्ह थिार्ी नहीं है।
a member of either House of Parliament or the
State Legislature. It is not permanent.
83. कनम्नलिखित में से कौनसी मकहिा संकवधान सभा की मकहिा 83. Who among the following women was not
सदस्य के रूप में कनवाुधचत नहीं थी? elected as a woman member of the
(a) िंसा मेिता Constituent Assembly?
(a) Hansa Mehta
(b) राजकयमारी अमृत कौर
(b) Princess Amrit Kaur
(c) सयचेता कृपलानी
(c) Sucheta Kriplani
(d) लक्ष्मी सिगल [d]
(d) Lakshmi Sehgal [d]
व्याख्या – Explanation -
– संकवधान सभा में कनवाुधचत कयि मकहिाएाँ – – Women elected in the Constituent Assembly –
1. अम्मू स्वामी नाथन 1. Ammu Swami Nathan
2. बेगम एजाज रसूल 2. Begum Ejaz Rasool
3. िंसा मेिता 3. Hansa Mehta
4. लीला रॉर् 4. Leela Roy
5. पूर्णिमा बनजी 5. Poornima Banerjee
6. Renuka Roy
6. रेणयका रॉर्
7. Sucheta Kripalani
7. सयचेता कृपलानी
8. Annie Mascarene
8. एनी मास्करीन
9. Dakshayani Velayudhan
9. दक्षर्नी वेलार्यधन 10. Kamla Choudhary
10. कमला चौधरी 11. Malati Choudhary
11. मालती चौधरी 12. Sarojini Naidu
12. सरोजजनी नार्डू 13. Rajkumari Amrit Kaur
13. राजकयमारी अमृत कौर 14. Vijayalakshmi Pandit
14. हवजर्लक्ष्मी पल्डडत 15. Durgabhai Deshmukh
15. दुगाुभाई दे शमयख
84. ककस मामिे में ओ.बी.सी. के आरक्षण में नवोन्नत वगु की 84. In what case the concept of innovative class
अवधारणा को प्रस्तयत ककया गया है? has been presented in the reservation of
(a) एम. नागराज बनाम भारत संघ वाद O.B.C?
(a) M. Nagaraj v/s Union of India case
(b) इंरा सािनी बनाम भारत संघ वाद
(b) Indra Sahney v/s Union of India case
(c) आई. आर. कोलिो बनाम तधमलनाडय वाद
(c) I.R. Coelho v/s Tamil Nadu Case
(d) धमनवाु धमलस बनाम भारत संघ वाद
(d) Minerva Mills v/s Union of India case
[b] [b]
व्याख्या – Explanation -
इंद्रा साहनी बनाम भारत संघ वाद (1992)- Indra Sawhney vs Union of India case (1992)-
– इस वाद में पदोन्द्नहत में आरक्षण को सवोच्च न्द्र्ार्ालर् द्वारा – In this case, reservation in promotion was
असंवैधाहनक और शून्द्र् माना गर्ा क्र्ोंहक सावुजहनक सेवाओं में considered unconstitutional and void by the
भती के समर् केवल प्रदे श के स्तर पर अनयच्छे द 16(4) के तित् Supreme Court because at the time of
राज्र् को हपछड़े वगों के नागररकों के पक्ष में आरक्षण की शक्तक्त recruitment in public services, only at the state
level, under Article 16 (4), the state was given the
प्रदान की गई िैं।
power of reservation in favor of citizens
belonging to backward classes.

-:: 44 ::-
RPT 05
85. कनम्नलिखित में से कौनसा मामिा संसद की संकवधान के 85. Which one of the following matters is not
संशोधन की शलक्त से संबंधधत नहीं है? related to the power of Parliament to amend
I. शंकरी प्रसाद मामला the Constitution?
I. Shankari Prasad case
II. सज्जन ससिि मामला
II. Sajjan singh case
III. एस. आर. बोम्मई मामला
III. S. R. Bommai case
IV. गोलकनाथ मामला
IV. Golaknath case
सही कूट- Correct Code-
(a) केवल I और II (a) I and II only
(b) केवल II (b) II only
(c) केवल III (c) III only
(d) केवल III और IV (d) III and IV only
[c] [c]
व्याख्या – Explanation -
Following are the cases related to the power
संसद की संकवधान के संशोधन की शलक्त से संबंधधत मामिे
of the Parliament to amend the Constitution:
कनम्नलिखित हैं-
Shankari Prasad case-1951-
शंकरी प्रसाद मामिा-1951-
– Main topic: The constitutional validity of the First
– मुख्य विषय: इस मामिे में पहिे संशोधन अधधवनर्म (िषय 1951) Amendment Act (1951) was challenged in this
की संिैधावनक िैधता को चुनौती दी गई िी। case.
– सुप्रीम कोिय ने िैसिा सुनार्ा वक अनुच्छे द 368 के तहत् संविधान – The Supreme Court ruled that Parliament's
में संशोधन करने की संसद की शलि में मौलिक अधधकारों में power to amend the Constitution under Article
संशोधन करने की शलि भी शाधमि है। 368 included the power to amend fundamental
सज्िन ससिंह मामिा-1965- rights.
– सज्जन लसिंह बनाम राजथिान सरकार मामिे में सिोच्च न्र्ार्ािर् Sajjan Singh Case-1965-
– In Sajjan Singh v. State of Rajasthan, the Supreme
ने अपने पूियिती वनर्यर् को दोहरार्ा अिायत् संसद सामान्र् विधध
Court reiterated its earlier decision that
द्वारा मूि अधधकारों र्ा मौलिक अधधकारों को सीधमत नहीं कर
Parliament cannot limit fundamental rights or
सकती है िेवकन संविधान संशोधन अधधवनर्म के द्वारा सीधमत कर
fundamental rights by ordinary law but can limit
सकती है। by constitutional amendment act.
गोिकनाथ मामिा-1967- Golaknath case-1967-
– मुख्य विषय: इस मामिे में सुप्रीम कोिय ने िैसिा सुनार्ा वक संसद – Main topic: In this case, the Supreme Court
वकसी भी मौलिक अधधकार को कम र्ा छीन निीं सकती िै। pronuced vadic that the Parliament cannot take
– कोिय ने कहा वक वनदे शक लसद्धांतों के कार्ायन्िर्न के लिर्े मौलिक away or abridge any fundamental right.
अधधकारों में संशोधन नहीं वकर्ा जा सकता है। – The Court said that Fundamental Rights cannot
एस. आर. बोम्मई मामिा-1994- be amended for the implementation of Directive
Principles.
– मुख्य विषय: इस मामिे में सिोच्च न्र्ार्ािर् ने र्ह वनधायररत वकर्ा
S. R. Bommai Case-1994-
वक संविधान संर्ीर् है और संर्िाद इसके 'मूि ढााँचे' में शाधमि है।
– Main Theme: In this case, the Supreme Court
– इस मामिे में गौर वकर्ा गर्ा वक हमारे संविधान की र्ोजना के तहत्
determined that the Constitution is federal and
राज्र्ों की तुिना में केंद्र को अधधक शलि प्रदान की जाती है, इसका federalism is included in its 'Basic Structure'.
मतिब र्ह नहीं है वक राज्र् केिि केंद्र के उपांग हैं। – It is observed in this matter that the Center is
given more power than the States under the
scheme of our Constitution, it does not mean that
the States are only appendages of the Centre.
86. िोकसभा अध्यि के कायाथिय के संबंध में वनम्नलिखित 86. Consider the following statements regarding
कथनों पर विचार करें - the office of the Speaker of the Lok Sabha -
I. िह राष्ट्रपवत के प्रसादपर्ंत पद धारर् करता है। I. He holds office during the pleasure of the
President.
II. उसे अपने चुनाि के समर् सदन का सदथर् होने की
II. He need not be a member of the House at
आिश्र्कता नहीं है, िेवकन उसे अपने चुनाि की तारीख से
the time of his election, but he has to become
छह महीने के भीतर सदन का सदथर् बनना होगा।

-:: 45 ::-
RPT 05
III. र्टद िह इथतीिा दे ना चाहता है, तो उसे इथतीिे का पत्र धडप्िी a member of the House within six months
थपीकर को संबोधधत करना होगा। from the date of his election.
उपययुक्त में से कौनसा/से कथन सत्य है/हैं? III. If he wants to resign, his resignation letter
has to be addressed to the Deputy Speaker.
(a) केिि I और II
Which of the above statement(s) is/are true?
(b) केिि III
(a) I and II only
(c) I, II और III
(b) III only
(d) इनमें से कोई नहीं (c) I, II and III
[b] (d) None of these
व्याख्या – [b]
- अनुच्छे द 93 - िोकसभा सदन के दो सदथर्ों को िमश: अध्र्क्ष Explanation -
और उपाध्र्क्ष के रूप में चुनती है तिा र्टद अध्र्क्ष इथतीिा दे ना - Article 93 - The Lok Sabha elects two members of
चाहता है, तो उसे इथतीिे का पत्र धडप्िी थपीकर को संबोधधत करना the House as Speaker and Deputy Speaker
होगा। respectively and if the Speaker wants to resign,
he has to address the letter of resignation to the
Deputy Speaker.
87. बजट के वनम्नलिखित चरणों को सही क्रम में लिखिए– 87. Write the following phases of budget in
I. आम बहस। correct sequence-
II. अनुदान की मााँग पर मतदान। I. Common debate.
II. Voting on Demands for Grants.
III. बजि का प्रथतुतीकरर्।
III. Budget presentation.
IV. विभागीर् सधमवतर्ों द्वारा जााँच।
IV. Enquiry by departmental committees.
V. वित्त विधेर्क का पाररत होना।
V. Passing of the Finance Bill.
VI. विवनर्ोग विधेर्क का पाररत होना। VI. Passing of Appropriation Bill.
कूट - Code -
(a) III, I, IV, II, VI, V (a) III, I, IV, II, VI, V
(b) I, III, II, IV, V, VI (b) I, III, II, IV, V, VI
(c) IV, II, I, III, VI, V (c) IV, II, I, III, VI, V
(d) I, III, IV, II, V, VI [a] (d) I, III, IV, II, V, VI [a]
व्याख्या – Explanation -
- Budget passes through 6 phases in
संसद में बजट 6 चरणों से गुजरता है –
Parliament–
I. बजि का प्रथतुतीकरर्।
I. Presentation of Budget.
II. आम बहस।
II. General debate.
III. विभागीर् सधमवतर्ों द्वारा जााँच। III. Enquiry by departmental committees.
IV. अनुदान की मााँग पर मतदान। IV. Voting on Demands for Grants.
V. विवनर्ोग विधेर्क का पाररत होना। V. Passing of the Appropriation Bill.
VI. वित्त विधेर्क का पाररत होना। VI. Passing of Finance Bill.
88. भारत में, यह सुवनक्षित करने के अिािा वक सािथजवनक धन 88. In India, apart from ensuring that public
का कुशितापूिथक और इस्च्छत उद्दे श्य के लिए उपयोग वकया money is used efficiently and for the intended
जाता है, वनयंरक और महािेिा परीिक (CAG) के कायाथिय purpose, what is the importance of the office
of the Comptroller and Auditor General
का क्या महत्ि है?
(CAG)?
I. वनर्ंत्रक और महािेखा परीक्षक संसद की ओर से राजकोष पर
I. The Comptroller and Auditor General
वनर्ंत्रर् रखता है जब भारत के राष्ट्रपवत राष्ट्रीर्
exercises control over the exchequer on
आपातकाि/वित्तीर् आपातकाि की र्ोषर्ा करते हैं। behalf of the Parliament when the President
II. मंत्रािर्ों द्वारा पररर्ोजनाओं र्ा कार्यिमों के वनष्पादन पर of India declares National Emergency
सीएजी की ररपोिय पर िोक िेखा सधमवत द्वारा चचाय की जाती /Financial Emergency.
है। II. The CAG's reports on the execution of
III. सीएजी ररपोिय की जानकारी का उपर्ोग जााँच एजेंलसर्ों द्वारा projects or programs by the ministries are
उन िोगों के खखिाि आरोप िगाने के लिए वकर्ा जा सकता discussed by the Public Accounts Committee.

-:: 46 ::-
RPT 05
है, जजन्होंने साियजवनक वित्त का प्रबंधन करते हुए कानून का III. The information in the CAG report can be
उल्िंर्न वकर्ा है। used by the investigating agencies to frame
IV. सरकारी कंपवनर्ों के ऑधडि और अकाउंटििंग से वनपिने के charges against those who have violated the
law while managing public finances.
दौरान CAG के पास कानून का उल्िंर्न करने िािों के
IV. While dealing with the audit and accounting
खखिाि मुकदमा चिाने के लिए कुछ न्र्ावर्क शलिर्ााँ हैं।
of government companies, the CAG has
सही कूट
certain judicial powers to prosecute those
(a) केिि I, III और IV who violate the law.
(b) केिि II Correct code
(c) केिि II और III (a) I, III and IV only
(d) I, II, III और IV (b) II only
[c] (c) II and III only
व्याख्या – (d) I, II, III and IV
- भारतीर् संविधान के अनुच्छे द 148 के अनुसार भारत के वनर्ंत्रक [c]
Explanation -
और महािेखापरीक्षक (Comptroller & Auditor General
- According to Constitution Article 148 the
of India-CAG) भारत के संविधान के तहत् एक थितंत्र
Comptroller & Auditor General of India (CAG) is
प्राधधकरर् है।
an independent authority under the Constitution
- र्ह साियजवनक क्षेत्र की भारतीर् िेखा परीक्षा और िेखा विभाग of India.
(Indian Audit & Accounts Department) का प्रमुख - It is the head of the Indian Audit & Accounts
संरक्षक है। Department and the main patron of the public
- इसके माध्र्म से संसद और राज्र् विधानसभाओं के लिर्े सरकार sector.
और अन्र् साियजवनक प्राधधकरर्ों (साियजवनक धन खचय करने - Through it, the accountability of the government
िािे) की जिाबदे ही सुवनणित की जाती है और र्ह जानकारी and other public authorities (spending public
जनसाधारर् को दी जाती है। money) to the Parliament and State Legislatures
is ensured and this information is given to the
- सरकारी कंपवनर्ों के ऑधडि और अकाउंटििंग से वनपिने के दौरान
general public.
CAG के पास कानून का उल्िंर्न करने िािों के खखिाि मुकदमा
- While dealing with the audit and accounting of
चिाने के लिए कोई न्र्ावर्क शलि नहीं हैं।
government companies, the CAG has no judicial
powers to prosecute those who violate the law.
89. कथन (A):-नीवत आयोग एक गैर संिैधावनक, गैर िैधावनक 89. Assertion (A):-NITI Aayog is an
वनकाय है। unconstitutional, non-statutory body.
कारण (R):- नीवत आयोग की स्थापना भारत सरकार के एक Reason (R) :- NITI Aayog has been established
by an executive resolution of the Government
कायथकारी संकल्प द्वारा की गई है।
of India.
(a) A ि R दोनों सही है तिा R, A की सही व्याख्र्ा करता है।
(a) Both A and R are true and R is the correct
(b) A ि R दोनों सही है तिा R, A की सही व्याख्र्ा नहीं करता
explanation of A.
है। (b) Both A and R are correct and R is not the
(c) ‘A‘ सही है, परन्तु ‘R‘ गित है। correct explanation of A.
(d) ‘A‘ गित है, परन्तु ‘R‘ सही है। (c) 'A' is correct but 'R' is incorrect.
[a] (d) 'A' is incorrect but 'R' is correct.
व्याख्या – [a]
– नीवत आर्ोग (राष्रीर् भारत पररितयन संथिान) भारत सरकार द्वारा Explanation -
गटठत एक संथिान है, जजसे र्ोजना आर्ोग के थिान पर बनार्ा गर्ा – NITI Aayog (National Institute for Transforming
India) is an institution formed by the
है। 1 जनिरी, 2015 को इस नए संथिान के सम्बन्ध में मन्न्त्रमण्डि
Government of India, which has been created in
का प्रथताि जारी वकर्ा गर्ा। र्ह संथिान सरकार के लििंक िैं क के
place of the Planning Commission. On January 1,
रूप में सेिाएाँ, वनदे श एिं नीवतगत गवतशीिता प्रदान करता है। नीवत
2015, a cabinet resolution regarding this new
आर्ोग भारत सरकार के एक कार्यकारी संकल्प द्वारा सृजजत institute was issued. The institute serves as a
वनकार् है अिायत् र्ह न तो संिैधावनक है, न ही िैधावनक वनकार्। think tank to the government providing guidance
and policy dynamics. NITI Aayog is a body
created by an executive resolution of the
-:: 47 ::-
RPT 05
Government of India i.e. it is neither a
constitutional nor a statutory body.
90. केंद्र सरकार द्वारा सरकारी व्यिसाय के संचािन के लिए करों 90. All the revenue received by the Central
और अन्त्य प्राम्प्तयों के माध्यम से प्राप्त सभी राजस्ि को वकसमें Government by way of taxes and other
जमा वकया जाता है? receipts for the conduct of Government
business is deposited.
(a) भारत की आकस्थमकता वनधध
(a) Contingency Fund of India
(b) साियजवनक खाता
(b) Public Account
(c) भारत की संधचत वनधध
(c) Consolidated Fund of India
(d) जमा और अवग्रम वनधध (d) Deposit and Advance Fund
[c] [c]
व्याख्या – Explanation -
- भारतीर् संविधान का अनुच्छे द-266(1) भारत की संधचत वनधध से - Article-266 (1) of the Indian Constitution is
संबंधधत है। केंद्र सरकार द्वारा सरकारी व्यिसार् के संचािन के लिए related to the Consolidated Fund of India. All
करों और अन्र् प्राप्प्तर्ों के माध्र्म से प्राप्त सभी राजथि भारत की revenue received by the Central Government by
संधचत वनधध में जमा वकए जाते हैं। way of taxes and other receipts for the conduct
of Government business is credited to the
Consolidated Fund of India.
91. भारत में अंतररम सरकार के संदभथ में सूची-I को सूची-II से 91. Match List-I with List-II with reference to
सुमेलित कीजजए- Interim Government in India-
सूची–I सूची-II List – I List – II
A. Sardar Vallabh bhai I. Home
A. सरदार िल्िभ भाई I. गृह विभाग
Patel Department
पिे ि
B. Liaquat Ali Khan II. Railway and
B. लिर्ाकत अिी खान II. रेििे एिं पररिहन विभाग
Transport
C. डॉ. राजेन्द्र प्रसाद III. वित्त विभाग Department
D. आसि अिी IV. खाद्य एिं कृवष विभाग C. Dr. Rajendra III. Finance
कूट: Prasad department
(a) A-IV B-II C-I D-III D. Asaf Ali IV. Food and
(b) A-III B-IV C-II D-I Agriculture
(c) A-I B-II C-III D-IV Department
(d) A-I B-III C-IV D-II Code:
(a) A-IV B-II C-I D-III
[d]
(b) A-III B-IV C-II D-I
व्याख्या –
(c) A-I B-II C-III D-IV
सदस्य धाररत विभाग
(d) A-I B-III C-IV D-II
सरदार िल्िभ भाई पिे ि – गृह, सूचना एिं प्रसारर् [d]
लिर्ाकत अिी खान – वित्त Explanation –
डॉ. राजेन्द्र प्रसाद – खाद्य एिं कृवष Member Holding department
आसि अिी – रेििे एिं पररिहन Sardar Vallabh – Home, Information
bhai Patel and Broadcasting
Liaquat Ali Khan – Finance
Dr. Rajendra – Food and
Prasad Agriculture
Asaf Ali – Railways and
Transport
92. ‘िेिानुदान’ और ‘अंतररम बजट’ के बीच अंतर है- 92. The difference between ‘Vote on Account’ and
I. एक ‘िेखानुदान’ केिि सरकार के बजि में व्यर् से संबंधधत "Interim Budget" is-
होता है, जबवक एक ‘अंतररम बजि’ में व्यर् और प्राप्प्तर्ााँ I. A ‘vote-on-account’ deals only with
expenditures in the government's budget,
दोनों शाधमि होते हैं।
while an ‘interim budget’ includes both
expenditures and receipts.
-:: 48 ::-
RPT 05
II. ‘िेखानुदान’ का प्रािधान एक वनर्धमत सरकार द्वारा उपर्ोग II. A ‘Vote on Account’ is a provision used by a
वकर्ा जाता है, जबवक एक ‘अंतररम बजि’ एक कार्यिाहक regular government, whereas an ‘Interim
सरकार द्वारा उपर्ोग वकर्ा जाने िािा प्रािधान है। Budget’ is a provision used by a caretaker
government.
उपययुक्त में से कौनसा/से कथन सत्य है/हैं?
Which of the above statement(s) is/are true?
(a) केिि I
(a) I only
(b) केिि II
(b) II only
(c) I और II दोनों (c) Both I and II
(d) न तो I, न ही II (d) Neither I nor II
[a] [a]
व्याख्या – Explanation -
- िेखानुदान केिि सरकार के बजि के व्यर् पक्ष से संबंधधत होता है। - Parliament's approval is required before the
- सरकार (कार्यपालिका) को धन वनकािने और व्यर् करने से पहिे government (executive) withdraws and spends
संसद के अनुमोदन की आिश्र्कता होती है। बजि पेश करने और money. The presentation and passing of the
budget took more time than expected. Since
पाररत करने में अनुमान से अधधक समर् िगता है। चूंवक संसद नए
Parliament is not able to vote on the full budget
वित्तीर् िषय की शुरुआत से पहिे पूर्य बजि पर मतदान करने में
before the start of the new financial year, money
सक्षम नहीं है, इसलिए चीजों को हमेशा की तरह चिाने के लिए
is needed by the government to keep things
सरकार द्वारा धन की आिश्र्कता होती है। running as usual.
- िेखानुदान अवग्रम अनुदान है जो सरकार को पूर्य बजि, अनुदान - Vote on Account is an advance grant that enables
की मााँग, वित्त विधेर्क और विवनर्ोग विधेर्क पाररत होने तक कार्य the government to continue functioning until the
जारी रखने में सक्षम बनाता है। full budget, demands for grants, finance bills and
- अंतररम बजि एक तरह से पूर्य बजि होता है िेवकन इसे सरकार appropriation bills are passed.
द्वारा अपने कार्यकाि के अंवतम िषय में र्ानी चुनाि से पहिे बनार्ा - Vote-on-account is only related to the
जाता है। इसमें व्यर् और प्राप्प्तर्ााँ दोनों शाधमि हैं। expenditure side of the budget of the
government.
- Interim budget is a kind of full budget but it is
made by the government in the last year of its
tenure i.e. before elections. It includes both
expenditure and receipts.
93. भारत के संविधान के तहत्, वनम्नलिखित में से कौन सा 93. Under the Constitution of India, which one of
मौलिक कत्तथव्य नहीं है? the following is not a fundamental duty?
(a) साियजवनक चुनािों में मतदान करना। (a) To vote in public elections
(b) To develop scientific temper
(b) िैज्ञावनक थिभाि विकलसत करना।
(c) To protect public property
(c) साियजवनक संपलत्त की रक्षा करना।
(d) To abide by the constitution and respect its
(d) संविधान का पािन करना और उसके आदशों का सम्मान
ideals. [a]
करना। [a] Explanation -
व्याख्या – - Voting in public elections is not a fundamental
- साियजवनक चुनािों में मतदान करना कोई मौलिक कत्तयव्य नहीं है। duty. Developing scientific temper, protecting
िैज्ञावनक थिभाि विकलसत करना, साियजवनक संपलत्त की रक्षा public property, following our constitution and
करना, हमारे संविधान का पािन करना और उसके आदशों का respecting its ideals are all fundamental duties.
सम्मान करना र्े सभी मौलिक कत्तयव्य िैं।
94. भारतीय न्त्यायपालिका के संदभथ में वनम्नलिखित कथनों पर 94. With reference to the Indian Judiciary,
विचार कीजजए - consider the following statements
I. भारत के सिोच्च न्र्ार्ािर् के वकसी भी सेिावनिृत्त न्र्ार्ाधीश I. Any retired Judge of the Supreme Court of
India can be called back to sit by the Chief
को भारत के मुख्र् न्र्ार्ाधीश द्वारा भारत के राष्ट्रपवत की पूिय
Justice of India with the prior permission of
अनुमवत से बैठने के लिए िापस बुिार्ा जा सकता है।
the President of India.
II. भारत में एक उच्च न्र्ार्ािर् के पास अपने वनर्यर् की समीक्षा
II. A High Court in India has the power to review
करने की शलि है जैसा वक सिोच्च न्र्ार्ािर् करता है। its decision as does the Supreme Court.
उपयुथक्त कथनों में से कौन सा/से सत्य है/हैं? Which of the above statements is/are true?
-:: 49 ::-
RPT 05
(a) केिि I (a) I only
(b) केिि II (b) II only
(c) I और II दोनों (c) Both I and II
(d) Neither I nor II
(d) न तो I, न ही II
[c]
[c]
Explanation -
व्याख्या –
- Subject to the provisions of any law and rules
- भारत के संविधान के अनुच्छे द 137 के अनुसार, अनुच्छे द 145 के made under Article 145 as per Article 137 of the
तहत् बनाए गए वकसी भी कानून और वनर्मों के प्रािधानों के अधीन, Constitution of India, the Supreme Court has the
सिोच्च न्र्ार्ािर् के पास उसके द्वारा सुनाए गए वकसी भी वनर्यर् power to review any judgment pronounced or
र्ा वकए गए आदे श की समीक्षा करने की शलि है। order made by it.
- भारतीर् संविधान के अनुच्छे द 128 के अनुसार, भारत के मुख्र् - According to Article 128 of the Constitution of
न्र्ार्ाधीश वकसी भी समर् सिोच्च न्र्ार्ािर् के सेिावनिृत्त India, the Chief Justice of India may at any time
न्र्ार्ाधीश र्ा उच्च न्र्ार्ािर् के सेिावनिृत्त न्र्ार्ाधीश से अनुरोध request a retired Supreme Court judge or a
retired High Court judge. He can do so only with
कर सकते हैं। िह ऐसा केिि राष्ट्रपवत और इस प्रकार वनर्ुि वकए
the prior consent of the President and the person
जाने िािे व्यलि की पूिय सहमवत से ही कर सकता है। अणभिेख
so appointed. As a court of record, the High Court
न्र्ार्ािर् के रूप में, उच्च न्र्ार्ािर् संविधान के अनुच्छे द 226 के
can review its decisions under Article 226 of the
तहत् अपने वनर्यर्ों की समीक्षा कर सकता है। Constitution.
95. वनम्नलिखित में से कौन सा/से भारतीय संविधान में वनधाथररत 95. Which of the following is/are the fundamental
नागररकों के मौलिक कत्तथव्य है/हैं? duties of the citizens prescribed in the
I. हमारी सामालसक संथकृवत की समृद्ध विरासत को संरणक्षत Constitution of India?
I. Preserving the rich heritage of our
करना।
composite culture.
II. कमजोर िगों को सामाजजक अन्र्ार् से बचाना।
II. To protect the weaker sections from social
III. िैज्ञावनक सोच और जााँच की भािना का विकास करना।
injustice.
IV. व्यलिगत और सामूवहक गवतविधध के सभी क्षेत्रों में उत्कृिता III. To develop scientific temper and the spirit of
की ओर प्रर्ास करना। inquiry.
सही कूट IV. To strive towards excellence in all spheres of
(a) केिि I और II individual and collective activity.
(b) केिि II Correct code
(c) केिि I, III और IV (a) I and II only
(d) I, II, III और IV (b) II only
(c) I, III and IV only
[c]
(d) I, II, III and IV
व्याख्या –
[c]
- संविधान के भाग 4A के अनुसार हमारी सामालसक संथकृवत की
Explanation -
समृद्ध विरासत को संरणक्षत करना, िैज्ञावनक सोच और जााँच की - According to Part 4A of the Constitution, it is one
भािना का विकास करना तिा व्यलिगत और सामूवहक गवतविधध of the fundamental duties of the citizens to
के सभी क्षेत्रों में उत्कृिता की ओर प्रर्ास करना नागररकों के मौलिक preserve the rich heritage of our composite
कत्तयव्य िैं। culture, to develop scientific temper and the
spirit of inquiry and to strive towards excellence
in all spheres of individual and collective activity.
96. वनम्नलिखित कथनों पर विचार कीजजए - 96. Consider the following statements -
I. भारत के राष्ट्रपवत संसद का सत्र ऐसे थिान पर आहूत कर I. The President of India can summon a session
सकते हैं, जैसा िह उधचत समझें। of Parliament at such place as he thinks fit.
II. The Constitution of India provides three
II. भारत का संविधान एक िषय में संसद के तीन सत्रों का प्रािधान
sessions of the Parliament in a year, but it is
करता है, िेवकन तीनों सत्र आर्ोजजत करना अवनिार्य नहीं है।
not mandatory to hold all the three sessions.
III. एक िषय में संसद की बैठक के लिए न्र्ूनतम टदनों की कोई
III. There is no requirement for the minimum
आिश्र्कता नहीं है। number of days for which Parliament
उपयुथक्त कथनों में से कौन सा/से सत्य है/हैं? meeting in a year.
-:: 50 ::-
RPT 05
(a) केिि I Which of the above statements is/are true?
(b) केिि II (a) I only
(c) केिि I और III (b) II only
(c) I and III only
(d) केिि II और III
(d) II and III only
[c]
[c]
व्याख्या –
Explanation -
- संविधान के अनुच्छे द 85(1) में कहा गर्ा है वक राष्ट्रपवत समर्- - Article 85(1) of the Constitution states that the
समर् पर संसद के प्रत्र्ेक सदन को ऐसे समर् और थिान पर बैठक President shall summon each House of
के लिए बुिाएगा, जैसा वक िह उधचत समझे। Parliament from time to time to meet at such
- िोकसभा का सत्र िषय में कम से कम दो बार बुिार्ा जाता है। time and place as he thinks fit.
- संविधान में र्ह नहीं कहा गर्ा है वक संसद के तीन सत्र एक िषय में - The session of the Lok Sabha is called at least
बुिाए जाएाँ। संसद में एक िषय में बैठक के टदनों की न्र्ूनतम संख्र्ा twice in a year.
का कोई प्रािधान नहीं है। - It is not said in the constitution that three
sessions of the parliament should be called in a
year. There is no provision for minimum number
of days of meetings in a year in the Parliament.
97. भारत की संधचत वनधध से धन की वनकासी के लिए प्राधधकरण 97. The authority to withdraw money from the
आना चावहए- Consolidated Fund of India should come
(a) भारत के राष्ट्रपवत from—
(a) President of India
(b) भारत की संसद
(b) Parliament of India
(c) भारत के प्रधान मंत्री
(c) Prime Minister of India
(d) केंद्रीर् वित्त मंत्री
(d) Union Finance Minister
[b] [b]
व्याख्या – Explanation -
- भारत की संधचत वनधध का गठन संविधान के अनुच्छे द 266(1) के - The Consolidated Fund of India is constituted
तहत होता है। केंद्रीर् कर, आर् कर, उत्पाद शुल्क और कुछ गैर कर under Article 266(1) of the Constitution. Taxes
राजथि सवहत सरकार द्वारा प्राप्त करों को समेवकत वनधध में जमा received by the government including central
वकर्ा जाता है। सरकार के सभी व्यर् इस कोष से वकर्ा जाता है। tax, income tax, excise duty and certain non-tax
इस कोष से वकसी भी रालश को वनकािने के लिए संसद के revenue are credited to the Consolidated Fund.
All the expenditure of the government is done
प्राधधकरर् की आिश्र्कता होती है।
from this fund. Withdrawal of any amount from
this fund requires the authorization of the
Parliament.
98. भारत के संविधान की वनम्नलिखित अनुसूधचयों में से वकस एक 98. Which one of the following schedules of the
में दि-बदि विरोधी प्रािधान हैं? Constitution of India contains anti-defection
(a) दूसरी अनुसूची provisions?
(a) Second Schedule
(b) पांचिीं अनुसूची
(b) Fifth Schedule
(c) आठिीं अनुसूची
(c) Eighth Schedule
(d) दसिीं अनुसूची
(d) Tenth Schedule
[d] [d]
व्याख्या – Explanation -
- भारतीर् संविधान की दसिीं अनुसूची में दिबदि के आधार पर - The Tenth Schedule of the Constitution of India
संसद और राज्र् विधावर्का के सदथर्ों की अर्ोग्र्ता से संबंधधत deals with the provisions relating to
प्रािधानों से संबंधधत है। भारतीर् संविधान की दसिीं अनुसूची में disqualification of members of Parliament and
दिबदि विरोधी कानून का समावेश है। State Legislatures on the ground of defection.
Anti-defection law is included in the Tenth
Schedule of the Indian Constitution.
99. केंद्र सरकार के संदभथ में वनम्नलिखित कथनों पर विचार 99. With reference to the Central Government,
कीजजए - consider the following statements -
-:: 51 ::-
RPT 05
I. ‘एन. गोपाि थिामी आर्ंगर सधमवत’ ने सुझाि टदर्ा वक I. ‘The N.Gopalswami Ayyangar Committee’
प्रशासवनक सुधार के विषर् को आगे बढाने और इसे बढािा suggested that a minister and a secretary
दे ने के लिए एक मंत्री और एक सधचि को नाधमत वकर्ा जाना should be designated to pursue and
promote the theme of administrative reform.
चावहए।
II. In 1970, the Department of Personnel was
II. 1970 में, प्रशासवनक सुधार आर्ोग, 1966 की लसिाररश पर
formed on the recommendation of the
कार्मिंक विभाग का गठन वकर्ा गर्ा िा और इसे प्रधानमंत्री के
Administrative Reforms Commission, 1966
प्रभार में रखा गर्ा िा। and was placed under the charge of the
उपयुथक्त कथनों में से कौन-सा/से सत्य है/हैं? Prime Minister.
(a) केिि I Which of the above statements is/are true?
(b) केिि II (a) I only
(c) I और II दोनों (b) II only
(d) न तो I, न ही II (c) Both I and II
[b] (d) Neither I nor II
[b]
व्याख्या –
Explanation -
- िषय 1949 में, गोपाि थिामी आर्ंगर सधमवत ने केंद्रीर् सधचिािर्
- In the year 1949, the Gopalswami Ayyangar
के पुनगयठन की लसिाररश करते हुए सुझाि टदर्ा वक एक विभाग
Committee, while recommending the
की पहचान एक सधचि के प्रभार के साि की जानी चावहए और एक reorganization of the Central Secretariat,
मंत्रािर् की पहचान एक मंत्री के प्रभार के साि की जानी चावहए। suggested that a Department should be
मंत्रािर् को िमशः सधचि के प्रभार और एक मंत्री के प्रभार के साि identified with the charge of a Secretary and a
पहचाना जाना चावहए। Ministry should be identified with the charge of a
- िषय 1970 में प्रशासवनक सुधार आर्ोग की लसिाररशों के आधार Minister. The ministry should be identified with
पर कैवबनेि सधचिािर् में कार्मिंक विभाग की थिापना की गई। चूाँवक the charge of a secretary and the charge of a
कैवबनेि सधचिािर् प्रधानमंत्री के अंतगयत आता है, इसलिए र्ह minister, respectively.
कहना सही होगा वक कार्मिंक और प्रलशक्षर् विभाग (DoPT) को - In the year 1970, on the basis of the
recommendations of the Administrative Reforms
प्रधानमंत्री के प्रभार के अधीन रखा गर्ा।
Commission, the Department of Personnel was
established in the Cabinet Secretariat. Since the
Cabinet Secretariat comes under the Prime
Minister, it would be correct to say that the
Department of Personnel and Training (DoPT)
was placed under the charge of the Prime
Minister.
100. भारत के संविधान का कौन सा भाग कल्याणकारी राज्य के 100. Which part of the Constitution of India
आदशथ की घोषणा करता है? declares the ideal of Welfare State?
(a) राज्र् नीवत के वनदे शक तत्त्व (a) Directive Principles of State Policy
(b) Fundamental Rights
(b) मौलिक अधधकार
(c) Preamble
(c) प्रथतािना
(d) Seventh Schedule
(d) सातिीं अनुसूची
[a]
[a] Explanation -
व्याख्या – - Welfare state is a concept of government in
- कल्र्ार्कारी राज्र्, सरकार की एक अिधारर्ा है जजसमें राज्र् which the state plays an important role in
अपने नागररकों की आर्ििंक और सामाजजक भिाई के संरक्षर् और protecting and promoting the economic and
संिधयन में मित्त्वपूणु भूधमका वनभाता है। सरकार एक सामाजजक social well-being of its citizens. The government
सुरक्षा जाि की उपिब्धता सुवनणित करती है जजसमें लशक्षा, ensures the availability of a social safety net
आिास, जीविका, थिाथथ्र् सेिा आटद शाधमि हो सकते हैं। which may include education, housing,
livelihood, healthcare etc.
101. ‘िि िीवन धमशन’ के संदभु में कनम्नलिखित कथनों पर 101. With reference to 'Jal Jeevan Mission',
कवचार कीजिए - consider the following statements -

-:: 52 ::-
RPT 05
I. इस र्ोजना के तित् वषु 2024 तक प्रत्र्ेक ग्रामीण पररवार I. Under this scheme, by the year 2024, potable
को घरेलू नल कनेक्शन के माध्र्म से पीने र्ोग्र् पानी उपलब्ध water is to be made available to every rural
करवाना िै। household through domestic tap connection.
II. The participation of the Center and the State
II. इस र्ोजना में केन्द्र एवं राज्र् की भागीदारी 60 : 40 प्रहतशत
in this scheme is 60: 40 percent.
िै।
Which of the above statement(s) is/are true?
उपयुथक्त में कौन-सा/से कथन सत्य है/हैं?
(a) I only
(a) केिि I (b) II only
(b) केिि II (c) Both I and II
(c) I और II दोनों (d) Neither I nor II
(d) न तो I, न ही II [a]
[a] Explanation -
व्याख्या – - Under this scheme, by the year 2024, potable
– इस र्ोजना के तित् वषु 2024 तक प्रत्र्ेक ग्रामीण पररवार को घरेलू water has to be made available to every rural
household through domestic tap connection.
नल कनेक्शन के माध्र्म से पीने र्ोग्र् पानी उपलब्ध करवाना िै।
- The participation of the Center and the State in
– इस र्ोजना में केन्द्र एवं राज्र् की भागीदारी 50 : 50 प्रहतशत िै।
this scheme is 50: 50 percent.
102. ‘प्रधानमंत्री मातृ वन्दना योिना’ के संदभु में कनम्नलिखित 102. With reference to ‘Pradhan Mantri Matri
कथनों पर कवचार कीजिए - Vandana Yojana,’ consider the following
I. इस र्ोजना का उद्दे श्र् गभुवती, धात्री महिलाओं तथा 1 साल statements -
I. The objective of the scheme is to improve the
तक के क्तशशयओं के स्वास््र् एवं पोषण की ल्स्थहत में सयधार
health and nutritional status of pregnant,
करना िै।
lactating women and infants up to one year
II. इस र्ोजना के तित् लाभाथी को 6000 रूपर्े की राक्तश ददर्े
of age.
जाने का प्रावधान िै। II. Under this scheme, there is a provision to
III. इस र्ोजना के तित् लाभाथी के खाते में सीधा भयगतान करने give an amount of Rs.6000 to the beneficiary.
का प्रावधान िै। III. Under this scheme, there is a provision for
उपयुथक्त में कौन-सा/से कथन सत्य है/हैं? direct payment to the beneficiary's account.
(a) केिि III Which of the above statement(s) is/are true?
(b) केिि II और III (a) III only
(c) I और II दोनों (b) II and III only
(c) Both I and II
(d) उपर्युक्त सभी
(d) All of the above
[a]
[a]
व्याख्या –
Explanation -
– इस र्ोजना का उद्दे श्र् गभुवती, धात्री महिलाओं तथा 6 माि तक के - The objective of this scheme is to improve the
क्तशशयओं के स्वास््र् एवं पोषण की ल्स्थहत में सयधार करना िै। health and nutritional status of pregnant,
– इस र्ोजना के तित् लाभाथी को 5000 रूपर्े की राक्तश तीन हकश्तों lactating women and infants up to 6 months.
(1000, 2000, 2000) में ददर्े जाने का प्रावधान िै। – Under this scheme, there is a provision to give an
– इस र्ोजना के तित् लाभाथी के खाते में सीधा भयगतान करने का amount of Rs.5000 in three installments (1000,
प्रावधान िै। 2000, 2000) to the beneficiary.
– Under this scheme, there is a provision for direct
payment to the beneficiary's account.
103. कनम्नलिखित कथनों पर कवचार कीजिए – 103. Consider the following statements -
I. ‘जागृहत बैंक टू वकु र्ोजना’ की लाभाथी केवल महिला िो I. The beneficiary of the 'Jagruti Bank to Work
सकती िै। Scheme' can only be a woman.
II. 'Chief Minister Work from Home-Job Work
II. ‘मयख्र्मंत्री वकु फ्रॉम िोम–जॉब वकु र्ोजना’ में लाभाथी
Scheme' beneficiaries can be both male and
महिला और पयरुष दोनों िो सकते िैं।
female.
उपयुथक्त में कौन-सा/से कथन असत्य है/हैं?
Which of the above statement(s) is/are false?
(a) केिि I (a) I only
(b) केिि II (b) II only
-:: 53 ::-
RPT 05
(c) I और II दोनों (c) Both I and II
(d) न तो I, न ही II (d) Neither I nor II
[b] [b]
Explanation -
व्याख्या –
– 'Jagriti Bank to Work Scheme' has been brought
– ‘जागृहत बैंक टू वकु र्ोजना’ कामकाजी और व्यावसाहर्क क्षेत्र में
for the re-employment of trained women in the
प्रक्तशभक्षत महिलाओं के पयन: रोजगार िेतय लार्ी गर्ी िै, जो शादी के
working and commercial sector, who leave the
बाद घर पररवार संभालने र्ा अन्द्र् कारणों से नौकरी छोड़ दे ती िैं। job after marriage to take care of the family at
– ‘मयख्र्मंत्री वकु फ्रॉम िोम – जॉब वकु र्ोजना’ वकु फ्रॉम िोम द्वारा home or for other reasons.
पररवार की आजीहवका में र्ोगदान दे सकने वाली महिलाओं िेतय िै। – ‘Chief Minister Work from Home – Job Work
Scheme’ is for women who can contribute to the
livelihood of the family through work from home.
104. ‘मयख्यमंत्री धचरंिीवी श्रधमक संबि योिना’ के सदं भु में असत्य 104. In the context of ‘Chief Minister Chiranjeevi
कथन की पहचान कीजिए - Shramik Sambal Yojana,’ identify the false
(a) 25 से 60 वषु आर्य वगु के व्यक्तक्त िी इस र्ोजना के लाभाथी statement -
(a) Only persons in the age group of 25 to 60
िो सकते िैं।
years can be the beneficiaries of this scheme.
(b) इस र्ोजना के तित् पंजीकृत श्रधमक तथा स्रीट वेडडर को
(b) Registered workers and street vendors will
सिार्ता प्राप्त िोगी।
get assistance under this scheme.
(c) इस र्ोजना के तित् रोजगार शयरू करने िेतय 50,000 रूपर्े (c) Financial assistance of up to Rs 50,000 will be
तक की आर्थिक सिार्ता प्राप्त िोगी। received for starting employment under this
(d) इस र्ोजना के तित् श्रधमक को दै हनक मजदूरी समाप्त िोने पर scheme.
7 ददन तक 200 रूपर्े की मजदूरी प्राप्त िोगी। (d) Under this scheme, the laborer will get a
[c] salary of Rs.200 for 7 days on completion of
व्याख्या – daily wages.
– 25 से 60 वषु आर्य वगु के व्यक्तक्त िी इस र्ोजना के लाभाथी िो [c]
Explanation -
सकते िैं।
- Only persons in the age group of 25 to 60 years
– इस र्ोजना के तित् पंजीकृत श्रधमक तथा स्रीट वेडडर को सिार्ता
can be the beneficiaries of this scheme.
प्राप्त िोगी।
- Registered laborers and street vendors will get
– इस र्ोजना के तित् अस्पताल में भती पंजीकृत श्रधमक तथा स्रीट assistance under this scheme.
वेडडर को सिार्ता प्राप्त िोगी। – Under this scheme, registered laborers and
- इस र्ोजना के तित् श्रधमक को दै हनक मजदूरी समाप्त िोने पर 7 street vendors admitted to the hospital will get
ददन तक 200 रूपर्े की मजदूरी प्राप्त िोगी। assistance.
- Under this scheme, the laborer will get a salary of
Rs.200 for 7 days on completion of daily wages.
105. रािस्थान सरकार द्वारा संचालित ‘इजन्दरा मकहिा शलक्त उड़ान 105. The objectives of the 'Indira Mahila Shakti
योिना’ के उद्दे श्य हैं - Udaan Yojana' run by the Government of
(a) महिला एवं स्वर्ं सिार्ता समूिों को उधचत मूलर् की दुकानों Rajasthan are -
(a) Allotment of fair price shops to women and
का आवंटन।
self-help groups.
(b) लड़हकर्ों/महिलाओं को हन:शयलक आर.एस.सी.आई.टी.
(b) Providing Free RSCIT traning to girls/women.
प्रक्तशक्षण प्रदान करना।
(c) To establish an effective monitoring system
(c) मयख्र्मंत्री बीस सूत्री कार्ुक्रम की प्रभावी हनगरानी व्यवस्था of the Chief Minister's twenty-point
स्थाहपत करना। program.
(d) राजस्थान में लड़हकर्ों और महिलाओं को मयफ्त सैहनटरी (d) Providing free sanitary napkins to girls and
नैपहकन प्रदान करना। women in Rajasthan.
[d] [d]
व्याख्या – Explanation -
‘इजन्दरा मकहिा शलक्त उड़ान योिना’ का उद्दे श्य- Objective of the 'Indira Mahila Shakti Udan
Yojana'-

-:: 54 ::-
RPT 05
– लड़हकर्ों व महिलाओं में माक्तसक धमु स्वास््र् और स्वच्छता प्रबंधन - To create awareness about menstrual health and
के बारे में जागरूकता पैदा करना। hygiene management among girls and women.
– राजस्थान में लड़हकर्ों और महिलाओं को मयफ्त सैहनटरी नैपहकन - Providing free sanitary napkins to girls and
women in Rajasthan.
प्रदान करना।
106. कनम्नलिखित में से कौनसी योिना िापान अंतरराष्ट्रीय कॉ- 106. Which of the following scheme is operated by
ऑपरेशन एिेन्सी (JICA) द्वारा संचालित है - Japan International Co-operation Agency
(a) बााँध पयनुवास और सयधार पररर्ोजना-II (JICA)
(a) Dam Rehabilitation and Improvement
(b) रेहगस्तान क्षेत्र के क्तलए जल क्षेत्र पयन: संरचना पररर्ोजना
Project II
(c) राजस्थान जल क्षेत्र आजीहवका सयधार पररर्ोजना
(b) Watershed Restructuring Project for Desert
(d) राजस्थान शिरी क्षेत्र हवकास कार्ुक्रम
Area
[c] (c) Rajasthan Water Sector Livelihood
व्याख्या – Improvement Project
– राजस्थान में जापान अन्द्तराुष्ट्रीर् कॉ–ऑपरेशन एजेन्द्सी (JICA) (d) Rajasthan Urban Area Development
द्वारा संचाक्तलत बाह्य सिार्ता प्राप्त पररर्ोजना। Program
1. राजस्थान जल क्षेत्र आजीहवका सयधार पररर्ोजना। [c]
2. राजस्थान ग्रामीण जलापूर्ति एवं फ्लोरोक्तसस हनराकरण Explanation -
पररर्ोजना। – Externally Aided Project in Rajasthan to be
implemented by Japan International Co-
– बााँध पयनवाुस और सयधार पररर्ोजना–II हवश्व बैंक की सिार्ता से
Operation Agency (JICA).
संचाक्तलत िै।
1. Rajasthan Water Sector Livelihood
– रेहगस्तान क्षेत्र के क्तलए जल क्षेत्र पयन: संरचना पररर्ोजना न्द्र्ू
Improvement Project.
डवलपमेंट बैंक की सिार्ता से संचाक्तलत िै। 2. Rajasthan Rural Water Supply and Fluorosis
– राजस्थान शिरी क्षेत्र हवकास कार्ुक्रम एक्तशर्न हवकास बैंक की Removal Project.
सिार्ता से संचाक्तलत िै। - The Dam Rehabilitation and Improvement
Project–II is operated with the assistance of the
World Bank.
- The Water Sector Restructuring Project for the
desert region is being operated with the
assistance of the New Bank.
- Rajasthan Urban Area Development Program is
operated with the assistance of Asian
Development Bank.
107. कनम्नलिखित कथनों पर कवचार कीजिए – 107. Consider the following statements -
I. काउन्न्द्सल फॉर इन्द्फ्रास्रक्चर डवलपमेंट (CID) की अध्र्क्षता I. The Council for Infrastructure Development
मयख्र्मंत्री द्वारा की जाती िै। (CID) is headed by the Chief Minister.
II. The Council for Infrastructure Development
II. काउन्न्द्सल फॉर इन्द्फ्रास्रक्चर डवलपमेंट (CID) उन
(CID) gives permission to projects that cost
पररर्ोजनाओं को अनयमहत प्रदान करती िै जजनकी लागत 100
more than 100 crores.
करोड़ से अधधक िो।
Which of the above statement(s) is/are false?
उपयुथक्त में कौन-सा/से कथन असत्य है/हैं? (a) I only
(a) केिि I (b) II only
(b) केिि II (c) Both I and II
(c) I और II दोनों (d) Neither I nor II
(d) न तो I, न ही II [b]
[b] Explanation -
व्याख्या – - The Council for Infrastructure Development (CID)
is headed by the Chief Minister.
– काउन्न्द्सल फॉर इन्द्फ्रास्रक्चर डवलपमेंट (CID) की अध्र्क्षता
– The Council for Infrastructure Development (CID)
मयख्र्मंत्री द्वारा की जाती िै।
gives permission to projects that cost more than
500 crores.

-:: 55 ::-
RPT 05
– काउन्न्द्सल फॉर इन्द्फ्रास्रक्चर डवलपमेंट (CID) उन पररर्ोजनाओं
को अनयमहत प्रदान करती िै जजनकी लागत 500 करोड़ से अधधक
िो।
108. ‘रािस्थान मध्यम नगरीय क्षेत्र कवकास कायुक्रम’ के संदभु में 108. With reference to the ‘Rajasthan Medium
कनम्नलिखित कथनों पर कवचार कीजिए - Urban Area Development Program,’ consider
I. र्ि पररर्ोजना एक्तशर्न हवकास बैंक (ADB) द्वारा हवत्त पोहषत the following statements -
I. The project is funded by the Asian
िै।
Development Bank (ADB).
II. इस पररर्ोजना के अंतगुत चर्हनत शिरों में हवद्ययतीकरण एवं
II. Under this project, electrification and traffic
र्ातार्ात प्रबंधन में सयधार हकर्ा जाता िै।
management are improved in selected cities.
III. इस पररर्ोजना में राजस्थान के 12 शिरों को शाधमल हकर्ा III. The project covers 12 cities of Rajasthan,
गर्ा िै, जजसमें कोई जजला मयख्र्ालर् शाधमल निीं िै। which do not include any district
उपयुथक्त में कौन-सा/से कथन सत्य है/हैं? headquarters.
(a) केिि I Which of the above statement(s) is/are true?
(b) केिि I और III (a) I only
(c) केवल II और III (b) I and III only
(d) उपर्युक्त सभी (c) II and III only
(d) All of the above
[a]
[a]
व्याख्या –
Explanation -
– राजस्थान मध्र्म नगरीर् क्षेत्र हवकास कार्ुक्रम एक्तशर्न हवकास बैंक
– The Rajasthan Medium Urban Area Development
(ADB) द्वारा हवत्त पोहषत िै। Program is funded by the Asian Development
– इस पररर्ोजना में राजस्थान के 14 शिरों को शाधमल हकर्ा गर्ा िै। Bank (ADB).
जजनमें क्तसरोिी तथा प्रतापगढ़ दो जजला मयख्र्ालर् शिर भी शाधमल – 14 cities of Rajasthan have been included in this
िै। project. In which Sirohi and Pratapgarh are also
– इस पररर्ोजना का उद्दे श्र् चर्हनत शिरों में जलापूर्ति और स्वच्छता two district headquarter cities.
में सयधार करना िै। अत: इस पररर्ोजना में जलापूर्ति प्रबंधन, मल, - The project aims to improve water supply and
कीचड़ से राित के कार्ु हकए जाते िैं। sanitation in selected cities. Therefore, the works
of water supply management, faecal sludge are
done in this project.
109. कनम्नलिखित कथनों पर कवचार कीजिए – 109. Consider the following statements -
I. वषु 2021–22 में राजस्थान का राजकोषीर् घाटा राज्र् सकल I. The fiscal deficit of Rajasthan in the year
घरेलू उत्पाद (SGDP) का 3.96% िै। 2021–22 is 3.96% of the State Gross Domestic
Product (SGDP).
II. राजस्थान को राजस्व प्राप्प्त का सवाुधधक भाग स्वर्ं के कर
II. Rajasthan receives most of its revenue from
राजस्व से प्राप्त िोता िै।
self-tax revenue.
उपयुथक्त में कौन-सा/से कथन सत्य है/हैं?
Which of the above statement(s) is/are true?
(a) केिि I (a) I only
(b) केिि II (b) II only
(c) I और II दोनों (c) Both I and II
(d) न तो I, न ही II (d) Neither I nor II
[c] [c]
व्याख्या – Explanation -
– वषु 2021–22 में राजस्थान का राजकोषीर् घाटा राज्र् सकल घरेलू – The fiscal deficit of Rajasthan in the year 2021–22
is 3.96% of the State Gross Domestic Product
उत्पाद (SGDP) का 3.96% िै।
(SGDP).
– राजस्थान को राजस्व प्राप्प्त का सवाुधधक भाग स्वर्ं के कर राजस्व
– Rajasthan receives the most part of its revenue
से प्राप्त िोता िै।
from self-tax revenue.
– राजस्थान की राजस्व प्राप्प्त के स्त्रोत् (घटते क्रम में) – – Sources of revenue receipt of Rajasthan (in
1. स्वर्ं का कर राजस्व decreasing order) –
2. केन्द्रीर् करों में हिस्सा 1. Own tax revenue

-:: 56 ::-
RPT 05
3. केन्द्रीर् सिार्ता 2. Share in Central Taxes
4. कर भभन्द्न राजस्व 3. Central Assistance
4. Non-Tax Revenue
110. कनम्नलिखित कथनों पर कवचार कीजिए - 110. Consider the following statements -
I. एम्पावडु कमेटी फॉर रोड सेक्टर प्रोजेक्ट (RHSDP) I. Empowered Committee for Road Sector
राजस्थान में स्टे ट िाइवे डवलपमेंट प्रोग्राम के अंतगुत सड़क Project (RHSDP) is a committee constituted
under the chairmanship of Chief Secretary
पररर्ोजनाओं की स्वीकृहत िेतय मयख्र् सधचव की अध्र्क्षता में
for approval of road projects under the State
गदठत सधमहत िै।
Highway Development Program in
II. सौर ऊजाु कंपनी ऑफ राजस्थान क्तलधमटे ड (SUCRL) का
Rajasthan.
गठन जैसलमेर में 750 मेगावाट ऊजाु के सौर पाकों के हवकास II. Solar Energy Company of Rajasthan Limited
के क्तलए हकर्ा गर्ा िै। (SUCRL) has been formed for the
उपयुथक्त में कौन-सा/से कथन सत्य है/हैं? development of 750 MW solar parks in
(a) केिि I Jaisalmer.
(b) केिि II Which of the above statement(s) is/are true?
(c) I और II दोनों (a) I only
(d) न तो I, न ही II (b) II only
(c) Both I and II
[a]
(d) Neither I nor II
व्याख्या –
[a]
– एम्पावडु कमेटी फॉर रोड सेक्टर प्रोजेक्ट (RHSDP) राजस्थान में
Explanation -
स्टे ट िाइवे डवलपमेंट प्रोग्राम के अंतगुत सड़क पररर्ोजनाओं की - Empowered Committee for Road Sector Project
स्वीकृहत िेतय मयख्र् सधचव की अधक्षता में गदठत सधमहत िै। (RHSDP) is a committee constituted under the
– सौर ऊजाु कंपनी ऑफ राजस्थान क्तलधमटे ड (SUCRL) का गठन chairmanship of the Chief Secretary for approval
भड़ला (जोधपयर) में 1000 मेगावाट ऊजाु के सौर पाकों के of road projects under the State Highway
चरणबद् हवकास िेतय वषु 2014 के क्तलए हकर्ा गर्ा िै। Development Program in Rajasthan.
- Solar Energy Company of Rajasthan Limited
(SUCRL) has been formed for the year 2014 for
phased development of 1000 MW solar parks in
Bhadla (Jodhpur).
111. ददए गए ग्राफ का अध्ययन करें और नीचे ददए गए प्रश्न का उत्तर 111. Study the given graph and answer the
दें । question given below.

विभाग E में कायथ करने िािे कमथचाररयों का 20% ज्ञात Find 20% of the employees working in
कीजिए। department E.
(a) 135 (a) 135
(b) 165 (b) 165
(c) 170 (c) 170
(d) 155 [d] (d) 155 [d]
व्याख्या:- Explanation:-
- कमयचाररर्ों की कुि संख्र्ा = 3000 - Total Number of employees = 3000
विभाग E में कार्य करने िािे कमयचाररर्ों की संख्र्ा = Number of employees working in Department E

-:: 57 ::-
RPT 05
93 93
× 3000 = 775  3000 = 775
360 360
20
20 775  = 155
775 × = 155 100
100
112. ददए गए ग्राफ का अध्ययन करें और नीचे ददए गए प्रश्न का उत्तर 112. Study the given graph and answer the
दें । question given below.

विभाग B में कमथचाररयों की संख्या, विभाग D और E में कायथ The number of employees in department B is
करने िािे कमथचाररयों की संख्या का ककतना प्रवतशत है? what percent of the number of employees
(a) 45.8 % working in departments D and E?
(b) 48.6 % (a) 45.8 %
(c) 50.4 % (b) 48.6 %
(d) 49.2 % [d] (c) 50.4 %
व्याख्या:-
(d) 49.2 % [d]
विभाग B में कमयचाररर्ों की संख्र्ा, विभाग D और E में कार्य करने
Explanation:-
िािे कमयचाररर्ों की कुि संख्र्ा का प्रवतशत
Number of employees in department B,
73.8
=  100 percentage of total number of employees
(57 + 93 ) working in department D and E
73.8 73.8
=  100 = 49.2% =  100
150 ( + 93 )
57

73.8
= 100 = 49.2%
150
113. ददए गए ग्राफ का अध्ययन करें और नीचे ददए गए प्रश्न का उत्तर 113 Study the given graph and answer the
दें । question given below.

विभाग A और C में कायथ करने िािे कमथचाररयों की कुि


The total number of employees working in
संख्या, विभाग B ि D में कायथ करने िािे कमथचाररयों की कुि departments A and C exceeds the total
संख्या से x अधधक है। x का मान वकसके बीच होगा? number, of employees working in

-:: 58 ::-
RPT 05
(a) 36 से 44 departments B and D by x. The value of x lies
(b) 44 से 52 between:

(c) 28 से 36 (a) 36 to 44
(b) 44 to 52
(d) 20 से 28 [b]
(c) 28 to 36
व्याख्या :-
(d) 20 to 28 [b]
विभाग A ि C में कार्यरत कमयचाररर्ों की कुि संख्र्ा =
Explanation :-
64.2 + 72 = 136.2 The total number of employees working in
विभाग B ि D में कार्यरत कमयचाररर्ों की कुि संख्र्ा =
departments A and C = 64.2 + 72 = 136.2
  

73.8 + 57 = 130.8 The total number of employees working in


(64.2 + 72 ) − (73.8 + 57 )
x=  3000 departments B and D = 73.8 + 57 = 130.8
  

360
(64.2° + 72° ) - (73.8° + 57° )
136.2 − 130.8 x= × 3000
=  3000 360°
360
136.2° -130.8°
5.4 = × 3000
=  3000 = 45 360°
360
5.4
=  3000 = 45
360
114. वनम्नांवकत बार ग्राफ का अध्ययन कीजजए और उसके बाद ददए 114. Study the following bar graph and answer the
गए प्रश्न का उत्तर दीजजए- questions given below -

स्कूि A, B, C, D और E में िडकों और िडवकयों की कुि Total number of boys and girls in schools A, B,
C, D and E.
संख्या

स्कूिों A, B, C, D और E में िडकों और िडवकयों की संख्या Difference between the number of boys and
girls in schools A, B, C, D and E.
में अंतर

If the number of boys is more than that of


girls, then the number of boys in school B is

-:: 59 ::-
RPT 05
यठद िड़कों की संख्या िड़ककयों से ज्यादा हो तो स्कूि B में what percent of the total number of students
िडकों की संख्या, उस स्कूि में विद्यार्थियों की कुि संख्या की in that school?

वकतने प्रवतशत है? (a) 50%


(b) 55%
(a) 50%
(c) 40%
(b) 55%
(d) 60% [d]
(c) 40%
Explanation:-
(d) 60% [d] Total number of students in school B= 2600
व्याख्या:- Difference between the number of boys and girls
थकूि B में विद्यार्ििंर्ों की कुि संख्र्ा = 2600 in school B= 520
थकूि B में िड़कों और िड़वकर्ों की संख्र्ा में अंतर = 520 B+ G= 2600
B + G = 2600 B-G= 520
B - G = 520 2B= 3120
2B = 3120 B= 1560
B= 1560 Number of boys in school B= 1560
िड़कों की संख्र्ा = 1560 Required percentage = 1560 ×100 = 60%
अभीि प्रवतशत = 2600
×100 = 60%
1560
2600
115. वनम्नांवकत बार ग्राफ का अध्ययन कीजजए और उसके बाद ददए 115. Study the following bar graph and answer the
गए प्रश्न का उत्तर दीजजए- questions given below.

स्कूि A, B, C, D और E में िडकों और िडवकयों की कुि Total number of boys and girls in schools A, B,
C, D and E.
संख्या

Difference between the number of boys and


स्कूिों A, B, C, D और E में िडकों और िडवकयों की संख्या girls in schools A, B, C, D and E.
में अंतर

What is the ratio of number of boys to the


स्कूि E में िडकों की संख्या का, िडवकयों की संख्या से number of girls in School E?
अनुपात वकतना है? (a) 5:3

-:: 60 ::-
RPT 05
(a) 5:3 (b) 4:3
(b) 4:3 (c) 5:4

(c) 5:4 (d) 7:4 [a]


Explanation:-
(d) 7:4 [a]
School E: →
व्याख्या:-
B→Boys
School E: =
G→Girls
B = Boys
B+ G= 2800
G = Girls
B-G= 700
B+ G= 2800
B + G 2800
B-G= 700 =
B+G 2800
B-G 700
= B+G 4
B-G 700 =
B+G 4 B-G 1
=
(B +G)+ (B - G) 4 +1 a +b
B-G 1
=  a − b 
र्ोगांतरानुपात िगाने पर (B +G)- (B - G) 4 -1
(B + G) + (B - G) 4 +1  a + b  2B 5
=
(B + G) - (B - G) 4 -1  a - b 
=
2G 3
B: G= 5:3
B: G= 5:3
2B 5
=
2G 3
116. वनम्नांवकत बार ग्राफ का अध्ययन कीजजए और उसके बाद ददए 116. Study the following bar graph and answer the
गए प्रश्न का उत्तर दीजजए- questions given below.

स्कूि A, B, C, D और E में िडकों और िडवकयों की कुि Total number of boys and girls in schools A, B,
C, D and E.
संख्या

Difference between the number of boys and


स्कूिों A, B, C, D और E में िडकों और िडवकयों की संख्या
girls in schools A, B, C, D and E.
में अंतर

स्कूि A में िडवकयों की संख्या और स्कूि C में िडवकयों की


संख्या में वकतना अंतर है?
-:: 61 ::-
RPT 05
(a) 25 What is the difference between the number of
(b) 20 girls in school A and the number of girls in

(c) 30 school C?
(a) 25
(d) 35 [a]
(b) 20
व्याख्या:-
(c) 30
School A:-
(d) 35 [a]
B+G= 1800 (1)
व्याख्या:-
B-G= 350 (2)
समी. (1) में से समी. (2) र्िाने पर School A:-
B+G- (B-G) = 1800-350 B+G= 1800 (1)
2G= 1450 B-G= 350 (2)
G= 725 By subtracting eq. (2) from eq (1)
थकूि A में िड़वकर्ों की संख्र्ा = 725 B+G- (B-G)= 1800-350
School C:-
2G= 1450
B+G= 2000 (3)
G= 725
B-G= 500 (4)
Number of girls in school A= 725
समी. (3) में से समी. (4) र्िाने पर
(B+G)- (B-G) = 2000-500 School C:-
2G= 1500 B+G= 2000 (3)
G= 750 B-G= 500 (4)
थकूि C में िड़वकर्ों की संख्र्ा = 750 By subtracting eq. (4) from eq (3)
अभीि अंतर = 750-725 = 25 (B+G)- (B-G)= 2000-500
2G= 1500
G= 750
Number of girls in school C= 750
Required difference = 750-725 = 25
117. पाई चाटथ का अध्ययन करें और प्रश्न का उत्तर दें - 117. Study the pie-chart and answer the options:

E में विशेषज्ञता प्राप्त छारों की संख्या, c में विशेषज्ञता प्राप्त The number of students specialising in E is
छारों की तुिना में वकतने प्रवतशत अधधक है? (छारों की कुि what per cent more than that of students
specialising in C? (Total number of students =
संख्या = 2100)
2100)
(a) 25.9% (a) 25.9%
(b) 32% (b) 32%
(c) 35% (c) 35%
(d) 30.4% [c] (d) 30.4% [c]
व्याख्या:- Explanation:-
E में विशेषज्ञता प्राप्त छात्रों की संख्र्ा, C में विशेषज्ञता प्राप्त छात्रों The number of students specializing in E is
percent more than that of students specializing
की तुिना में अधधक है।
in C
-:: 62 ::-
RPT 05
= 97.2 - 72 ×100 =
97.2 - 72
× 100
72 72
25.2
= ×100 = 35 % =
25.2
×100 = 35%
72 72
118. वनम्नांवकत पाई चाटथ का अध्ययन कीजजए और उसके बाद ददए 118. Study the following pie-chart and answer the
गए प्रश्न का उत्तर दीजजए– questions given below -

एक विश्वविद्यािय में विक्षभन्त्न संकायों में प्रिेश िेने िािे Total number of students admitted in a
university in various fields 5000. Distribution
विद्यार्थियों की संख्या 5000 है। विक्षभन्त्न संकायों में विद्यार्थियों
of the number of students into various fields -
की संख्या का बंटन -

The ratio of the number of boys in Economics


to the number of students in Economics is:
इकोनॉधमक्स में िडकों की संख्या का, इकोनॉधमक्स में कुि
(a) 13 : 25
विद्यार्थियों की संख्या से क्या अनुपात है?
(b) 14:25
(a) 13 : 25 (c) 12:25
(b) 14:25 (d) 17:25 [b]
(c) 12:25 Explanation :-
(d) 17:25 [b] - Total Students in Economics = 12% of 5000
व्याख्या :- 5000 × 12
= = 600
- इकोनॉधमक्स में कुि विद्यािी = 5000 का 12% 100
5000 ×12 No. of Boys in Economics = 56% of 600 = 336
= = 600
100 According to question,
इकोनॉधमक्स में िड़कों की संख्र्ा = 600 का 56% = 336 336 : 600
प्रश्नानुसार, 56 : 100
336 : 600 14 : 25
56 : 100
14 : 25
119. वनम्नांवकत पाई चाटथ का अध्ययन कीजजए और उसके बाद ददए 119. Study the following pie-chart and answer the
गए प्रश्न का उत्तर दीजजए– questions given below.
Total number of students admitted in a
एक विश्वविद्यािय में विक्षभन्त्न संकायों में प्रिेश िेने िािे
university in various fields = 5000. Distribution
विद्यार्थियों की संख्या 5000 है। विक्षभन्त्न संकायों में विद्यार्थियों of the number of students into various fields -
की संख्या का बंटन -

-:: 63 ::-
RPT 05
What is the difference between the number of
girls in IT and number of girls in ECE?
(a) 25
(b) 21
(c) 20
(d) 30 [b]
Explanation :-
- Total No. of Students in IT = 14% of 5000
5000 × 14
IT में िडवकयों की संख्या और ECE में िडवकयों की संख्या में = = 700
100
वकतना अंतर है? No. of Girls in IT = (100-65)% of 700= 35% of 700
(a) 25 700 × 35
= = 245
(b) 21 100
(c) 20 Total Students in ECE = 16% of 5000
(d) 30 [b] 5000 × 16
= = 800
100
व्याख्या :-
No. of Girls in ECE = (100-72)% of 800
- IT में कुि विद्यािी = 5000 का 14% =
5000 ×14
= 700 = 28% of 800
100 800 × 28
IT में िड़वकर्ों की संख्र्ा = 700 का (100-65)% = 700 का = = 224
100
700 × 35 Difference = 245 – 224 = 21
35% = = 245
100
5000 ×16
ECE में कुि विद्यािी = 5000 का 16% = = 800
100
ECE में िड़वकर्ों की संख्र्ा = 800का(100-72)%
800 × 28
= 800 का 28% = = 224
100
अन्तर = 245 – 224 = 21

120. वनदे श:- नीचे दी गई जानकारी को ध्यानपूिथक पढकर उत्तर 120. Direction: Read the information given below
दीजजए- carefully and answer-
एक फमथ के उत्पादन की कुि िागत 250 िाि रुपए है। The total cost of production of a firm is Rs. 250
lakhs. Is. The following pie chart shows the
वनम्नलिखित पाई चाटथ में विक्षभन्त्न प्रयोजनों में उत्पादन िागत
percentage of production cost in different
की प्रवतशतता दशाथयी गई है –
purposes-
कच्चे माि की िागत Raw material cost

40%
cost of packing
40%
पैबकिग सामग्री की 25% 5% material
25%
िागत विज्ञापन की िागत 5%
30% advertising cost
30%

श्रम की िागत
labor cost

पैबकिग सामग्री और कच्चे माि की कुि िागत वकतनी है?


What is the total cost of packing material and raw
(a) 150 िाख रुपए material?
(b) 157.5 िाख रुपए (a) Rs.150 lakh
(c) 160 िाख रुपए (b) Rs 157.5 lakh

-:: 64 ::-
RPT 05
(d) 162.5 िाख रुपए [d] (c) Rs.160 lakh
व्याख्या – (d) Rs 162.5 lakh [d]
िमय के उत्पादन की कुि िागत = 250 िाख रुपए Explanation -
Total cost of production of the firm = 250 lakhs
पैककिंग सामग्री और कच्चे माि की कुि िागत का प्रवतशत
Packing material and percentage of total cost of
= 25 + 40 = 65%
raw material = 25 + 40 = 65%
पैककिंग सामग्री और कच्चे माि की कुि िागत = 250 का 65%
Total cost of packing material and raw material=
= 250 × 65 = 162.5 िाख रुपए 65% of 250
100
= 250 × 65 = Rs 162.5 lakh.
100
121. नीचे ददए गए प्रत्येक प्रश्न में पहिे एक कथन है वफर उसके नीचे 121. In each of the questions given below are a
कायथिाही के तीन उपाय I, II और III ददए गए हैं। कथन में दी statement followed by three course of action
गई जानकारी के आधार पर आपको कथन में दी गई प्रत्येक numbered I, II and III. On the basis of the
information given in the statement, you have
बात को सही मानना है। और उसके बाद तय करना है वक ददए
to assume everything given in the statement
गए सुझाि में से कौनसी कायथिाही तकथ संगत रूप से अनुसरण
to be true. And then decide which of the given
करती है।
suggestions logically follows the course of
कथन: action.
हर िषय मानूसन के दौरान शहर की अधधकांश सड़कों की हाित Statement:
खराब हो जाती है जजससे र्ावत्रर्ों को भारी समथर्ा होती है। Every year during the monsoon, most of the
कायथिावहयााँ : roads in the city are in bad condition causing a lot
I. नगर वनकार् को भविष्र् में सड़कों की मरम्मत के ठे के दे ते of problems to the commuters.
समर् भारी जुमायने की शतय शाधमि करनी चावहए। Course of action:
II. शहर की सड़कों के रख-रखाि के प्रभारी नगर वनकार् I. The civic body should include a clause of
heavy penalty while awarding contracts for
अधधकाररर्ों से थपिीकरर् मााँगा जाना चावहए वक हर साि
repairing roads in future.
सड़कों की हाित खराब क्र्ों हो जाती है।
II. An explanation should be sought from the
III. मानसून के दौरान आम जनता को अपने िाहन सड़क पर िे
municipal officials in charge of the
जाने से बचना चावहए। maintenance of the city roads as to why the
(a) केिि I अनुसरर् करता है। condition of the roads deteriorates every
(b) केिि II अनुसरर् करता है। year.
(c) केिि I और II अनुसरर् करते िैं। III. The general public should avoid taking their
(d) केिि II और III अनुसरर् करते है। [c] vehicles on the road during monsoon.
व्याख्या :- (a) Only I follows.
- ठे के दे ते समर् जुमायने की शतय शाधमि करने पर ठे केदार सही काम (b) Only II follows.
(c) Only I and II follow.
करेंगे। नगर वनकार् अधधकाररर्ों से थपिीकरर् से िे सड़कों की
(d) Only II and III follow. [c]
हाित पर ध्र्ान रखेंगे। अतः केिि I और II कार्यिावहर्ााँ उधचत िैं
Explanation :-
तिा कार्यिाही III महत्िहीन है।
- The contractors will do the right thing if the
condition of penalty is included while awarding
the contracts. With the clarification from the
municipal officials, they will take care of the
condition of the roads. Hence only courses of
action I and II are justified and course of action
III is unimportant.
122. नीचे ददए गए प्रत्येक प्रश्न में पहिे एक कथन है वफर उसके नीचे 122. In each of the questions given below are a
कायथिाही के तीन उपाय I, II और III ददए गए हैं। कथन में दी statement followed by three course of action
गई जानकारी के आधार पर आपको कथन में दी गई प्रत्येक numbered I, II and III. On the basis of the
information given in the statement, you have
बात को सही मानना है। और उसके बाद तय करना है वक ददए
to assume everything given in the statement
गए सुझाि में से कौनसी कायथिाही तकथ संगत रूप से अनुसरण
to be true. And then decide which of the given
करती है।

-:: 65 ::-
RPT 05
कथन: suggestions logically follows the course of
साियजवनक क्षेत्र के कई उपिम वपछिे िषों से र्ािा उठा रहे हैं और Action.
चािू िषय में भी स्थिवत उतनी ही खराब है। Statement: Many Public Sector Undertakings
have been incurring losses since previous years
कायथिावहयााँ :
and the situation is equally bad in the current
I. र्ािा उठाने िािे साियजवनक क्षेत्र की इन कम्पवनर्ों को तत्काि
year as well.
बंद कर टदर्ा जाना चावहए।
Course of action:
II. सरकार को इन कम्पवनर्ों को बेचने के लिए वनजी क्षेत्र में I. These loss making public sector companies
सम्भािनाशीि खरीददार ढूाँ ढना चावहए तावक सरकार द्वारा should be shut down immediately.
वकए गए वनिेश का कुछ हिस्सा िापस धमि जाए। II. The government should find potential
III. इन कम्पवनर्ों के सभी कमयचारर्ों को उधचत मुआिजा दे कर buyers in the private sector to sell these
छिनी कर दी जानी चावहए और अचि आस्थतर्ों को वबिी के companies so that some part of the
लिए रख दे ना चावहए। investment made by the government is
(a) केिि I और II अनुसरर् करता है। recovered.
III. All the employees of these companies should
(b) केिि II अनुसरर् करता है।
be retrenched by giving proper
(c) केिि III अनुसरर् करता है।
compensation and the fixed assets should be
(d) केिि I, II और III अनुसरर् करते िैं। [b]
put up for sale.
व्याख्या :- (a) Only I and II follow.
केिि कार्यिाही II अनुसरर् करती है, क्र्ोंवक र्ािे के कारर् कुछ (b) Only II follows.
भाग भी धमि जाए िही अच्छा है। (c) Only III follows.
(d) Only I, II and III follow. [b]
Explanation :-
- Only action II follows, because it is better to get
some part due to loss.
123. ददए गए कथनों और वनष्कषों को ध्यानपूिथक पदढए । यह 123. Read the given statements and conclusions
मानते हुए वक कथनों में दी गई जानकारी सत्य है, भिे ही िह carefully. Assuming that the information
सामान्त्य रूप से ज्ञात तथ्यों से क्षभन्त्न प्रतीत होती हो, तय given in the statements is true even if it seems
to be at variance from commonly known facts,
कीजजए वक ददए गए वनष्कषों में से कौन से वनष्कषथ कथनों का
decide which of the given conclusions
तार्किक रूप से अनुसरण करते हैं।
logically follows from the statements.
कथन :-
Statement :-
उच्च थतरीर् वनमायर् कार्य हेतु सक्षम अनुभिी ठे केदारों से मुहरबन्द Sealed tenders are invited from capable
वनविदाएाँ आमन्न्त्रत की जाती हैं। experienced contractors for high level
वनष्कषथ construction work.
I. वनविदाएाँ केिि सक्षम अनुभिी ठे केदारों से आमन्न्त्रत की Conclusion
जाती हैं। I. Tenders are invited from competent
II. वनमायर् कार्य की वनविदाओं में सक्षम व्यलि को ज्ञात करना experienced contractors only.
कटठन है। II. It is difficult to find competent person in
tenders for construction work.
(a) केिि वनष्कषय I वनकिता है।
(a) Only conclusion I follows.
(b) केिि वनष्कषय II वनकिता है।
(b) Only conclusion II follows.
(c) वनष्कषय I और वनष्कषय II दोनों वनकिते िैं।
(c) Both conclusion I and conclusion II follow.
(d) न तो वनष्कषय I और न ही II वनकिता है। (d) Neither conclusion I nor II follows. [a]
[a] Explanation:-
व्याख्या:- - Only conclusion I. is logical, since tenders are
- केिि वनष्कषय I तकयसंगत है, क्र्ोंवक वनविदाएाँ केिि सक्षम invited from competent experienced contractors
अनुभिी ठे केदारों से ही आमन्न्त्रत की जाती है अत: वनमायर् कार्य only, so it is not difficult to find out competent
की वनविदाओं में सक्षम व्यलि को ज्ञात करना कटठन नहीं है। person in tendering for construction work.
124. ददए गए कथनों और वनष्कषों को ध्यानपूिथक पदढए। यह मानते 124. Read the given statements and conclusions
हुए वक कथनों में दी गई जानकारी सत्य है, भिे ही िह सामान्त्य carefully. Assuming that the information
given in the statements is true even if it seems
-:: 66 ::-
RPT 05
रूप से ज्ञात तथ्यों से क्षभन्त्न प्रतीत होती हो, तय कीजजए वक to be at variance from commonly known facts,
ददए गए वनष्कषों में से कौन से वनष्कषथ कथनों का तार्किक रूप decide which of the given conclusions
से अनुसरण करते हैं। logically follows from the statements.
Statement:-
कथन:-
All those political prisoners were released on bail
उन सभी राजनीवतक बजन्दर्ों को जमानत पर छोड़ टदर्ा गर्ा जो
who went to jail for reasons other than political
वक राजनीवतक र्िनाओं के अिािा अन्र् कारर्ों से जेि गर्े िे।
incidents. Bail was denied to persons who were
िैसे व्यलिर्ों की जमानत नामन्जूर कर दी गई िी जो वक हत्र्ा में involved in the murder.
शाधमि िे। Conclusion:-
वनष्कषथ:- I. None of the political prisoners had
I. वकसी भी राजनीवतक बन्दी ने हत्र्ा नहीं की िी। committed the murder.
II. कुछ राजनीवतज्ञों को वगरफ्तार नहीं वकर्ा गर्ा िा। II. Some politicians were not arrested.
(a) केिि वनष्कषय I वनकिता है। (a) Only conclusion I follows.
(b) केिि वनष्कषय II वनकिता है। (b) Only conclusion II follows.
(c) Both conclusion I and conclusion II follow.
(c) वनष्कषय I और वनष्कषय II दोनों वनकिते िैं।
(d) Neither conclusion I nor II follows. [b]
(d) न तो वनष्कषय I और न ही II वनकिता है।
Explanation:-
[b]
- Only conclusion II. Logically, because all those
व्याख्या:- political prisoners who were jailed for reasons
- केिि वनष्कषय II तकयसंगत है, क्र्ोंवक उन सभी राजनीवतक बजन्दर्ों other than political incidents were released on
को जमानत पर छोड़ टदर्ा गर्ा जो वक राजनीवतक र्िनाओं के bail, that is, some politicians were not arrested,
अिािा अन्र् कारणों से जेि गए िे अिायत् कुछ राजनीवतज्ञों को while whether any political prisoner had
वगरफ्तार नहीं वकर्ा गर्ा िा, जबवक वकसी भी राजनीवतक बंदी ने committed murder or not - it does not matter. is
हत्र्ा की िी र्ा नहीं- इस बात की चचाय किन में नहीं की गई है। not discussed in the statement.
125. ददए गए कथनों पर विचार करें और वनणथय िें वक दी गई 125. Consider the given statement and decide
धारणाओं में से कौन-सी धारणा वनवहत हैं? which of the given assumptions is/are implicit
कथन:- in the statement?
Statement-
र्टद िोग बुजद्धमान हों, तो उन्हें सृजनशीि होना चावहए।
If people are intelligent, they must be creative.
धारणाएाँ :
Assumption:-
I. सृजनशीिता और बुजद्धमत्ता परथपर सम्बस्न्धत है।
I. Creativity and intelligence are interrelated.
II. सृजनशीि िोग बुजद्धमान होते हैं। II. Creative people are intelligent.
(a) केिि I मान्र् है। (a) Only I is valid.
(b) केिि II मान्र् है। (b) Only II is valid.
(c) दोनों I और II मान्र् हैं (c) Both I and II are valid
(d) दोनों I और II अमान्र् है। [a] (d) Both I and II are false. [a]
व्याख्या :- Explanation:-
- किन से थपि है वक सृजनशीिता और बुजद्धमता एक दूसरे से - It is clear from the statement that creativity and
intelligence are related to each other.
सम्बस्न्धत है।
126. ददए गए कथनों पर विचार करें और वनणथय िें वक दी गई 126. Consider the given statement and decide
धारणाओं में से कौन-सी धारणा वनवहत हैं? which of the given assumptions is/are implicit
कथन:- in the statement?
Statement-
अधधकांश ग्रामिासी अलशणक्षत हैं और इसलिए अंधविश्वासी िैं।
Most of the villagers are illiterate and therefore
धारणाएाँ :
superstitious.
I. लशक्षा से तकयसंगत धचन्तन में िृजद्ध होती है।
Assumption:-
II. ग्रामिासी विद्यािर् नहीं जाते। I. Education increases rational thinking.
(a) केिि I अन्तर्निंवहत है। II. The villagers do not go to school.
(b) केिि II अन्तर्निंवहत है। (a) Only I is implicit.
(c) I तिा II दोनों अन्तर्निंवहत है। (b) Only II is implicit.
(d) न I अन्तर्निंवहत है और न ही II [b] (c) Both I and II are implicit.

-:: 67 ::-
RPT 05
व्याख्या :- (d) Neither I nor II is implicit [b]
- पूिायनुमान II सही है क्र्ोंवक ग्रामिासी विद्यािर् नहीं जाते इसलिए Explanation :-
अलशणक्षत िैं। - Assumption II is correct as the villagers do not go
to school and hence are illiterate.
127. वनम्नलिखित कथन को पदढए और जिाब दीजजए 127. Read the following statements and answer
वनम्नलिखित में कौन-सा/से तकथ िोस है? the following Which of the following is/are the
कथन- arguments Strong?
Statement-
क्र्ा भारतीर् विल्मों से गानों को हिार्ा जाना चावहए?
Should songs be removed from Indian films?
तकथ-
Argument –
I. िााँ, हॉिीिुड की विल्में कोई गाने न होने के बािजूद वहि होती
I. Yes, Hollywood movies hit despite having no
हैं। songs there are.
II. नहीं, गाने विल्म की िंबाई बढाने में मदद करते है। II. No, the songs help in increasing the length of
(a) केिि तकय I ठोस है। the film.
(b) केिि तकय II ठोस है। (a) Only argument I is strong.
(c) दोनों तकय I ि तकय II ठोस हैं। (b) Only argument II is strong.
(d) न तो तकय I, ना ही तकय II ठोस है। [d] (c) Argument I and Argument II are strong.
व्याख्या – (d) Neither argument I nor argument II is strong.
[d]
भारतीर् विल्मों और हॉिीिुड की विल्मों दोनों की पृष्ठभूधम अिग-
Explanation -
अिग है। अत: र्ह आिश्र्क नहीं है वक भारतीर् विल्में वबना गानों
Both Indian films and Hollywood films have
के ही बनाई जाए। इस प्रकार तकय I मजबूत नहीं है, साि ही ऐसा
different backgrounds. Therefore, it is not
भी नहीं है वक गानें विल्मों की िंबाई बढाते है। इसलिए तकय II भी necessary that Indian films should be made
मजबूत नहीं है। अत: न तो तकय I और न ही तकय II मजबूत है। without songs. Thus Argument I is not strong,
also it is not that songs increase the length of
movies. Hence argument II is also not strong.
Hence, neither argument I nor argument II is
strong.
128. वनम्नलिखित कथन को पदढए और जिाब दीजजए 128. Read the following statements and answer
वनम्नलिखित में कौन- सा/से तकथ िोस है? the following Which of the following is/are the
कथन: arguments Strong?
Statement :
क्र्ा भारत में तंबाकू के सारे उत्पादों पर रोक िगा दे नी चावहए?
Should all tobacco products be banned in India.
तकथ-
Argument :
I. नहीं, इससे िोग बड़ी संख्र्ा में बेरोजगार हो जार्ेंगे।
I. No, in large number people will become
II. नहीं, इससे सरकार को भारी रालश की हावन होगी क्र्ोंवक िह unemployed
इन उत्पादों से प्राप्त होने िािे करों को अर्जिंत नहीं कर सकेगी। II. No, the government will lose huge amount of
सही विकल्प चुवनए- money as it will not earn by taxes on these
(a) केिि तकय I ठोस है। products.
(b) केिि तकय II ठोस है। Choose the correct option.
(c) दोनों तकय I ि तकय II ठोस हैं। (a) Only argument I is strong.
(d) न तो तकय I, ना ही तकय II ठोस है। [c] (b) Only argument II is strong.
(c) Argument I and Argument II are strong.
व्याख्या –
(d) Neither argument I nor argument II is strong
दोनों तकय ठोस है क्र्ोंवक बहुत सारे िोग इस रोजगार में िैं और
[c]
इससे सरकार को कािी कर प्राप्त होता है।
Explanation -
Both arguments are strong because by this many
people are employed and by this government
collects a lot of tax.
129. वनम्नलिखित कथन को पदढए और जिाब दीजजए 129. Read the following statements and answer
वनम्नलिखित में कौन- सा/से तकथ िोस है? the following Which of the following is/are the
arguments Strong?
-:: 68 ::-
RPT 05
कथन: Statement:
क्र्ा भारत को केिि परमार्ु ऊजाय पर भरोसा करना चावहए? Should India rely only on nuclear energy?
तकथ: Arguments:
I. Yes, the demand for electricity cannot be met
I. हां, वबजिी की मााँग को मौजूदा संसाधनों के साि पूरा नहीं
with the existing resources.
वकर्ा जा सकता है।
II. No, if there is an accident, there will be huge
II. नहीं, अगर कोई दुर्यिना होती है, तो भारी नुकसान हो जाएगा।
loss.
(a) केिि तकय I ठोस है। (a) Only argument I is strong.
(b) केिि तकय II ठोस है। (b) Only argument II is strong.
(c) दोनों तकय I ि तकय II ठोस हैं। (c) Argument I and Argument II are strong.
(d) न तो तकय I ना ही तकय II ठोस है। [d] (d) Neither argument I nor argument II is strong
व्याख्या – [d]
- तकय I मजबूत नहीं है, क्र्ोंवक र्ह कहना वक वबजिी की पूर्तिं मौजूदा Explanation -
संसाधनों से पूरा नहीं वकर्ा जा सकता है, र्ह सही नहीं है, क्र्ोंवक - Argument I is not strong as saying that electricity
cannot be met from existing resources is not
भारत परमार्ु ऊजाय से बहुत कम वबजिी उत्पन्न करता है तिा तकय
correct as India generates very little electricity
II भी मजबूत नहीं है, क्र्ोंवक दुर्यिना को ध्र्ान में रखकर परमार्ु
from nuclear energy and Argument II is also not
ऊजाय का प्रर्ोग नहीं वकर्ा जाता है इसलिए र्ह भी सही नहीं है।
strong as the accident is Considering that nuclear
अतः न तो तकय I और न ही तकय II मजबूत है। energy is not used, it is also not correct. Hence
neither argument I nor argument II is strong.
130. वनम्नलिखित कथन को पदढए और जिाब दीजजए 130. Read the following statements and answer
वनम्नलिखित में कौन- सा/से तकथ िोस है? the following Which of the following is/are the
कथन: arguments Strong?
Statement:
भारतीर्ों की जीिन प्रत्र्ाशा बढ रही है।
Life expectancy of Indians is increasing.
तकथ:
Argument:
I. हााँ, िोगों को बेहतर धचवकत्सा सुविधाएाँ धमि रही है।
I. Yes, People are getting better medical
II. हााँ, िोग अब अधधक शारीररक व्यार्ाम कर रहे हैं। facilities.
(a) केिि तकय I ठोस है। II. Yes, People are doing more physical exercise
(b) केिि तकय II ठोस है। now.
(c) दोनों तकय I ि तकय II ठोस हैं। (a) Only argument I is strong.
(d) न तो तकय I, ना ही तकय II ठोस है। [c] (b) Only argument II is strong.
व्याख्या :- (c) Argument I and Argument II are strong.
- किनानुसार तकय (i) और (ii) दोनों अनुसरर् करते हैं। (d) Neither argument I nor argument II is strong
[c]
Explanation :-
- From the statement both argument (i) and (ii)
follow.
131. हाि ही में, ककस मंठदर को पयुटन स्थि का दिाु दे कर कवकास 131. Recently, an amount of Rs 12 crore was
कायो एवं म्यूजियम के लिए 12 करोड़ रुपये की रालश की announced for the development work and
घोषणा की गई? museum by giving the status of tourist spot to
which temple?
(a) अथूणाु
(a) Arthuna
(b) नानेरा
(b) Nanera
(c) सावंतसर
(c) Sawantsar
(d) सांसेरा–रेलमगरा (d) Sansera–Railmagra
[d] [d]
व्याख्या – Explanation -
िाल िी में, पर्ुटन स्थल जलदे वी माताजी मंददर, सांसेरा – रेलमगरा Recently, an amount of Rs 12 crore has been
नाथद्वारा, राजसमंद को पर्ुटन स्थल का दजाु दे कर हवकास कार्ो announced for the development works and
museum by giving the status of tourist

-:: 69 ::-
RPT 05
एवं म्र्ूजजर्म के क्तलए 12 करोड़ रुपर्े की राक्तश की घोषणा की गई destination to Jaldevi Mataji Temple, Sansera –
िै। Railmagra Nathdwara, Rajsamand.
132. सूची-I का सूची–II से धमिान कीजिए – 132. Match List–I with List–II –
सूची-I सूची-II List-I List-II
A. हवश्व तंबाकू हनषेध ददवस I. 1 मई A. World No Tobacco Day I. May 1
B. Commonwealth Day II. May 11
B. राष्ट्रमडडल ददवस II. 11 मई
C. International Labor Day III. May 24
C. अन्द्तरराष्ट्रीर् श्रम ददवस III. 24 मई
D. National Technology Day IV. May 31
D. राष्ट्रीर् प्रौद्योहगकी ददवस IV. 31 मई
Code:
कूट: (a) A- I B- II C- III D- IV
(a) A-I B-II C-III D-IV (b) A- IV B- II C- III D- I
(b) A-IV B-II C-III D-I (c) A- IV B- III C- I D- II
(c) A-IV B-III C-I D-II (d) A-III B- IV C- II D- I
(d) A-III B-IV C-II D-I [c]
[c] Explanation -
व्याख्या – World no tobacco day - 31 may
Commonwealth Day - 24 May
हवश्व तंबाकू हनषेध ददवस 31 मई
International Labor Day -1 May
राष्ट्रमडडल ददवस 24 मई
National Technology Day - 11 May
अन्द्तरराष्ट्रीर् श्रम ददवस 1 मई
राष्ट्रीर् प्रौद्योहगकी ददवस 11 मई
133. ककन दो दे शों ने प्रधानमंत्री नरेन्द्र मोदी को अपने सवोच्च 133. Which two countries have honored Prime
नागररक सम्मानों से सम्माकनत ककया है? Minister Narendra Modi with their highest
(a) हफजी और गयर्ाना civilian honours?
(a) Fiji and French Guyana
(b) हफजी और हगनी
(b) Fiji and Guinea
(c) फ्रेंच गयर्ाना और पापयआ न्द्र्ू हगनी
(c) French Guyana and Papua New Guinea
(d) हफजी और पापयआ न्द्र्ू हगनी
(d) Fiji and Papua New Guinea
[d] [d]
व्याख्या – Explanation -
– प्रधानमंत्री नरेन्द्र मोदी को ‘हफजी’ और ‘पापयआ न्द्र्ू हगनी’ दे शों ने - Fiji and Papua New Guinea have honored Prime
वैभश्वक नेतृत्व के क्तलए अपने सवोच्च नागररक सम्मानों से सम्माहनत Minister Narendra Modi with their highest
हकर्ा िै। civilian honors for global leadership.
– हफजी का सवोच्च नागररक सम्मान – ‘कम्पेहनर्न ऑफ द ऑडुर - Fiji's highest civilian award – ‘The Companion of
ऑफ हफजी’। the order of Fiji’.
– पापयआ न्द्र्ू हगनी का सवोच्च नागररक सम्मान – ‘द ग्रैंड कम्पेहनर्न – Highest civilian honor of Papua New Guinea –
‘Grand Companion of the order of logehv’.
ऑफ द ऑडुर ऑफ लोगोहु’।
134. ‘ज्ञानपीि पयरस्कार -2022’ से संबंधधत कनम्नलिखित कथनों 134. Consider the following statements related to
पर कवचार कीजिए - 'Jnanpith Award-2022' -
I. र्ि भारत का सवोच्च साहित्त्र्क सम्मान पयरस्कार िै। I. It is the highest literary honor award of India.
II. Damodar Moujo was recently honored with
II. दामोदर मौजो को िाल िी में 58 वें ज्ञानपीठ पयरस्कार - 2022
the 58th Jnanpith Award - 2022.
से सम्माहनत हकर्ा गर्ा।
III. Maujo's famous novel 'Tamas' received the
III. मौजो के प्रक्तसद् उपन्द्र्ास ‘तमस’ को वषु 1983 में साहित्र्
Sahitya Puraskar in the year 1983.
पयरस्कार धमला था। Which of the above statement(s) is/are true?
उपयुथक्त में कौन-सा/से कथन सत्य है/हैं? (a) I only
(a) केिि I (b) I and II only
(b) केवल I और II (c) II and III only
(c) केवल II और III (d) All of the above
(d) उपर्युक्त सभी [a]
[a] Explanation -

-:: 70 ::-
RPT 05
व्याख्या – - Jnanpith Award is India's highest literary honor
– ज्ञानपीठ पयरस्कार भारत का सवोच्च साहित्त्र्क सम्मान पयरस्कार िै। award.
– दामोदर मौजो को िाल िी में 57 वें ज्ञानपीठ पयरस्कार 2022 से - Damodar Moujo was recently honored with the
57th Jnanpith Award 2022.
सम्माहनत हकर्ा गर्ा।
- Maujo's famous novel 'Carmelin' received the
- मौजो के प्रक्तसद् उपन्द्र्ास ‘करमेक्तलन’ को वषु 1983 में साहित्र्
Literature Award in the year 1983.
पयरस्कार धमला था।
135. UN ररपोटु के अनयसार कवश्व में सवाुधधक प्रीमैच्योर बच्चों का 135. According to the UN report, which country
िन्म ककस दे श में हुआ है? have the maximum number of premature
(a) ब्राजील babies been born in the world?
(a) Brazil
(b) कनाडा
(b) Canada
(c) भारत
(c) India
(d) नेपाल
(d) Nepal
[c] [c]
व्याख्या – Explanation -
– UN की ररपोटु के अनयसार हवश्व में सवाुधधक Premature बच्चों - According to the UN report, most premature
का जन्द्म भारत में िोता िै। babies in the world are born in India.
136. एलशया का सबसे बड़ा बायोडायवर्सिंटी पाकु कहााँ स्स्थत है - 136. Where is the largest Biodiversity Park of Asia
(a) अलथान (सूरत) situated -
(b) रानपयर (कोटा) (a) Althan (Surat)
(b) Ranpur (Kota)
(c) अमरावती (मिाराष्ट्र)
(c) Amravati (Maharashtra)
(d) पचमारी (मध्र्प्रदे श)
(d Pachmari (Madhya Pradesh)
[a]
[a]
व्याख्या – Explanation -
– एक्तशर्ा का सबसे बड़ा बार्ोडार्वर्सिटी पाकु अलथान (सूरत) में िै। - Asia's largest Biodiversity Park is in Althan
(Surat).
137. कनम्नलिखित कथनों पर कवचार कीजिए - 137. Consider the following statements -
I. ‘MH-60 R’ अमेररका द्वारा हनर्मित भारतीर् नौसेना के क्तलए I The 'MH-60R' is the latest helicopter for the
नवीनतम िेलीकॉप्टर िै। Indian Navy built by the US.
II. Zorawar is a light tank developed by DRDO
II. जोरावर, डीआरडीओ एवं L&T द्वारा हवकक्तसत लाइट टैं क िै।
and L&T.
उपयुथक्त में कौन-सा/से कथन सत्य है/हैं?
Which of the above statement(s) is/are true?
(a) केिि I
(a) I only
(b) केवल II (b) II only
(c) I और II दोनों (c) Both I and II
(d) न तो I, न ही II (d) Neither I nor II
[c] [c]
व्याख्या – Explanation -
– भारतीर् नौसेना के नवीनतम िेलीकॉप्टर MH-60 R का अन्द्र् नाम - Romeo, another name for the Indian Navy's
रोधमर्ो जो हक अमेररका द्वारा हनर्मित िै। latest helicopter MH-60R, which is manufactured
by the US.
- पवुतीर् क्षेत्रों में रक्षा बढ़ाने िेतय DRDO एवं L&T द्वारा संर्यक्त रूप
- Zorawar is a light tank jointly developed by DRDO
से हवकक्तसत लाइट टैं क जोरावर िै।
and L&T to enhance defense in mountainous
areas.
138. ‘समयद्र शलक्त-23’ ककन दो दे शों के मध्य एक कद्वपक्षीय अभ्यास 138. 'Samudra Shakti-23' is a bilateral exercise
है? between which two countries?
(a) भारत एवं इंडोनेक्तशर्ा (a) India and Indonesia
(b) India and Australia
(b) भारत एवं ऑस्रे क्तलर्ा
(c) India and China
(c) भारत एवं चीन
(d) India and Mongolia
(d) भारत एवं मंगोक्तलर्ा
-:: 71 ::-
RPT 05
[a] [a]
व्याख्या – Explanation -
– - Samudra Shakti-23 is a bilateral exercise between
‘समयन्द्र शक्तक्त-23’ भारत और इंडोनेक्तशर्ा के मध्र् एक हद्वपक्षीर्
India and Indonesia.
अभ्र्ास िै।
- Its objective is to promote interoperability and
– इसका उद्दे श्र् दोनों नौसेनाओं के मध्र् अंत:हक्रर्ाशीलता एवं आपसी
mutual cooperation between the two navies.
सिर्ोग को बढ़ावा दे ना िै।
139. ‘अन्तरराष्ट्रीय िैव कवकवधता ठदवस’ के अवसर पर प्रदे श के 139. On the occasion of ‘International Biodiversity
ककन दो वन क्षेत्रों में िंगि सफारी शयरू की गई है - Day’, Jungle Safari has been started in which
(a) जर्गढ़ अभर्ारडर्, बीड़ कन्द्जवेशन ररजवु two forest areas of the state?
(a) Jaigarh Sanctuary, Beed Conservation
(b) बीड़ कन्द्जवेशन ररजवु, जर्समंद अभर्ारडर्
Reserve
(c) बीड़ घास फूक्तलर्ाखयदु कन्द्जवेशन, जर्समंद अभर्ारडर्
(b) Beed Conservation Reserve, Jaisamand
(d) राजसमंद अभर्ारडर्, बीड़ कन्द्जवेशन ररजवु
Sanctuary
[b] (c) Beed Ghas Fuliyakhurd Conservation,
व्याख्या – Jaisamand Sanctuary
– ‘अन्द्तरराष्ट्रीर् जैव हवहवधता ददवस’ 22 मई, के अवसर पर प्रदे श के (d) Rajsamand Sanctuary, Beed Conservation
दो वन क्षेत्रों में जंगल सफारी शयरू की गई िै - Reserve
1. जर्समंद अभर्ारडर्, उदर्पयर [b]
2. बीड़ कन्द्जवेशन, झयंझन य ू Explanation -
- On the occasion of ‘International Biodiversity
Day’, 22 May, Jungle Safari has been started in
two forest areas of the state -
1. Jaisamand Sanctuary, Udaipur
2. Beed Conservation, Jhunjhunu.
140. IPL-2023 (16वााँ) के इमर्ििंग प्िेयर तथा मोस्ट वेल्यूबि 140. Which players were given the title of
प्िेयर का खिताब ककन खििाधड़यों को ठदया गया? Emerging Player and Most Valuable Player of
(a) र्शस्वी जार्सवाल तथा शयभमन हगल IPL-2023 (16th)?
(a) Yashasvi Jaiswal and Shubman Gill
(b) शयभमन हगल तथा मोहम्मद शमी
(b) Shubman Gill and Mohammed Shami
(c) शयभमन हगल तथा र्शस्वी जार्सवाल
(c) Shubman Gill and Yashasvi Jaiswal
(d) रोहित शमाु तथा हवराट कोिली
(d) Rohit Sharma and Virat Kohli
[a] [a]
व्याख्या – Explanation -
इमर्जिग प्लेर्र र्शस्वी जार्सवाल Emerging Player Yashasvi Jaiswal
मोस्ट वेलर्ूबल प्लेर्र शयभमन हगल Most Valuable Player Shubman Gill
पपुल कैप मोहम्मद शमी Purple Cap Mohammed Shami
ऑरेंज कैप शयभमन हगल Orange Cap Shubman Gill
141. 8 मई, 2023 को मैधिड ओपन टे कनस टू नाुमेंट का पयरुष एकि 141. Who won the men's singles title of Madrid
खिताब ककसने िीता? Open tennis tournament on 8th May, 2023?
(a) राफेल नडाल (a) Rafael Nadal
(b) Matthew Ebden
(b) मै्र्ू एब्डेन
(c) Carlos Alcaraz
(c) कालोस अलकराज
(d) Rohan Bopanna
(d) रोिन बोपन्द्ना
[c]
[c] Explanation -
व्याख्या – – On May 8, 2023, Carlos Alcaraz won the men's
– 8 मई, 2023 को मैधिड ओपन टे हनस टू नाुमेंट का पयरुष एकल singles title of the Madrid Open tennis
खखताब कालोस अलकराज ने जीता। tournament.
142. कनम्नलिखित में से असयमेलित ययग्म को छााँठटए – 142. Find out the odd pair out of the following –
खििाड़ी संबंधधत िेि Player Related games
(a) प्रवीण धचत्रवेल – दरपल जम्प स्पद्ाु
-:: 72 ::-
RPT 05
(b) दीहपका कयमारी – तीरंदाजी (a) Praveen – Triple Jump
(c) शैली ससिि – बैडममिटन Chitravel
(d) दे वेन्द्र झाझधड़र्ा – जेवक्तलन थ्रोअर (b) Deepika – Archery
Kumari
[c]
(c) Shailey Singh – Badminton
व्याख्या –
(d) Devendra – Javelin thrower
खििाड़ी संबंधधत िेि
Jhajhadia
प्रवीण धचत्रवेल दरपल जम्प स्पद्ाु [c]
दीहपका कयमारी तीरंदाजी Explanation -
शैली ससिि लम्बी कूद Player Related games
दे वेन्द्र झाझधड़र्ा जेवक्तलन थ्रोअर Praveen Chitravel Triple Jump
Competition
Deepika Kumari Archery
Shailey Singh Long Jump
Devendra Jhajharia Javelin Thrower
143. रािस्थान के ‘यूथ को िोड़ो’ अभभयान का संबंध ककस कवभाग 143. Rajasthan's 'Connect Youth' campaign is
से है? related to which department?
(a) भाषा एवं पयस्तकालर् हवभाग (a) Language and Library Department
(b) Department of Secondary Education
(b) माध्र्धमक क्तशक्षा हवभाग
(c) Department of Women and Child
(c) महिला एवं बाल हवकास हवभाग
Development
(d) प्रारल्म्भक क्तशक्षा हवभाग
(d) Elementary Education Department
[a] [a]
व्याख्या – Explanation -
– ‘र्ूथ को जोडो’, प्रदे श के भाषा एवं पयस्तकालर् हवभाग द्वारा – 'Connect Youth' is a campaign to connect new
संचाक्तलत पयस्तकालर्ों से नर्े पाठकों को जोड़ने के क्तलए चलार्ा readers with the libraries run by the Language
गर्ा अभभर्ान िै। and Library Department of the state.
144. रािस्थान अक्षय ऊिाु कनगम ने फ्िोटटिंग सोिर प्िांट के 144. Rajasthan Renewable energy limited has
पायिट प्रोिेक्ट हेतय कनम्नलिखित में से ककन स्थानों का चयन selected which of the following places for pilot
ककया है? project of floating solar plant?
I. Rajsamand Lake
I. राजसमंद झील
II. Lake Pichola in Udaipur
II. उदर्पयर की हपछोला झील
III. Anasagar Lake of Ajmer
III. अजमेर की आनासागर झील
IV. Jaipur's Mansagar Lake
IV. जर्पयर की मानसागर झील Code :
कूट : (a) III and IV only
(a) केिि III और IV (b) I and II only
(b) केवल I और II (c) I and III only
(c) केवल I और III (d) II and III only
(d) केवल II और III [a]
[a] Explanation -
- Rajasthan Renewable energy limited has selected
व्याख्या –
Ajmer's Anasagar lake and Jaipur's Mansagar
- राजस्थान अक्षर् ऊजाु हनगम ने फ्लोटटिग सोलर प्लांट के पार्लट
lake has selected for the pilot project of floating
प्रोजेक्ट िेतय अजमेर की आनासागर झील व जर्पयर की मानसागर
solar plant.
झील का चर्न हकर्ा गर्ा िै।
145. ‘प्रेस की स्वतंत्रता’ के संदभु में कनम्नलिखित कथनों पर कवचार 145. With reference to the ‘Freedom of the press’,
कीजिए - consider the following statements -
I. र्ि सूचकांक ‘ररपोटसु हवदाउट बॉडुसु’ (RSF) द्वारा जारी I. This index is released by 'Reporters Without
Borders' (RSF).
हकर्ा जाता िै।
II. Finland has topped this index.
II. इस सूचकांक में हफनलैंड ने शीषु पर कब्जा हकर्ा िै।

-:: 73 ::-
RPT 05
III. हवश्व प्रेस स्वतंत्रता सूचकांक में भारत 180 में से 161 वें स्थान III. India has been ranked 161 out of 180 in the
पर िै। World Press Freedom Index.
उपयुथक्त में कौन-सा/से कथन असत्य है/हैं? Which of the above statement(s) is/are false?
(a) I only
(a) केिि I
(b) I and III only
(b) केवल I और III
(c) III only
(c) केवल III
(d) II only
(d) केवल II [d]
[d] Explanation -
व्याख्या – - This index is released by 'Reporters Without
– र्ि सूचकांक ‘ररपोटसु हवदाउट बॉडुसु’ (RSF) द्वारा जारी हकर्ा Borders' (RSF).
जाता िै। - Norway has topped this index.
– इस सूचकांक में नावे ने शीषु पर कब्जा हकर्ा िै। - India has been ranked 161st out of 180 in the
– हवश्व प्रेस स्वतंत्रता सूचकांक में भारत 180 में से 161 वें स्थान पर World Press Freedom Index.
रिा िै।
146. G–7 दे शों का 49 वााँ लशिर सम्मेिन कहााँ आयोजित ककया 146. Where was the 49th summit of G-7 countries
गया? held?
(a) जम्मू कश्मीर, भारत (a) Jammu and Kashmir, India
(b) Berlin, Germany
(b) बर्लिन, जमुनी
(c) Hiroshima, Japan
(c) हिरोक्तशमा, जापान
(d) Paris, France
(d) पेररस, फ्रांस
[c]
[c] Explanation -
व्याख्या – – The 49th summit of G-7 countries was held on
– G–7 दे शों का 49 वााँ क्तशखर सम्मेलन 19-21 मई, 2023 को May 19-21, 2023 in Hiroshima, Japan.
हिरोक्तशमा, जापान में सम्पन्द्न हुआ। - G–7 includes Canada, France, Germany, Italy,
- G–7 में कनाड़ा, फ्रांस, जमुनी, इटली, जापान, र्ू.के. और संर्यक्त Japan, U.K. and the United States are included.
राज्र् अमेररका शाधमल िैं।
147. हाि ही में, ‘राष्ट्रीय महत्त्व का स्मारक’ घोकषत ककए गए तयंगनाथ 147. Recently, the Tunganath Temple, which was
मंठदर का संबंध है - declared a 'Monument of National
(a) उत्तराखडड Importance', is related to -
(a) Uttarakhand
(b) तधमलनाडय
(b) Tamil Nadu
(c) केरल
(c) Kerala
(d) आंध्र प्रदे श
(d) Andhra Pradesh
[a] [a]
व्याख्या – Explanation -
– िाल िी में, केन्द्र सरकार द्वारा ‘तयंगनाथ मंददर’ जो उत्तराखडड के - Recently, the ‘Tungnath Temple’, located in the
रूरप्रर्ाग जजले में ल्स्थत िै, को राष्ट्रीर् मित्त्व का स्मारक घोहषत Rudraprayag district of Uttarakhand, has been
हकर्ा गर्ा िै। declared a monument of national importance by
– र्ि हिन्द्दुओं के क्तलए एक मित्त्वपूणु तीथु स्थल िै। the Central Government.
- It is an important pilgrimage site for Hindus.
148. हाि ही में, चक्रवात ‘मोचा’ से प्रभाकवत म्यांमार की सहायता 148. Recently, what was the operation launched by
के लिए भारत सरकार द्वारा चिाया गया अभभयान कौनसा था- the Government of India to help Myanmar
(a) ऑपरेशन राित affected by Cyclone Mocha?
(a) Operation Rahat
(b) ऑपरेशन मदद
(b) Operation Madad
(c) ऑपरेशन करुणा
(c) Operation Karuna
(d) ऑपरेशन सयदशुन शक्तक्त
(d) Operation Sudarshan Shakti
[c] [c]
व्याख्या – Explanation -

-:: 74 ::-
RPT 05
– चक्रवात ‘मोचा’ से प्रभाहवत म्र्ांमार की सिार्ता के क्तलए भारत - Operation Karuna campaign was launched by the
सरकार द्वारा ‘ऑपरेशन करुणा’ अभभर्ान चलार्ा गर्ा। Government of India to help Myanmar affected
by Cyclone Mocha.
149. हाि ही में, कनम्नलिखित में ककन सांसदों को िोकसभा की 149. Recently, which of the following MPs were
प्राक्किन सधमकत में मनोनीत ककया गया - nominated to the Estimates Committee of the
I. हनिालचंद चौिान Lok Sabha?
I. Nihalchand Chauhan
II. पीपी चौधरी
II. PP chaudhary
III. संजर् जार्सवाल
III. Sanjay Jaiswal
IV. राज्र्वद्ु नससिि राठौड़
IV. Rajyavardhan Singh Rathore
उपयुथक्त में कौन-सा/से कथन सत्य है/हैं? Which of the above statement(s) is/are true?
(a) केिि I, II और IV (a) I, II and IV only
(b) केवल I, II और III (b) I, II and III only
(c) केवल II, III और IV (c) II, III and IV only
(d) केवल I, III और IV (d) I, III and IV only
[a] [a]
व्याख्या – Explanation -
- Nihalchand Chauhan, PP Chowdhary and
- हनिालचंद चौिान, पीपी चौधरी व राज्र्वद्ु नससिि राठौड़ को
Rajyavardhan Singh Rathore were nominated in
लोकसभा की प्राक्कलन सधमहत में मनोनीत हकर्ा गर्ा।
the Estimates Committee of the Lok Sabha.
150. नये संसद भवन के संदभु में कनम्नलिखित कथनों पर कवचार 150. With reference to the new Parliament House,
कीजिए - consider the following statements -
I. इसके अन्द्तगुत लोकसभा कक्ष को मोर हवषर् वस्तय, जबहक I. Under this, the Lok Sabha chamber has a
peacock theme, while the Rajya Sabha
राज्र्सभा कक्ष राष्ट्रीर् फूल कमल पर आधाररत िै।
chamber theme is based on the national
II. इसके वास्तयकार हवमल पटे ल िैं।
flower lotus.
III. इसका उद्घाटन 28 मई, 2023 को हकर्ा गर्ा।
II. Its architect is Vimal Patel.
उपयुथक्त में कौन-सा/से कथन सत्य है/हैं? III. It was inaugurated on May 28, 2023.
(a) केिि I और III Which of the above statement(s) is/are true?
(b) केवल II और III (a) I and III only
(c) केिि I और II (b) II and III only
(d) उपर्युक्त सभी (c) I and II only
[d] (d) All of the above
व्याख्या – [d]
Explanation -
– इसके अन्द्तगुत लोकसभा कक्ष को मोर हवषर् वस्तय, जबहक
– Under this, the Lok Sabha chamber has a peacock
राज्र्सभा कक्ष राष्ट्रीर् फूल कमल पर आधाररत िै।
theme, while the Rajya Sabha chamber theme is
- इसके वास्तयकार हवमल पटे ल िै।
based on the national flower lotus.
- इसका उद्घाटन 28 मई, 2023 को हकर्ा गर्ा। - Its architect is Vimal Patel.
- It was inaugurated on May 28, 2023.

-:: 75 ::-

You might also like